You are on page 1of 113

គណៈកមមការនិពនធ រូបវិទា

លោក ល ៉ោ លបង
៉ោ ឡង

លោក ធី សាវរន

គីមី
លោក សិត លសង
លោកស្សី បុន លជៀនសុីផល

ជីវវិទា
លោក ម៉ោម ចាន់លសៀន
កញ្ញដ បុន សុផានី
លោកស្សី ហួ ឃីម

ផផនដីវទិ ា
៉ោ ប៉ោូលីវ ីន
លោកស្សី ណាលរត
លោកស្សី គឹម បូរផល

គណៈកមមការររួរពិនិរយ លោក សិត លសង


លោក ម៉ោម ចាន់លសៀន
លោក ល ៉ោ លប៉ោងឡង

ររៀបររៀង និងចងរកង លោក ស្សី រតនៈ

i
អារមភកថា

លោករគូ-អបករគូ និងរបិយ៍មិតថអបកអានទង
ាំ អស់ជាទីលមរតី ករមងឯកសារដដលលោករគូ-អបករគូ និងរបិយ៍
មិតថអបកអានកាំពុងកាន់លៅនឹងដដលនេះ គឺជាឯកសារទី៥ដដលចងរកងបានពីវគគបាំប៉ោនរគូឧលទធសវ ិទាសាស្រសថដន
មជឈមណ្ឍលគរុលកាសលយភូមិភាគទង ាំ របលទស។ វគគបាំប៉ោនលនេះបានលធវើលៅវ ិទាសាទនជាតិអប់រ ាំ ពីថ្ថៃទី ១៧
ាំ ៦លៅទូទង
កញ្ញដ ដលថ្់ ថៃទី ២៣ កញ្ញដ ឆ្ន ាំ ២០១១ លរកាមកិចចសហរបតិបតថិការរបស់រកសួងអប់រយុ
ាំ វជន និងកីឡា និងគលរោង
ាំ វ ិទាសាស្រសថ (STEPSAM2)កបងលោលប
អប់ររគូ ុ ាំណ្ងពរងឹងសមតទភាពរគូឧលទធសបដនទមលទៀតលលើដផបក វ ិធីសាស្រសថ
បលរងៀន និងលរៀនលោយដផែកលលើការរេះរក
ិ ការសិកាស្សាវរជាវលមលរៀន និងជាំនាញកបងការសលងេ
ុ តថាបក់ ។

លដើមផីជាជាំនួយដល់ការសិកាស្សាវរជាវរបស់លោករគូ - អបករគូ និងរបិយ៍មិតថអបកអានទង


ាំ អស់ រកុមការ រ
លយើងខញុាំបានរបមូលនូវរាល់ឯកសារទង
ាំ អស់ដដលបានលរបើកងវគគ
បុ បាំប៉ោន សរោប់រគូ សាោគរុលកាសលយទង
ាំ អស់មក
ចងរកងជាលសៀវលៅលនេះលឡង
ើ ។ ករមងឯកសារលៅកបងលសៀវលៅលនេះគ
ុ ឺ ជាទុនសរោបជ
់ ួយលោករគូ -អបករគូ លអាយ
អាចដកលមែការបលរងៀនរបស់ខនល្
លួ ព េះលៅរកការបលរងៀន និងលរៀនដផែកលលើការរេះរក
ិ ដដលលផាថតលៅលលើការគិតរបស់
សិសសជាចមផង។ លៅកបងករមងឯកសារលនេះដដរ
ុ ោនប ា ញពីររលបៀបលរៀបចាំឯកសារមួយចាំនួន លដើមផីឱ្យការបលរងៀន
របស់លោករគូ-អបករគូោនសកមមភាពលែ ដូចជា កិចចដតងការបលរងៀន សនលឹកកិចចការ បលង់កាថរលខៀន និងកាំណ្ត់
សោគល់លមលរៀនជាលដើម។

រកុមការ រលយើងខញុាំសូមអធាស្ស័យនូវរាល់ ចាំណ្ុចខវេះខាតទង


ាំ ឡាយដដលអាចលកើតោនលោយយថាលហតុ
ទង
ាំ ខលឹមសារ និងបលចចកលទសកបងការចងរកងឯកសារលនេះ។

រកុមការ រលយង
ើ ខញុាំរងចាទ
ាំ ទួលការរេះគន
ិ ដ់ កលមែពស
ី ណា
ាំ កល់ ោករគូ -អបករគូ នង
ិ របយ
ិ ម
៍ ត
ិ ថអបកអានទង
ាំ អស់
លដើមផីលធវើឱ្យករមងឯកសារលនេះកាន់ដតោនសាររបលោជន៍ដល់ការលលើកកមពស់ការបលរងៀន និងលរៀនដែម លទៀត។

រគូឧលទធសថាបក់ជាតិ
និង
ាំ វ ិទាសាស្រសថ (STEPSAM2)
រកុមការ រគលរោងអប់ររគូ

ii
ោតកា

គណ្ៈកមមការនិពនន ...................................................................................................................................................... i

អារមភកថា ................................................................................................................................................................. ii

ោតិកា ................................................................................................................................................................... iii

១. កិចដច តងការបលរងៀនគរមូរបស់រគូឧលទធសថាបក់ជាតិ

១.១. រូបវ ិទា

១.១.១ សោពធ ................................................................................................................................................................................... 3

១.១.២ ចាប់ញូ តុ
៉ោ នទី១ ...................................................................................................................................................................... 9

១.១.៣ ចរនថវ ិលវល់ ......................................................................................................................................................................... 15

១.២. គីមី

១.២.១ អុកសុីត អាសុីត បាស (ភាបក់ រចោចត់ដរចេះ) ................................................................................................................... 23

១.២.២ លាយ និងការដញកលាយ .............................................................................................................................................. 29

១.២.៣ ការកាំណ្ត់ដង់សុីលតអងគធាតុរាវ ......................................................................................................................................... 35

១.៣. ជីវវ ិទា

១.៣.១ ការពិនត
ិ យទរមង់រូបផគរុាំ ោប់ដភបកែនិកសតវ (រជូក) ............................................................................................................. 43

១.៣.២ ការរ ាំភាយចាំហាយទឹករបស់រុកខជាតិ ................................................................................................................................... 50

១.៣.៣ ចលនាតាមឆនធេះ និងលរផលច


ិ ............................................................................................................................................... 56

១.៣.៤ ការសលងេតទរមង់របោប់បនថពូជរបស់រុកខជាតិោនផាេ ....................................................................................................... 67

១.៤. ដផនដីវ ិទា

១.៤.១ វគគរពេះចនធ(១) ..................................................................................................................................................................... 75

១.៤.១ វគគរពេះចនធ(២) ..................................................................................................................................................................... 81

១.៤.២ ដាំលណ្ើររបចាាំដែៃរបស់រពេះអាទិតយ........................................................................................................................................ 88

១.៤.២ វដថទឹក .................................................................................................................................................................................. 91

២. ដផនការអនុវតថការសិកាស្សាវរជាវលមលរៀន លៅកបងមជឈមណ្
ុ ឍ លគរុលកាសលយភូមិភាគ រាជធានី-លខតថ

៣.១. រាជធានីភបាំលពញ ........................................................................................................................................... 99

៣.២. លខតថបាត់ដាំបង ........................................................................................................................................... 102

៣.៣. លខតថកាំពង់ចាម ........................................................................................................................................... 105

៣.៤. ដរពដវង ..................................................................................................................................................... 108

៣.៥. លខតថតាដកវ ................................................................................................................................................. 111

ថ ល ............................................................................................................................................. 115
៣.៦. លខតថកណា

iii
១. កិចចតរងការបររងៀនគរមូរបស់រគូឧរទេសថ្នាក់ជារិ

១.១. រ ូបវិទា

១.១.១ សម្ភាធ

១.១.២ ចាប់ញូរុនទី១

១.១.៣ ចរនតវិលវល់
កិច្តច ែងការបងរងៀន

1. ង ៀវងៅរូបវ ិទ្យាថ្ននក់ ទ្យី៧


2. របធានបទ្យ៖ ម្ពាធ
3. បលង់ងមងរៀន៖ 1- ម្ពាធអងគធាែុ រ ឹង
- កំ ណែ់ និយមន័ យ ម្ពាធតាមរយៈកម្ពលំង និ ងផ្ផទរងកម្ពលំង

- អនុ វែតទ្យំនាក់ ទ្យំនងរវាង ម្ពាធ កម្ពលំង និ ងផ្ផទរងកម្ពលំង

ម្ពាធ=កម្ពលំង / ផ្ផទ

2- ការបញ្ូជ ន ម្ពាធកនុង នទនីយ៍ (Demo-lesson)

3- វា ់ ម្ពាធរប ់អងគធាែុ រាវ

- វា ់ ម្ពាធរប ់អងគធាែុ រាវងោយងរបើ ម្ព៉ាណូតម៉ា រែ


- អនុ វែត ម្ពាធរប ់អងគធាែុ រាវកនុងជី វភាពរ ់ងៅរបចំផ្ងៃ
4- ម្ពាធបរ ិយាកា

- វា ់ ម្ពាធបរ ិយាកា ងោយងរបើ បារតូ ៉ា ម៉ា រែ

- អនុ វែត ម្ពាធបរ ិយាកា កនុងជី វភាពរ ់ងៅរបចំផ្ងៃ


4. វែថុបំ ណង៖
ររូឧងទ្យទ RTTC និ ង ររុ ិ ស RTTC អាច្៖

- ងោលងោលការណ៍បា៉ា ស្កាល់បានតាមរយៈការរកង ើញពី លទ្យធផលពិ ងស្កធន៍


- ងធវើពិងស្កធន៍ អំពីការកំ ណែ់ លកខណៈបញ្ូជ ន ម្ពាធកនុង នទនីយ៍
- បកស្រស្កយការអនុ វែតផ្នការបញ្ូជ ន ម្ពាធអងគធាែុ រាវកនុងជី វភាពរ ់ងៅរបចំផ្ងៃ
5. រយៈងពល ៥០នាទ្យី
6. ម្ពារៈឧបងទ្យ ៖ កូ នបាល់១ កូ នបាល់៣ ទ្យឹ ក ឃ្ននបអុី រដូលិច្ និ ងបនាទែ់ តម៉ា ែ.
រយៈ
កមមភាពររូ ខ្លឹមស្ករងមងរៀន កមមភាព ិ ស
ងពល
1. រដឋបាលថ្ននក់
២ 2. រ ំលឹកងមងរៀន:
នាទ្យី - ងែើ ម្ពាធជាអវី? - ម្ពាធរឺ ជាកម្ពលំងកនុងមួ យ - ម្ពាធរឺ ជាកម្ពលំងកនុងមួ យខ្ននែ
ខ្ននែផ្ផទ ផ្ផទ
- ងែើ ម្ពាធម្ពនរូបមនតដូច្ - P = F/A - P = F/A
ងមតច្?
- ងែើ ម្ពាធម្ពនខ្ននែដូ ច្ងមតច្? - 1Pa = N/m2 - Pa ឬ atm

3
៥ 3.ងមងរៀនងមី
នាទ្យី - របាប់ ិ សឱ្យង ើងមកខ្នង - ិ សទំងអ ់ង ើងមកខ្នង
មុ ខ្ងដើ មបីងាយស្រ ួល ងងាែ មុ ខ្ងដើ មបីងមើ លបាែុ ភូែជាក់ ត ង

-ឥ ូវ ូមអនកទំងអ ់ោន តដលនឹ ងរែូវបងាាញ
ងងាែឧបករណ៍ទ្យី ១ ងោយ
កមា ់រប ់តកវខ្ា ់ជាងបាល់
- ងធវើការបងាាញងោយងធវើបតរម រូបទ១

បរមួលកមា ់តកវងធៀបនឹ ង - ិ សទំងអ ់ ងងាែយា៉ា ង
បាល់ យកច្ិ ែតទ្យុកោក់
- ងធវើការបងាាញរូបទ្យី ២ងោយ
ផតល់កម្ពលំងងៅងលើ ុីរាង
- របាប់ ិ សទំងអ ់រែ ប់
អងគុយតាមកតនលងងរៀងខ្លួនវ ិញ - ិ
សអងគុយតាមរកុម.
ង ើយតបងតច្កជារកុមៗ - ពិ ភាកាតាមរកុម និ ងង ើង
- តច្ក នលឹកកិ ច្ចការដល់ បងាាញលទ្យធផល
ិ ស រូបទ២
ី - ងលើកតកវកាន់ តែខ្ា ់ទ្យឹកបាញ់
- ងែើ អនក ងងាែង ើញដូ ច្ កាន់ តែខ្នលំង។
ងមតច្? ូមរូ ររូបតាមអវីតដល - ផតល់កម្ពលំងកាន់ តែខ្នលំងទ្យឹ កបាញ់
អនក ងងាែង ើញោក់ កុង
ន កាន់ តែខ្ា ់។
នលឹកកិ ច្ចការ រួច្ង ើងបងាាញ

ំនួររនលឹឹះ ងែើ ការបញ្ូជ ន ម្ពាធងៅកនុងទ្យឹ កម្ពនលកខណៈដូ ច្ងមតច្?


១០ បងងាើែ មមែិកមម ច្ងមលើយស្កមនទ្យុ ក
នាទ្យី - ងែើ អី ងវ ធវើឱ្យទ្យឹ កបាញ់ងច្ញ ? - កម្ពលំង
(រូបទ្យី ១)
- ងែើ កម្ពលំងងនឹះងកើ ែមកពី ណា? - បានមកពី ទ្យមៃន់ទ្យឹកកនុងតកវ
ងាែ់ងៅងលើទ្យឹកកនុងបាល់។
- ងែើ អី ងវ ធវើឱ្យទ្យឹ កបាញ់ងច្ញ ? - ងោយស្ករកម្ពលំង ងាែ់រប ់
(រូបទ្យី ២) រូបទ្យី៣ ផ្ដ។
- ងបើ ងយើង ងកន
ុត ុងរូបទ្យី ងាែ់ពិ - ម្ពនងលបឿនឬកមា ់មិនង ើោ
ម ន
៣ (បាល់ទំង៣ម្ពនរនធប៉ាុនៗ ម្ពាធបញ្ូជ នងៅកនុងទ្យឹ កម្ពន ងទ្យ។
ោន) ែផ្មលង ើោ
ម ន ររប់ ច្ំណុច្ទំង -ម្ពនងលបឿនឬកមា ់ង ើោ
ម ន។
ងែើ ទ្យឹកបាញ់ងច្ញពី បាល់ទំង អ ់
៣ងោយងលបឿនដូ ច្ោនឬម្ពន
កមា ់ង ើោ
ម ន តដរឬងទ្យ?
ពិ ភាកាតាមរកុម!

4
- ឱ្យែំ ណាងរកុមង ើងរាយ - ែំ ណាងរកុមនី មួយៗពនយល់
ការណ៍លទ្យធផលផ្នការ ង ែុ ផលរប ់ពួកងរតាមតបប
ពិ ភាកា ង ើយពនយល់ង ែុ វ ិទ្យាស្កស្រ ត
ផលរប ់ពួកងរ
- ួរ ិ សថ្នងែើ ម្ពនអនកច្ង់ - ិ សមួ យច្ំ នួនបតូរច្ងមលើយ
បតូរច្ងមលើយតដរឬងទ្យ? - រង រច្ងមលើយស្កមនទ្យុ កោក់ កុង

នលឹកកិ ច្ចការ

១៣ ពិ ងស្កធន៍
នាទ្យី - ឱ្យ ិ សបងងាើែបលង់ពិងស្កធន៍ -បងងាើែបលង់ពិងស្កធន៍ តាមរកុម
- បងាាញបលង់ពិងស្កធន៍ និ ង -បងាាញ និ ងពនយល់ដំងណើរការ
ពនយល់ដំងណើរការពិ ងស្កធន៍ ពិ ងស្កធន៍ តាមរកុមនិ មួយៗ។
ងោយបងាាញរងបៀបងរបើ របា ់
ម្ពារៈ(រូបទ្យី ២)។
- ឱ្យ ិ សងធវើពិងស្កធន៍ ។ - ពិ ងស្កធ និ ងកែ់ រតាទ្យិ ននន័យោក់
- រែួែពិ និែយររប់រកុម ិ ស កនុង នលឹកកិ ច្ចការ
ទំងអ ់ងដើ មបី រមួលការ
ពិ ងស្កធរប ់ ិ ស។
១០ របមូ ល និ ងវ ិភារទ្យិ ននន័យ
នាទ្យី
- រូ រតារាងលទ្យធផលងលើ - ិ សង ើងបងាាញលទ្យធផល
កាតរងខ្ៀន រប ់ងរមតងមួ យៗ

- ងរជើ ងរ ើ លទ្យធផលតដលលអ - ិ សពនយល់អំពីដំងណើរការ


និ ងមិ នលអជាងងរ ង ើយឱ្យ ពិ ងស្កធន៍ រប ់ពួកងរ
ិ សពនយល់អំពីដំងណើរការ
ពិ ងស្កធន៍ រប ់ពួកងរ
- ឱ្យ ិ សរកកំ ុ រប ់ងរ - ិ សពាយាមរកកំ ុ រប ់
ខ្លួនឯងផ្ទទល់ ងរតាមរយៈការពនយល់ពីដំងណើរ
ការពិ ងស្កធន៍ រប ់ងរ
- តាមរយៈលទ្យធផលពិ ងស្កធន៍ -ទ្យឹ កបាញ់ងច្ញពី បាល់ទំង៣ម្ពន
ងែើ អនកអាច្រកង ើញអវីខ្លឹះ? ងលបឿនដូ ច្ោនឬ ម្ពនកមា ់ង ើម
ោន។
- ងែើ វាម្ពនន័ យដូ ច្ងមតច្? - ម្ពាធម្ពនែផ្មលដូច្ោន
-ឱ្យ ិ សង្លើយ ំណួររនលឹឹះ - ង្លើយនឹ ង ំណួររនលឹឹះ

5
៥ ននិោឋន
នាទ្យី - ងលើកទ្យឹ កច្ិ ែត ិ សឱ្យ ិ ស
ងៅកនុងអងគធាែុ រាវ ម្ពាធរែូវ - ិ សពាយាមងធវើការ ននិោឋន
ងធវើការ ននិោឋនងោយខ្លួនឯង បានបញ្ូជ នងៅង ើៗ ម ោនររប់ ង ើយ រង រងៅងលើកាតរងខ្ៀន
តាមរយៈការរកង ើញរប ់ ច្ំ នុច្ទំងអ ់ - ងរបៀបងធៀបង ច្កតី ននិោឋនរួម
ពួ កងរ។ ជាមួ យនឹ ងងោលការណ៍បា៉ា ស្កា
ល់ងៅកនុងង ៀវងៅ ិ កា

៥ 4.អនុ វែតន៍
នាទ្យី ការអនុ វែតកុងជី
ន វភាពរ ់ ងៅ - ពាយាមពនយល់ងោយងរបើ
ពវផ្ងៃ បញ្ញ ែិ ផ្ត ន ម្ពាធអងគធាែុ រាវ
- របាំងរងយនត

6
នលឹកកិច្កា
ច រ

រូរនូវអវីតដលអនក ងងគែង ើញ:

រូបទ្យី ១ រូបទ្យី ២

KQ: “ងែើការបញ្ូជ ន ម្ពាធកនុងទ្យឹកម្ពនលកខណ:ដូច្ងមតច្?”

ច្ងមលើយស្កមនទ្យុក:……………………………………………………………………………………………...
ពិ ងស្កធ

រូបទ្យី ៣

រង រលទ្យធផលពិងស្កធន៍ កុងរបអប់
ន និ ងរូ ររូបរបាញ់ទ្យឹកងលើរប
ូ ទ្យី៣

- ងែើអនករកង ើញអវីខ្លឹះពីលទ្យធផលពិងស្កធន៍រប ់អនក?

……………………………………………………………………………………………………………………………
……………………………………………………………………………………………………………………………

- ូមង្លើយ ំណួររនលឹឹះ

……………………………………………………………………………………………………………………………

- ូមទញង ច្កតី ននិោឋន

………………………………………………………………………………………………………………………… ។

7
បលង់កាតរងខ្ៀន

ការបញ្ូជ ន ម្ពាធកនុងអងគធាែុរាវ

- ងែើ ម្ពាធជាអវ?

- ម្ពាធ = កម្ពលំង / ផ្ផទ បងាាញបាែុ ភូែ

-ងែើ ម្ពាធម្ពនរូបមនតដូច្ងមតច្?

-P = F/A

- ងែើ ម្ពាធម្ពនខ្ននែដូច្ងមតច្ ?

- 1Pa = N/m2 រូបទ្យី១ រូបទ្យី២

ំណួររនលឹឹះ ងែើការបញ្ជូ ន ម្ពាធងៅកនុងអងគធាែុរាវម្ពនលកខណៈដូច្ងមតច្ ?

ច្ងមលើយស្កមនទ្យុក

- ងបើ ងយើង ងាែ់ពិ ងកន


ុត ុងរូបទ្យី៣ (បាល់ទំង៣ម្ពនរនធប៉ាុនៗោន)
ងែើ ទ្យឹកបាញ់ងច្ញពី បាល់ទំង៣ងោយងលបឿនដូ ច្ោនឬ ម្ពនកមា ់ង ើោ
ម ន តដរឬងទ្យ?

- ម្ពនងលបឿនឬកមា ់មិនង ើោ
ម ន ងទ្យ។

-ម្ពនងលបឿនឬកមា ់ង ើោ
ម ន។

រូបទ្យី៣

លទ្យធផលពិងស្កធ៖ ទ្យឹ កបាញ់ងច្ញពី បាល់ទំង៣ងោយងលបឿនដូ ច្ោនឬ ម្ពនកមា ់ង ើោ


ម ន

- ងែើទ្យឹកបាញ់ងច្ញពីបាល់ទំង៣ងោយងលបឿនដូច្ោនឬ ម្ពនកមា ់ង ម ន ម្ពនន័ យយា៉ា ងដូ ច្ងមតច្?


ើោ

ម្ពនន័ យថ្ន ម្ពាធបញ្ូជ នតាមទ្យឹ ក ម្ពនែផ្មលង ើោ


ម ន ងៅរែង់ កំពូលបាល់ទំង៣។

ូមង្លើយ ំណូររនលឹឹះ

ងៅកនុងទ្យឹ ក ម្ពាធរែូវបានបញ្ូជ នងៅង ើៗ


ម ោនររប់ ច្ំនុច្ទំងអ ់។

ច្ូរងធវើការ ននិោឋន

ងៅកនុងអងគធាែុ រាវ ម្ពាធរែូវបានបញ្ូ ជ នងៅង ើៗ


ម ោនររប់ ច្ំនុច្ទំងអ ់។

8
កិច្ចតែងការបងរងៀន

1. ង ៀវងៅរូបវ ិទ្យាថ្ននក់ទ្យី៨
2. ងមងរៀន: ច្ាប់ ញូែុន
3. បលង់ងមងរៀន:
1- ច្ាប់ទ្យី១ញូែុ ន ឬច្ាប់និច្លភាព (Demo-lesson)

- កំណែ់ពីទ្យំងនារធមមជាែិ ផ្នអងគធាែុ

- ងោលច្ាប់ ទ្យីមួយញូែុ នបានតាមរយ:លទ្យធផលពិងស្កធន៍

- អនុវែតច្ាប់ទ្យី១ញូែុនកនុងជី វភាពរ ់ងៅរបចំផ្ងៃ

2-ច្ាប់ទ្យី២ញូែុ ន

- កំណែ់ច្ាប់ច្លនា ផ្នអងគធាែុ
- ងោលច្ាប់ទ្យី២ញូែុ ន F= ma
- អនុវែតច្ាប់ទ្យី២ញូែុនកនុងជី វភាពរ ់ងៅរបចំផ្ងៃ
3- ច្ាប់ទ្យី៣ញូែុន

- កំ ណែ់អំងពើ ងៅវ ិញងៅមករវាងអងគធាែុ ពីរ

- ងោលច្ាប់ទ្យី៣ញូែុ ន

- អនុវែតច្ាប់ទ្យី៣ញូែុ នកនុងជី វភាពរ ់ងៅរបចំផ្ងៃ

4. វែថុបំណង: ររូRTTC និងររុ ិ ស RTTC អាច្:


- កំណែ់ពីទ្យំងនារធមមជាែិផ្នអងគធាែុ តដលច្ង់តងរកាងលបឿនរប ់វា

- ងោលច្ាប់ ទ្យីមួយញូែុនបានតាមរយ:ការរកង ើញពីលទ្យធផលពិងស្កធន៍រប ់ពួកងរ

- អនុវែតច្ាប់ទ្យី១ញូែុ នកនុងជី វភាពរ ់ងៅរបចំផ្ងៃ

5. រយ:ងពល: ៥០ នាទ្យី
6. ម្ពារចំបាច្់: របអប់ថ្នលម្ពនពយួរអងគធាែុ ងៅខ្នងកនុង ដបទ្យឹក ុទ្យធ រកោ រ ឹង តខ្សអំងបាឹះ
កូនទ្យមៃន់។

សកមមភាពរគូ ខលម
ឹ សារ សកមមភាពសស
ិ ស
ជាំហានទ១
ី (២នាទី)
រដឋបាលថ្ននក់
. ស្សង់វតថោនសស
ិ ស . របធានថាបករា
់ យការណ្៍

9
ជាំហានទ២
ី (២នាទ)ី
រល
ាំ ឹកមមមរៀន
. លតើអវីលៅជាកោលង
ាំ . កោលង
ាំ គឺជាបុពវលហតុលធវើឱ្យវតទុ
ផាលស់ទី ផាលស់លលផឿន និងលធវើឱ្យវតទុ
ខូចរទង់រទយ។
ជាំហានទ៣
ី (៦នាទី)
លនា
ាំ ប
ាំ ញ្ញា ផាលស់ទយ
ី ត
ឺ ៗ

. ដចកឧបករណ្៍ដូចរូបភាពទី១ ដល់
សិសសរគប់រកុម។
. ឥឡវសូ
ូ មសលងេតរបអប់លនេះលពលអបក . លធវើប ា ញបាតុភូតតាមរកុម និង
ផាលស់ទីវាយឺតៗ និងលពលអបកផាលស់ទីវា សលងេតលោយខលនឯង

ោ៉ោងលលឿន . ពិភាកាតាមរកុម លហើយផថល់
. ឲ្សសិសសលធវើប ា ញបាតុភូត និងលធវើ ផាលស់ទីលលឿន
ចលមលើយដនការពិភាកា
ពិលសាធលោយខលនឯង
ួ . លពលលយើងផាលស់ទីរបអប់យឺតៗ
. លតើអបកបានសលងេតលឃើញោ៉ោងដូចលមថច វតទផា
ុ ល ស់ទីតាមរបអប់ លោយ
ខលេះ? លលផឿនដូចរបអប់ប៉ោុដនថ លពលលយើង
. ឱ្យសិសសពិភាកាតាមរកុម លហើយផថល់ ផាលស់ទីរបអប់ោ៉ោងលលឿន វតទម
ុ ិន
ចលមលើយដនការពិភាកា ផាលស់ទីតាមរបអប់លទ។
. ដចកសនលឹកកិចចការដល់សិសស
រូបទី១

សាំណួរគនលឹឹះ៖ មតើធមមជាតិរបស់អង្គធាតុមានទាំមនារយង្
៉ា ដូចមមេចមពលមយើង្ផ្លលស់ទី១?
ការបមង្កើតសមមតិកមម (១០ នាទី) ចលមលើយរព
ាំ ឹងទុក
. ឱ្យតរមុយដដលទក់ទងនិងបទ . ពិភាកា លហើយសរលសរចលមលើយ
ពិលសាធរបស់សិសស។ សាមនទុកលៅកបងសនល
ុ ឹកកិចចការតាម
. ឧ. ទូកពីរដូចោបដតោនោស
៉ោ ខុសោប ធមមជាតិរបស់អងគធាតុោនទាំលនារ រកុម (ធមមជាតិរបស់អងគធាតុោនទាំ
របសិនលបើលយើងចង់ផាលស់ទីទូកទង
ាំ ពីរ ពី រប្ង
ាំ ទប់និងចលនារបស់របអ លនាររប្ង
ាំ ទប់ និងចលនារបស់
លៅលសៃៀម លតើទូកមួយណា យផាលស់ទី ប់។ របអប់។
ជាង? . តាំណាងរកុមលឡើងពនយល់លទន
របសិនលបើទូកទង
ាំ ពីរកាំពុងផាលស់ទី ផលលោយលរបើបាំណ្ិនដនការគិត
លោយលលផឿន លសមើោប លតើទូកមួយណា តាមដបបវ ិទាសាស្រសថ។
យបញ្ឈប់ជាង? . សិសសមួយចាំនួនផថល់នូវគាំនិតលផស
. ឱ្យតាំណាងរកុមសិសសលឡើងរាយ ងៗ
ការណ្៍ពីលទនផលដដលបានពិភាកា
និងពនយល់លហតុផល។
. ឱ្យសិសសផថល់នូវគាំនិតលផសងៗលទៀត

10
ការពមិ ោធ (១០នាទ)ី សាថប់ នង
ិ សលងេត។
. ប ា ញសោភរៈពិតលៅកបងរូ
ុ បភាពទី
២ និងរូបភាពទ៣
ី បនាធប់មកសួរថា៖

១. របសិនលបើអបកទញរកោសយឺ
តៗ លតើអងគធាតុផាលស់ទីោ៉ោងដូច
លមថច? រូបទ២

២. របសិនលបើអបកទញរកោស់
ឱ្យលលឿន លតើអងគធាតុផាលស់ទីោ៉ោង
A
ដូចលមថច ? . ពិភាកា និងបាំលពញចលមលើយសាមន
៣. របសិនលបើអបកទញចុងកនធយដខស
ុ ទុកកបងសនល
ុ ឹកកិចចការ។
ចុេះលរកាមយឺតៗ លតើដខសនិងោច់លៅ . លធវើពិលសាធ លហើយបាំលពញលទនផល
រតង់ចាំណ្ុចណា? B កបងសនល ឹកកិចចការ។

៤. របសិនលបើអបកទញចុងកនធយដខស

ចុេះលរកាមឱ្យលលឿន លតើដខសនិងោច់
លៅរតង់ចាំណ្ុចណា? រូបទី៣
. ទុកឱ្យសិសសពិភាកា លដើមផីរក
ចលមលើយសាមនទុក។
. ដចកសោភរដូចលៅកបង
ុ រូបភាពទី២
និងរូបភាពទី៣ ។
. រតួតពិនិតយលមើលសិសសរគប់រកុម។

ការរបមូលទិនននយ
័ និង្វិភាគ
លទធផល(១០នាទី) ១. អងគធាតុផាលស់ទីតាមរកោស លៅ . សិសសលឡើងប ា ញលទនផលតាម
. គូសតារាងលទនផលលលើកាថរលខៀន ដដដលលយើងទញរកោសយឺតៗ។ រកុម។
. លរជើសលរសលទន
ើ ផលដដលលែបាំផុត ២. អងគធាតុធាលក់ចូលកបងដកវលៅ . ពនយល់ដាំលណ្ើរការដនការពិលសាធ

និងលទនផលដដលមិនសូវលែ លហើយ លពលដដលលយើងទញរកោស់លលឿ និងលៅសិសសលផសងលទៀត របសិនលបើ
ពួកលគលឡើងបកស្សាយពីដាំលណ្ើរការ ន។ ចាបា
ាំ ច់។
សកមមភាពរបស់ពួកលគ។
៣. ដខសោច់រតង់ចាំណ្ុចA លៅលពល
. ឱ្យសិសសរកនូវកាំហុសរបស់ពួកលគ
ដដលលយើងទញចុេះលរកាមយឺតៗ។
លោយខលនឯង។

៤. ដខសោច់រតង់ចាំណ្ុច B លៅលពល . សស
ិ សពាោមរកនូវកហ
ាំ ុសតាមរ
. តាមរយៈលទនផលពិលសាធន៍ លតើ
ដដលលយង
ើ ទញចុេះលរកាមលលឿន។ យៈដាំលណ្ើរការពលិ សាធន។

អបកអាចរកលឃញ
ើ អវខ
ី លេះ?

លសចកថស
ី នបោ
ិ ឌ ន (៥នាទ)ី ពុលាំ នាលចាបទ
់ ១
ី របសញ
់ ូ តុន . សស
ិ សពាោមទញលសចកថី
. សរមបសរមួលសស
ិ សលដើមផីទញ សនបោ
ិ ឌ នលោយខលនឯង
ួ លហយ
ើ សរ
លសចកថស
ី នបោ
ិ ឌ ន លោយខលនឯង
ួ លសរកបងលៅលល
ុ កា
ើ ថ រលខៀន។

11
តាមរយៈលទនផលពលិ សាធន។

. សាំលោគ និងសរលសរលសចកថី
សនបិោឌនលលើកាថរលខៀន។
. ប ា ញបាតុភូតកបងល
ុ ាំនាប
ាំ ញ្ញា
លឡើងវ ិញ រួចឱ្យសិសសលឆលើយសាំណ្ួរ . លឆលើយលៅនឹងសាំណ្ួរគនលឹេះ។

គនលឹេះ
ជាំហានទី៤
អនុវតេមៅកនុង្ជីវភាពរស់មៅរបចថ្ាំ ថៃ . ពាោមពនយល់លោយលរបើបញ្ញដតថ
. លហតុអវីបានជាលៅលពលឡានចាប់ និចលភាព។
លផថើមលចញដាំលណ្ើរលៅមុខ លយើងោន
ទាំលនារចង់ដួលលៅលរកាយ?

ជាំហានទី៥ លហតុអបា
វី នជាលយង
ើ កព
ាំ ុងរតជ
់ ាំពុប
កច
ិ ចការផធេះ លជើង លយង
ើ ដួលថាេបម
់ ុខ?

លហតុអវីបានជាលយើងលដើរជាន់សាំបក
លចក លយើងរអិលដួលផាៃរលរកាយ?

12
សនលឹកកិចចការ

ផាលស់ទីយឺតៗ ផាលស់ទីលលឿន

រូបទី១

សាំណ្ួរគនលឹេះ៖ លតើធមមជាតិរបស់អងគធាតុោនទាំលនារដូចលមថចលពលលយើងផាលស់ទីវា?
សរលសរចលមលើយសាមនទុករបស់អបក

ពិលសាធន៍

B
រូបទី៣
រូបទី២

ចលមលើយសាមនទុក A ឬ B លទនផលពិលសាធន៍
១. របសិនលបើអបកទញរកោសលៅមុខយឺតៗ លតើ
រូបទី អងគធាតុផាលស់ទីលៅណា?
២ ២. របសិនលបើអបកកន្រ្នាេក់រកោសលៅមុខ លតើអងគ
ធាតុផាលស់ទីលៅណា?
៣. របសិនលបើអបកទញកនធយដខសចុ
ុ េះលរកាមយឺតៗ
រូបទី លតើដខសោច់រតង់ណា?
៣ ៤. របសិនលបើអបកកន្រ្នាថក់កនធយដខសចុ
ុ េះលរកាម
លតើដខសោច់រតង់ណា?

សរលសរអវីដដលអបករកលឃើញ និងទញលសចកថីសនបិោឌនលហើយលឆលើយសាំណ្ួរគនលឹេះ៖

13
រ ំលឹកងមងរៀន : បលង់កាតរងខ្ៀន III. រប ិនងបើ អនកទញច្ុងកនទុយតខ្សច្ុឹះងរកាមយឺែៗ
ងែើតខ្សនឹងោច្់ងៅរែង់ច្ំនុច្ណា ?
-ងែើអីវងៅជាកម្ពលំង ? ពិងស្កធ:- បងាាញ ម្ពារៈជាក់ត ង

រកុម ច្ងមលើយស្កមន លទ្យធផល
កម្ពលំងរឺជាបុពវង ែុងធវើ ឱ្យវែថុផ្ទល ់ទ្យី ផ្ទល ់បូរងលបឿន
ត A ទ្យុក ពិងស្កធន៍

និងងធវើឱ្យវែថុខ្ូច្រទ្យង់រទយ។ 1
B
2
បងាាញបាែុភូែ 3
I. រប ិនងបើ អនកទញរកោ យឺែៗ ងែើអងគធាែុ
4
ផ្ទល ់ទ្យីយឺែៗ ផ្ទល ់ទ្យីយា៉ាងដូច្ងមតច្ ?
IV. រប ិនងបើ អនកទញច្ុងកនទុយតខ្សច្ុឹះងរកាម
រកុម ច្ងមលើយស្កមន លទ្យធផល
ឱ្យងលឿនងែើតខ្សនឹងោច្់ងៅរែង់ច្ំនុច្ណា ?
ទ្យុក ពិងស្កធន៍
រកុម ច្ងមលើយស្កមន លទ្យធផល
1
ទ្យុក ពិងស្កធន៍
2
1
ផ្ទល ់ទ្យីងលឿន 3
2
4
3
II. រប ិនងបើ អនកទញរកោ ឱ្យងលឿន ងែើអងគធាែុ
4
ផ្ទល ់ទ្យីយា៉ាងដូច្ងមតច្ ?
ននិោឋន
KQ. ងែើធមមជាែិរប ់អងគធាែុ ម្ពនទ្យំងនារយា៉ា ងដូច្ងមតច្ រកុម ច្ងមលើយស្កមន លទ្យធផល
…………………………………………………
ទ្យុក ពិងស្កធន៍
ងពលងយើងផ្ទល ់ទ្យីវា ?
1 …………………………………………………
ច្ងមលើយស្កមនទ្យុក :
2 ង្លើយនឹង ំណួររនលឹឹះ :

3 …………………………………………………
4 …………………………………………………

14
កិចចដតងការបលរងៀន
១. សរោប់បលរងៀនៈ សិសសថាបក់ទី៩

២. លមលរៀនៈ ចរនថវ ិលវល់

៣. រយលពលៈ ៥០នាទី

៤. វតទប
ុ ាំណ្ងៈ សិសសអាច

. ពិណ្នាអាំពីបាតុភូតចរនថវ ិលវល់បានោ៉ោងរតឹមរតូវ។

. ប ា ញឱ្យលឃើញថាបាតុភូតចរនថវ ិលវល់លកើតលឡើងលោយសារ ភាពខុសោបរវាងសីតុណ្ាភាពតាមរយៈ

ពលិ សាធន៍ យៗ។

. អនុវតថកងការោក
បុ ់របព័ននកលតថ និងោស
៉ោ ុីនរតជាក់កងផធ
បុ េះបានោ៉ោងរតឹមរតូវ។

៥. សោភរៈ

របអប់ ធូប សនលឹកផាលស់សធិចថាល សេត


ុ សនលឹកអាលុយមីញូ ម
៉ោ លជើងទាំរ ទឹក(ធមមតា និងពណ្៌) ដទម៉ោូដម៉ោរត ដដក
លកស ពិល ទឹក និងដកវថាល។

៦. ដាំលណ្ើរការបលរងៀន

សកមមភាពរគូ ខ្លឹមោរមមមរៀន សកមមភាពសិសស


រដឋបាលថ្ននក់
. រតួតពិនិតយវតថោនសិសស . អងគយសា
ុ ថ ប់ោ៉ោងលសៃៀមសាៃត់
រល
ាំ ឹកមមមរៀន
. លតើោស
៉ោ ោឌជាអវី? . ោស
៉ោ ោឌគឺោស
៉ោ កបងមួ
ុ យខាបតោឌ . ោស
៉ោ កបងមួ
ុ យខាបតោឌ
P=m/v
. ចាំល េះទឹក និងខយល់មួយណាោន . ទឹក
ោស
៉ោ ោឌធាំជាង?
. ចុេះទឹក និងដុាំទឹកកកោ៉ោងលម៉ោច . ទឹក
ដដរ?
. លហតុអវីបានជាដុាំទឹកកកដតងដតអ . អងគធាតុដដលោនោស
៉ោ ោឌតូចអ . ដុាំទឹកកកោនោស
៉ោ ោឌតូចជាង
ដណ្ថតលលើទឹក? ដណ្ថតលលើអងគធាតុដដលោនោស
៉ោ ធាំ ទឹក

15
មមមរៀនថមី
. របាប់សិសសឱ្យខិតជិតតុរគូលដើមផី . សលងេត
យស្សួលសលងេតបាតុភូត។ ោក់
ទឹកមួយដកវលៅលលើតុ រួចយកទឹក
ពណ្៌មួយដុាំោក់ចូលកបងដកវទ
ុ ឹក។ ទ្យឹកកកព៌ណ

. ឱ្យសិសសសលងេតមួយនាទី លហើយ . សលងេតលឃើញទឹកកកដដលរោយ


រតឡប់លៅកាន់កដនលងអងគយលដើ
ុ ម ធាលក់ចុេះលរកាម
មវ ិញ រួចឱ្យោត់គូរតាមអវីដដលោត់ . គូររូបោក់កងសនល
បុ ឹកកិចចការ
សលងេតលឃើញោក់សនលឹកកិចចការ។ រូបទ្យី ១

. លហតុអវីបានជាទឹកកកដដលរោយ . លរកាមលពលរោយរហូតដល់សីតុ . វាោនោស


៉ោ ោឌធាំជាង
ធាលក់ចុេះលរកាម? ណ្ាភាពធាំជាងទឹកលៅសីតុណ្ាភាព . សីតុណ្ាភាពទបជាង
ធមមតារបដហល ២៧០។

. លរៀបចាំពិលសាធន៍មួយលទៀតលោយ . សាថប់
លរបើរបអប់ោបពណ្៌លមមដផបកខាង
តផសង
កបង
ុ លហើយដផបកខាងមុខលរបើសនលឹក
ផាលស់សធិចថាល លដើមផី យស្សួលសលងេ
ត។ ចុេះរននតូចមួយចាំកណា
ថ លរបអប់
រូបទ្យី ២ ធូប
លដើមផីោក់ធូប។
. របាប់សិសសឱ្យខិតជិតតុរគូលដើមផី . លឡើងមកឈរជុាំវ ិញតុរគូ
យស្សួលសលងេតបាតុភូត។ ោក់
ទឹកមួយដកវលៅលលើតុ រួចយកទឹក
ពណ្៌មួយដុាំោក់ចូលកបងដកវទ
ុ ឹក
. អុចធូប លហើយោក់ចូលតាមរនន . សលងេតលឃើញដផសងលហើរលឡើងលលើ។
ដដលបានលចាេះរួច។ ឱ្យសិសសសលងេត
លមើលបាតុភូត រួចរូបតាមអវីដដលោត់
សលងេតលឃើញោក់សនលឹកកិចចការ។
. លហតុអវីបានជាដផសងធូបលហើរលឡើង . សីតុណ្ាភាពខស់លធវើឱ្យដផសងោន ៉ោ ោឌតូចជាងខយល់ជុាំវ ិញ
. ោស
លលើ? ៉ោ ោឌតូចជាងខយល់លៅជុាំវ ិញ
ោស . សីតុណ្ាភាពខពស់
លហើយលហើរលឡើងលលើ។ . លតថ

សមមតិកមម
. ដបងដចកសិសសជា៥រកុម . ដផសងដដលលៅដផបកខាងលលើសទិត . សិសសចូលអងគយតាមរកុ
ុ ម
លៅខាងលលើដដដល

16
. ឱ្យសស
ិ សសរលសរចលមលយ
ើ សាមនទុក . ដផសងដដលលៅដផបកខាងលលផា
ើ ល សទ
់ ី . ពភា
ិ កាោបតាមរកុមរួចសរលសរ
របស់ពួកលគោក់កងសនល
បុ ឹកកិចចការរួច ចុេះលរកាម ចលមលើយរបស់ពួកលគកបងសនល
ុ ឹកកិចចការ
លហើយឱ្យតាំណាងរកុមនីមួយៗលឡើង លរៀងខលន។

សរលសរចលមលើយរបស់ពួកលគសរលសរ
ោក់លលើកាថរលខៀន។
. ឱ្យសិសសពនយល់លហតុផលរបស់ . ដផសងវ ិលចុេះលរកាម លឡើងលលើបលងេើត . ដាំណាងរកុមលឡើងសរលសរលលើកាថរ
ពួកលគ បានជាដខសបិទ លខៀន។
. ដាំណាងរកុមលឡើងបកស្សាយ។

ពិមោធន ៍
. ដចកឧបករណ្៍ពិលសាធន៍ដល់រកុម . ទទួលសោភរ និងលរៀបចាំលធវើពិលសាន៍
នីមួយៗនូវ របអប់១ ដដកលកស១ និងសលងេតោ៉ោងយកចិតថទុកោក់
ធូប ទឹកកក ពិល១។ លដើមផីរកចលមលើយសាំណ្ួរគនលឹេះ។
. លតើអបកសលងេតលឃើញដូចលមថច? . ដផសងធូបវ ិលវង់បលងេើតបានជាកង
តផសង ពីរ។
. លតើអបកអាចពនយល់បាតុភូតដដល . ដផសងធូបលតថស្សាលលហើរលឡើងលលើ
អបកបានលឃើញបានដដរឬលទ? ប៉ោេះនឹងចាំហាយទឹកកករតជាក់ធាលក់
រូបទ្យី ៤
ធូប ចុេះលរកាមលហើយប៉ោេះ នឹងធូបលតថលឡើង
លលើវ ិញដដដល។
. ដណ្នាស
ាំ ិសសពិលសាធន៍មថងលទៀត . លរៀបចាំលធវើពិលសាធន៍មថងលទៀតវាស់
លហើយលធវើការវាស់សីតុណ្ាភាពលដើមផី សីតុណ្ាភាពដផបកខាងលលើ និងដផបក
លផធៀងផាធត់ចលមលើយរបស់ពួកលគ។ ខាងលរកាមបីដងរួចរកតដមលមធយម។

របមូលទិនននយ
័ និង្វិភាគ
. គូរតារាងលទនផលសរោប់រកុម . តាំណាងរកុមនីមួយៗសរលសរលទន
នីមួយៗលៅលលើកាថរលខៀន។ ផលោក់កងតារាងលល
បុ ើកាថរលខៀន
លហើយបកស្សាយលទនផលដដលលគទ
តទ្យម៉ាូតមរែ
តផសង ទួយបាន។
. តាមលទនផលពិលសាធន៍របស់អបក . សិសសលឆលើយតប និងសាំណ្ួរគនលឹេះ
លតើអបកអាចលឆលើយសាំណ្ួរគនលឹេះបាន
រូបទ្យី ៤
ធូប
ឬលទ?
សននិដ្ឋឋន
. លលើកទឹកចិតថសិសសឱ្យសិសសសនបិ . ដផសងធូបោនសីតុណ្ាភាពខពស់លធវើ
ោបនលោយខលនឯង។
ួ ោស
៉ោ ោឌរបស់វាតូច វាលហើរលឡើង
លលើ។ លពលវាោនប៉ោេះនិងចាំហាយទឹក
កក ចុេះរតជាក់ោស
៉ោ ោឌធាំធាលក់ចុេះ

17
លរកាមវ ិញ បាតុភូតលនេះលៅថាចរនថ
វ ិលវល់។
. សនបិោបនរួមលៅលលើកាថរលខៀន . ចរនថវ ិលវល់វាលកើតោនដតចាំល េះ . សិសសពាោមលធវើការសនបិោឌន
ម៉ោូលលគុលសនធនីយ៍(ឧសម័ន និងអងគ លោយខបនឯង
ួ រួចសរលសរលៅលលើកាថរ
ធាតុរាវ)។ ចរនថវ ិលវល់មិនអាចលកើត លខៀន។
ោនចាំល េះអងគធាតុរងលទ។

ពរង្ឹង្ពុទធិ
. រតឡប់លៅ Jisho Teji . សិសសបកស្សាយបានរតឹមរតូវ
បណ្
េ មាំ ផញើ
. ឱ្យសស
ិ សលធវព ើ វ ិញ
ើ លិ សាធនល៍ ឡង . សាថប់
លោយបថរទ
ូ តា
ី ងាំ មកខាងវ ិញ។

18
សនលឹកកិចចការ

តផសង ???

ទ្យឹ កកកព៌ ណ

រូបទ្យី ១ ធូ ប

សាំណ្ួរគនលឹេះ៖ លតើនឹងោនអវីលកើតលឡើងលបើលគោក់ទឹកកកលៅលលើរបអប់?

សមមតិកមម៖

ធូប
រូបទ្យី ៣

ពិលសាធន៍៖

តផសង តទ្យម៉ាូ
...??? តម៉ា រែ

ធូ ប
ធូប
រូបទ្យី ៤ រូបទ្យី ៥

តារាងទិនបន័យ៖

0
សីតុណ្ាភាព( C) ១ ២ ៣ មធយម
ដផបកខាងលលើ

ដផបកខាងលរកាម

សរលសរនូវអវីដដលបានរកលឃើញតាមរយៈពិលសាធន៍

19
សនបិោឌន៖

លឆលើយសាំណ្ួរគនលឹេះ៖

20
២.១. គីមី

២.១.១ អុកសុរ
ី អាសុរ
ី បាស (ភ្នាក់ងារចម្ភចរ់តរចែះ)

២.១.២ លាយ និងការតញកលាយ

២.១.៣ ការកំណរ់ដង់សរី ុ រអងគធារុរាវ


កិចចដតងការបលរងៀន
១. សរោប់បលរងៀនៈ សិសសថាបក់ទី៩

២. លមលរៀនៈ អុកសុីត អាសុីត និងបាស

៣. រយលពលៈ ៥០នាទី

៤. វតទប
ុ ាំណ្ងៈ សិសសអាច

. ពិពណ្៌នាអាំពីដាំលណ្ើរការសោែតដរចេះរបស់សូលុយសយងផធ
ុ េះបាយលលើវតទដដលលធវ
ុ ើពីលោហៈបានរតឹមរតូវ
តាមរយៈពិលសាធន៍ សមស្សប។

. កាំណ្ត់លកខខណ្ឍពិលសាធន៍ លធវើពិលសាធន៍ របមូលទិនបន័យ និងលរបៀបលធៀបបលងេើតរបកគាំលហើញសមស្សប


តាមរយៈពិលសាធន៍។

. លលក
ើ ទក
ឹ ចត
ិ ថសស ិ ថលរៀនវ ិទាសាស្រសថ លោយផាភាជបប
ិ សឱ្យចូលចត ់ ញ្ដតវិ ិទាសាស្រសថលៅនង
ឹ ជីវភាពរបចាដាំ ែៃ។

៥. សោពរៈ

ដដលលោលដរចេះ ដកវថាល ដលងេៀប អងគធាតុរាវផធេះបាយដូចជា ទក


ឹ លខមេះ ទក
ឹ រកូច្ម ទក
ឹ សាបូោងបងគន់ ទក

សាបូោងចាន លរបង្ កូកាកូឡា។

៦. ដាំលណ្ើរការបលរងៀន

សកមមភាពរគូ ខ្លឹមោរមមមរៀន សកមមភាពសិសស


រតួតពិនិតយវ ិន័យ សណា
ថ ប់ធាបប់ និង ជាំហានទី១ សិសសអងគយតាមរកុ
ុ ម លរតៀមខលន

វតថោនសិសស។ លាំនឹងថាបក់(២នាទី) សរោប់លមលរៀនដែៃលនេះ។
ជាំហានទី២
. អវីលៅជាអុកសុីត? លតើអុកសុីតលកើត រឭកលមលរៀន
ាំ (៣នាទី) . អុកសុីត គឺជាសោសធាតុដដលផសាំ
លឡើងោ៉ោងលម៉ោច? ពីធាតុណាមួយជាមួយអាតូមអុកសុី
តដសន។ វាលកើតលឡើងកាលណាធាតុ
មួយ របតិកមមជាមួយអុកសុីដសន។
. អវីលៅជាអាសុីត? . អាសុីតគឺជាសូលុយសយង
ុ ដដល
ោន pH < 7, បថរពណ្
ូ ៌ រកោសតួ
ណ្ឺសុលលៅជារកហម...។
ជាំហានទី៣
. លតើោនអវីលកើតលឡើងចាំល េះវតទលធវ
ុ ើពី លមលរៀនែមី (៤០នាទី) . វានឹងលឡើងដរចេះ
លោហៈរបសិនលបើលយើងលរបើលោយ លសចកថីលផថើមលមលរៀន (១០នាទី)
ខវេះការដែទ?
ាំ

23
. ចូររបាបវ់ តទលធវ
ុ ព ើ លី ោហៈដដលអបក . កាប
ាំ ត
ិ ដដកលោល...
សាគល់។
. ប ា ញវតទលធវ
ុ ើពីលោហៈដដលលឡើង
ដរចេះមួយចាំនួន
. លតើដរចេះផថល់ផលវ ិបាកអវីខលេះ? . វតទន
ុ ឹងពុកផុយ ដបកបាក់...
. លតើលយើងលធវើោ៉ោងដូចលមថចលដើមផី . សោែតលោយដុសខាត់ជាមួយនិង
ជរមេះដរចេះ? អងគធាតុរាវលផសងៗ។
សាំណួរគនលឹឹះ៖ លតើអងគធាតុរាវអវីខលេះដដលអាចលរបើលដើមផីសោែតដរចេះលអាយោនរបសិទនភាព?
. លលក
ើ ទក
ឹ ចត
ិ ថសស
ិ សឱ្យបលាំ ពញ . ការបលងេើតសមមតក
ិ មម (៥នាទី) ពភា
ិ កាតាមរកុម លដើមផីបលងេើតសមមតិ
សមមតក
ិ មមរបសល់ គលៅកបងសនល
ុ ក
ឹ កច
ិ ច កមម នង
ិ បលាំ ពញសនលក
ឹ កច
ិ ចការ នង

ការ។ ប ា ញគន
ាំ ត
ិ របសល់ គលៅរកុមដដទ។
. ឱ្យសស
ិ សប ា ញសមមតក
ិ មមរបស់
ពួកលគតាមរកុម។
. លលក
ើ ទក
ឹ ចត
ិ ថសស
ិ សឱ្យបលងេើតបលង់ . ការបលងេើតបលង់ពលិ សាធន៍ (៥នាទី) ពភា
ិ កាតាមរកុម លដើមផីបលង់
ពលិ សាធន៍ នង
ិ ប ា ញគន
ាំ ត
ិ របសល់ គ ពលិ សាធន៍ នង
ិ ប ា ញដលរ់ កុមដ
ដលរ់ កុមដដទ។ ដទ។
. ពនយលប
់ ដនទម អព
ាំ ដ
ី ាំលណ្ើរការ
ពលិ សាធន៦ បលចចកលទស នង
ិ បរមុង
របយត
័ បដលស
់ ស
ិ ស។
ផថល់សោភរពិលសាធន៍ និងសរមប . ការពិលសាធ(១០នាទី) លធវើពិលសាធន៍តាមរកុម និងកត់រតា
រមួលសកមមភាពសស
ិ ស។ បលាំ ពញចលនាលេះកបងសនល
ុ ក
ឹ កច
ិ ចការ។
លលើកទឹកចិតថសិសសឱ្យលធវើបទប ា ញ . ការលធវើបទប ា ញ(១០នាទី) លធវើបទប ា ញអាំពីអវីដដលពួកលគ
អាំពីអវីដដលពួកលគទទួលបាន និងរក ទទួលបាន និងរកលឃើញពីពិលសាធន៍
លឃើញពីពិលសាធន៍ (លសចកថីសនបិោឌន)
ជាំហានទី៤
លតើលយើងគួលធវើដូចលមថច លដើមផីសោែត រឭកលមលរៀន
ាំ (៣នាទី) . លយើងអាចសោែតវាលោយោក់រតាាំ
កាប
ាំ ិតដដលលឡើងដរចេះលនេះ? វាកបងទ
ុ ឹកោងបងគន់របដហល១០
ឧបោថា ដដកដរចេះគឺជា Fe2O3, ចូរ នាទី រួមោងជាមួយទឹក
សរលសររបតិកមម របសិនលបើវាោន Fe2O3(s) + 6HCL(aq)
របតិកមមជាមួយអាសុីត HCL(aq)? 2FeCl3 + 3H2O(l)
ជាំហានទី៥៖
លៅផធេះរបស់អបក ចូរពិនិតយលមើល កិចចការផធេះ (២នាទី) កត់រតាកិចចការផធេះ។
បដនទម ថាលតើណាខលេះលទៀតដដលអាច
លរបើសរោប់សោែតដរចេះបាន និងលធវើ
ការប ា ញលទនផលរបស់អបកលៅ
លោង
៉ោ លរកាយ។

24
សនលឹកកិចចការ

ភានក់ងារចរមឹះផរចឹះ

មសចកេីមផេើម
លតោ
ើ នអវលី កត
ើ លឡង
ើ ចលាំ េះឧបករណ្៍លោហៈ លបលើ យង
ើ លរបម
ើ ន
ិ ដែ?.......................................................................

ឧបករណ្៍លោហៈអវីខលេះ ដដល យលឡើងដរចេះ? ..................................................................................................

លតើដរចេះគឺជាអវី? ...................................................................................................................................................

សាំណ្ួរគនលឹេះ លតើអងគធាតុរាវអវីខលេះដដលអាចលរបើលដើមផីសោែតដរចេះលអាយោនរបសិទនភាព?

សមមតិកមម៖

ខញុាំគិតថា ..................................................................................................................................................

................................................................................................................................................................

ពីលរ េះ ................................................................................................................................................។

បលង់ពិលសាធន៍៖ លតើអបកបញ្ញជក់សមមតិកមមរបស់អបកោ៉ោងដូចលមថច?

លទនផលពិលសាធន៍៖

សូលុយសយង
ុ ការសលងេតលមល
ើ ដផធលោហៈដដលរតាក
ាំ ងសូ
បុ លុយសយង

សនបិោឌន

25
បលង់កាថរលខៀន
ភានក់ងារចរមឹះផរចឹះ

មសចកេមី ផេើម
លតើោនអវីលកើតលឡើងចាំល េះឧបករណ្៍លោហៈ លបើលយើងលរបើមិនដែ?................វានឹងលឡើងដរចេះ ពុកផុយ................

ឧបករណ្៍លោហៈអវីខលេះ ដដល យលឡើងដរចេះ? ..............កាប


ាំ ិត ដដកលោល.......................................................

លតើដរចេះគឺជាអវី? .........ដរចេះគឺជាអុកសុីតលោហៈ ដដលលកើតលពលប៉ោេះជាមួយខយល់

សាំណ្ួរគនលឹេះ លតើសូលុយសយងផធ
ុ េះបាយអវីខលេះដដលអាចលរបើសរោប់ជរមេះដរចេះបាន?
សមមតិកមម(ឧទហរណ្៍)

ខញុាំគិតថា ..................ទឹកលខមេះអាចជរមេះដរចេះបាន,............

ពីលរ េះ .............វាោនជាតិអាសុីត..........។

បលង់ពលិ សាធន៖៍ លតអ


ើ បកបញ្ញជកស
់ មមតក
ិ មមរបសអ
់ បកោង
៉ោ ដូចលមថច?

លយើងអាចសាកលផងជាមួយ ដដកលោលដរចេះតាមដាំលណ្ើរការខាងលរកាម៖
. លរជើសលរសដដកលោលព
ើ ីរ ដដលដរចេះរបដហលោប។ ោក់ដដកលោលមួយចូលកបងទ
ុ ឹកលខមេះ ដតមួយលទៀតមិនោក់ចូល
លទ ទុកលដើមផីលរបៀបលធៀប។ របដហល ១០នាទី លរកាយ ស្សង់យក ដដកលោលលចញពីទឹកលខមេះ រូចលរបៀបលធៀបជាមួយ
ដដកលោលដដលទុកលៅខាងលរត។

លទនផលពិលសាធន៍៖

សូលុយសយង
ុ ការសលងេតលមើលដផធលោហៈដដលរតាក
ាំ ងសូ
បុ លុយសយង

ឧ. ទឹកលខមេះ ដរចេះខលេះបានសោែតលចញពីដដកលោល

សនបិោឌន

សូលុយសយងដដលោនជាត
ុ ិអាសុីត ដូចជា ទឹកលខមេះ ទឹករកូច្ម សាបូោងបងគន់ ..... អាចលរបើសោែតដរចេះបាន។
លនេះគឺលោយសារដរចេះ (អុកសុីតលោហៈ Fe2O3) ោនរបតិកមមជាមួយអាសុីត។
Fe2O3(s) + 6HCL(aq) 2FeCl3 + 3H2O(l)

26
កំណរ់សគា
ំ ល់រមររៀនសរម្ភប់រគូ

1. មូលដ្ឋឋនរគឹឹះ
លមលរៀនដផែកលលើការរេះរកដដលោនច
ិ ាំណ្ងលជើងថា “ភាបក់ រជរមេះដរចេះ” គឺរតូវបានលរៀបចាំលឡើងសរោប់ថាបក់
ទ៩
ី ។ របធានបទលនេះទក់ទងលៅនឹងលមលរៀន អុកសុីត អាសុីត និងបាស។ ដូចោបលៅនឹងលមលរៀនរេះរកដដទ

លទៀតដដរ សិសសរតូវោនចាំលណ្េះដឹងវ ិទាសាស្រសថចាបា
ាំ ច់មួយចាំនួន លបើពុាំលនាេះលទលមលរៀនលនេះនឹងពុាំអាចអនុវតថ
បានលោយលជាគជ័យលឡើយ។ ដូលចបេះ មុននឹងបលរងៀនលមលរៀនលនេះ សិសសរតូវបានគិតថា បានលរៀនរួចលហើយ
នូវបញ្ញដតថសឋីអាំពី អុកសុីត អាសុិត និងបាស។
2. ការដាំម ើង្សមាារ
លមលរៀនលនេះ លរបើរបាស់សោភរឧបលទសសាមញ្ដបាំផុត។ លយើងរតូវការដតដដកលោលដដលោនដរចេះមួយចាំនួន
និងអងគធាតុរាវដដលអាចរកបានពីផធេះបាយដូចជា លរបង្ ទឹកលខមេះ ទឹករកូច្ម ទឹកោងបងគន់ ទឹកបរសុ
ិ ទន
។ល។ ជាការពិត រគូអាចលរបើរបាស់សូលុយសយងលផសងៗ
ុ លទៀតដដលអាចរកបានលៅផធេះបាយ សរោប់ឱ្យ
សិសសលធវើលតសថ។ វាអាចទក់ទញចាំណាប់អារមមណ្៍សិសសោ៉ោងខាលង
ាំ របសិនលបើលយើងអាចប ា ញ ឬលរបើ
របាស់សោភរលផសងៗដដលទក់ទងលៅនឹងជីវភាពរបចាដាំ ែៃរបស់សិសស សរោប់សកមមភាពកបងថា
ុ ប ក់លរៀន។

3. ដាំណឹកនាមាំ មមរៀន
គួររឭកដដរថា
ាំ លបើលទេះជាលយើងោនសោភរឧបលទសរគប់រោន់ក៍លោយ លទនផលសិការបស់សិសសអាន នឹង
មិនទទួលបានលែដដរ របសិនលបើដាំណ្ឹកនាលាំ មលរៀនមិនបានសមស្សប។ ដូលចបេះ រគូរតូវបានលគរព
ាំ ឹងទុកថា ជា
អបកសរមបសរមួលោ៉ោងសកមមលៅកបងថា
ុ ប ក់ និងោនសមតទភាពប ា ញដដលអាចទក់ទញ អារមមណ្៍
សិសស។ ោនគាំនិតជាលរចើនដដលអាចអនុវតថបានឱ្យោនរបសិទនភាព។ ខាងលរកាមលនេះគឺជាឧទហរណ្៍មួយ
- រគូចាប់លផឋើមនិោយលរឿង ឬលហតុការណ្៍លផសងៗដដលទក់ទងនឹងការលឡើងដរចេះ និងបញ្ញារបស់វា លៅ
កបងជី
ុ វភាពរបចាដាំ ែៃ ឧទហរណ្៍ដូចជា ឡាន សាំណ្ង់អាោរ សោភរផធេះបាយ ។ល។ លោយចាប់លផឋើម
សួរសិសសថា លតើោនអវីលកើតលឡើងចាំល េះវតទលធវ ុ ើពីលោហៈ របសិនលបើវារតូវបានលរបើរបាស់លោយ ពុាំោន
ការដែទរាំ តឹមរតូវ? រគូអាចបដនទមបញ្ដតថវ ិទាសាស្រសថបនថិច ដូចជាថា លតើដរចេះគឺជាអវី? វាលកើតលឡើងោ៉ោង
ដូចលមឋច?
- លោយដផែកលលើចលមលើយតបរបស់សិសស រគូអាចឱ្យសិសសនិោយអាំពីការលឡើងដរចេះដនវតទលធវ
ុ ើពីលោហៈ
លៅផធេះរបស់ពួកលគ ដដលលគបានជួបរបទេះ។
- នាស
ាំ ស
ិ សពភា
ិ កា អព
ាំ ប
ី ញ្ញាដដលបណា
ឋ លមកពកា
ី រលឡង
ើ ដរចេះ នង
ិ ថាលតលើ យង
ើ រតូវលោេះស្សាយោង
៉ោ ដូច
លមថចចលាំ េះវតទដដលបានលឡ
ុ ង
ើ ដរចេះទង
ាំ លនាេះ? រួចនាស ាំ ស
ិ សឱ្យចូលលៅដលស
់ ណ្
ាំ ួ រគនលេះឹ “លតអ
ើ ងគធាតុរាវ
អវខ
ី លេះដដលអាចលរបសើ រោបស
់ ោែតដរចេះោនរបសទ ិ នភាព?”
- លលក
ើ ទក
ឹ ចត
ិ ថសស
ិ សឱ្យគត
ិ អព
ាំ អ
ី ព
ាំ អ
ី ងគធាតុរាវដដលោនលៅកបងផធ
ុ េះរបសល់ គ ដដលអាចយកមកលរបជា

ភាបក់ រសោែតដរចេះ នង
ិ លលក
ើ ទក
ឹ ចត
ិ ថឱ្យពួកលគបលងេើតសមមតក
ិ មម លោយបលាំ ពញកបងសនល
ុ ក
ឹ កច
ិ ចការ នង
ិ ឱ្យ
រកុមនម
ី ួយៗប ា ញគន
ាំ ត
ិ របសព
់ ួកលគដលរ់ កុមដដទ។
- កបងករណ្
ុ ី សស
ិ សបលញ្ច ញលោបលទ
់ កទ
់ ងលៅនង
ឹ សូលុយសយងដដលោនជាត
ុ អា
ិ សុត
ី អាចសោែតដរចេះ
បានលែ លនាេះរគូរតូវផឋលឱ្
់ កាសឱ្យសស
ិ សលធវលើ តសថលកខណ្ៈអាសុត
ី របសអ
់ ងគធាតុរាវផធេះបាយទង
ាំ លនាេះ
និងថាលតើរតូវលធវើលតសថោ៉ោងដូចលមឋច។ កបងករណ្
ុ ី លនេះ រកោស់តួណ្ឺសុល ឬរកោស់ pH អាចរតូវបាន
លរបើ។ រកោស pH អាចរកទិញបានលៅហាងលក់ថាបលាំ ពទយ វាោនតដមលរបដហល ៣០០០ លរៀល កបងមួ
ុ យ
ដុាំ ដដលោន ៤០ សនលឹក។

27
- រគូរតូវសួរសិសសឱ្យបលងេើសមមតិកមមមថងលទៀត បនាធប់លគលធវើលតសថលកខណ្ៈអាសុីតដនអងគធាតុរាវផធេះបានរួច
លហើយ។
- បនថលលើទឹកចិតថសិសសឱ្យបលងេើតបលង់ពិលសាធន៍ និងឱ្យរកុមនីមួយៗប ា ញគាំនិតរបស់លគដល់រកុមដដទ
លហើយរគូអាចបាំលពញបដនទមគាំនិតខលេះលទៀតដដលសិសសលៅោនខវេះចលនាលេះ។
- ជួយសរមបសរមួលសិសសលធវើពិលសាធន៍ របមូលទិនបន័យ វ ិភាគ និងសនបិោឌនអវីដដលពួកលគរកលឃើញ
តាមរយៈពិលសាធន៍។
- ការពរងីកគាំនិតសិសសអាចរតូវបានលរៀបចាំ លោយឱ្យពួកលគស្សាវរជាវបដនទមចាំល េះសូលុយសយងលផសង

លទៀត លៅផធេះ និងលធវើបទប ា ញលទនផលរបស់លគ លៅលោង
៉ោ លរកាយ។
4. គាំនិតរឹះរក

តាមរយៈការនិោយលរឿងអាំពីការលឡើងដរចេះរបស់វតទលធវ
ុ ើជាលោហៈ រគូអាចទក់ទញចាំណាប់អារមមណ្៍
សិសស និងឱ្យពួកលគចាប់លផឋើមគិតអាំពីវតទដដលោនដរចេះលៅកប
ុ ងជី
ុ វភាពរបស់ពួកលគ។ តាមរយៈលរឿងលផសង
លទៀត ដដលនិោយអាំពីផលវ ិបាកដដលបណា
ថ លមកពីការលឡើងដរចេះ សិសសចាប់លផឋើមគិតថា លតើរតូវលធវើដូច
លមឋច លដើមផីសោែចដរចេះបាន លហើយដដលនឹងនាលាំ ៅដល់សាំណ្ួរគនលឹេះ។ លរកាមការសរមបសរមួលរបស់រគូ
សិសសអាចលោេះស្សាយបញ្ញាបានលោយខលនឯង
ួ លោយអាចកាំណ្ត់បានថាអងគធាតុរាវផធេះបាយណាខលេះដដល
អាចលរបើសរោប់សោែតដរចេះបានលោយោនរបសិទនភាព។ ដូលចបេះ សិសសអាចលឆលើយសាំណ្ួរបញ្ញាបានលោយ
ខលនឯង
ួ លៅចុងបញ្ច ប់ដនលមលរៀន។

28
កិចចដតងការបលរងៀន
១. សរោប់បលរងៀនៈ សិសសថាបក់ទី៨

២. លមលរៀនៈ លាយ និងការដញកលាយ (ការដញកអងគធាតុរាវជាសូលុយសយង


ុ និងជាអងគធាតុរាវសុទន)

៣. រយលពលៈ ៥០នាទី

៤. វតទប
ុ ាំណ្ងៈ សិសសអាច

. ពនយល់បានពីវ ិធីដញកអងគធាតុរាវដដលជាសូលុយសយង
ុ និងអងគធាតុរាវសុទន តាមរយៈការសលងេត
ពិលសាធន៍បានរតឹមរតូវ។
. ោនជាំនាញកបងការដញកអងគ
ុ ធាតុរាវដដលជាសូលុយសយង
ុ និងជាអងគធាតុរាវសុទនលចញពីោបតាមរយៈ
ពិលសាធន៍បានរតឹមរតូវ។
. បលងេើនការចូលចត ិ ាមុខវ ិជាជវ ិទាសាស្រសថ តាមរយៈយុទនសាស្រសថលោេះស្សាយបញ្ញា នង
ិ ថសក ិ ផាភាជប់
ជាមួយនិងជីវភាពរបចាដាំ ែៃ។
៥. សោភរៈ
ដកវថាល សាលបរ ទឹក អាំបិល សេរ លមៅសូោ ខាច់ ទឹកលខមេះ ចន្រ្ េ នហាគស ឬលទៀន។
៦. ដាំលណ្ើរការបលរងៀន នង
ិ លរៀន
សកមមភាពរគូ ខលឹមសារលមលរៀន សកមមភាពសិសស
ជាំហានទី១
រតួតពិនិតយវ ិន័យ រលបៀបរាបរយ លាំនឹងថាបក់ (២នាទី) សិសសអងគយតាមរកុ
ុ ម និងលរតៀម
របស់សិសស ខលនសរោប
ួ ់លមលរៀន។
ជាំហានទ២

. អវលី ៅជាអងគធាតុសុទន? រឭកលមលរៀន
ាំ (៣នាទ)ី . អងគធាតុគជា
ឺ ធាតុដដលផសាំលឡង
ើ ពី
. អវលី ៅជាលាយ? អងគធាតុដតមួយរបលភទ
. សូមរកលាយពីរដដលអបកសាថល់ . លាយគឺជាបនសាំដនអងេធាតុសុទន
កបងជី
ុ វភាពរបចាដាំ ែៃ។ ពីរ ឬលរចើនរបលភទ។
. ឧទហរណ្៍៖ សមល បដងាម នាំ...
ជាំហានទ៣
ី (លមលរៀនែមី) (៤០នាទ)ី
. ប ា ញរោយលាយមួយចាំនួនឱ្យ . សលងេត កត់រតា និងលឆលើយលៅ នឹង
. លសចកថលី ផថើម (១០នាទ)ី
សិសសលឃើញ សាំណ្ួរមួយចាំនួនរបស់រគូ
បទប ា ញដនការោយលាយ
. លធវើជារចឡាំដកវលាយជាមួយនឹង . ចាប់លផថើមគិតរកវ ិធីលដើមផីដញកដកវ
ដកវទឹករួចឱ្យសិសសជួយ រកថាដកវ លាយ និងដកវទឹកសុទន នឹងបលងេើត
ទឹករួចឱ្យសិសសជួយ រកថាដកវណា ជាសាំណ្ួរ។
ជាលាយ ណាជាទឹកសុទន និងលធវើ
ោ៉ោងដូចលមថច?

29
. រគូដណ្នាស
ាំ ស
ិ សោកល់ ្មេះដកវ
លឡើងវ ិញថា A, B, C, D, E លរៀងៗោប
ចូរមយង្
ើ ផែកសគា
ាំ លស
់ ូលុយសយុង្ នង្
ិ អង្គធាតុរាវសុទធ
. លលើទឹកចិតថសិសសឱ្យបាំលពញសមមតិ . ការបលងេើតសមមតិកមម (៥នាទី) ពិភាកាតាមរកុម លដើមផីបលងេើតសមមត
កមមរបស់លគលៅកបងសនល
ុ ឹកកិចចការ។ កមម និងបាំលពញសនលឹកកិចចការ និង
. ឱ្យសិសសប ា ញសមមតិកមមរបស់ ប ា ញគាំនិតរបស់លគលៅរកុមដដទ។
ពួកលគតាមរកុម
. លលើកទឹកចិតថសិសសឱ្យបលងេើតបលង់ ការបលងេើតបលង់ពិលសាធន៍ ពិភាកាតាមរកុម លដើមផីបលងេើតបលង់
ពិលសាធន៍ និងប ា ញគាំនិតរបស់លគ ពិលសាធន៍ និងប ា ញដល់រកុមដ
ដល់រកុមដដទ។ ដទ។
. ពនយល់បដនទម អាំពីដាំលណ្ើរការ
ពិលសាធន៍ បលចចកលទស និងបរមុង
របយត
័ បដលស
់ ស
ិ ស។
. ផថល់សោភរពិលសាធន៍ និងសរមប ការពិលសាធ (១០នាទី) លធវើពិលសាធន៍តាមរកុម និងកត់រតា
សរមួលសកមមភាពសិសស បាំលពញចលនាលេះកបងសនល
ុ ឹកកិចចការ។
. លលើកទឹកចិតថសិសសឱ្យលធវើបទ ការលធវើបទប ា ញ (១០នាទី) លធវើបទប ា ញអាំពីអវីដដលពួកលគ
ប ា ញអាំពីអវីដដលពួកលគទទួលបាន ទទួលបាន និងរកលឃើញពីល លសា
និងរកលឃើញពីពិលសាធន៍ ធន៍ (លសចកថីសនបិោឌន)
. លតើលយើងអាចលរបើវ ិធីអវីលដើមផីដញក ជាំហានទី៤៖ ពរងឹងពុទនិ (៣នាទី) . លយើងអាចដញកសាំោត់លោយលរបើ
សាំោល់សូលុយសយង
ុ និងអងគធាតុ វ ិធីឬាំហួតលោយកលតថ។
រាវសុទន?
. លតើោនភសថតាងអវ
ុ ី បញ្ញជក់ថាអងគ . អងគធាតុរាវសុទនពុាំបនសល់អងគធាតុ
ធាតុរាវមួយជាសូលុយសយង
ុ ឬជា រងលរកាយលពលរ
ឹ ហួ
ាំ តលទ ដដលវាផធយ

អងគធាតុរាវសុទន? ពីសូលុយសយង។

ឱ្យសិសសសិកាបដនទមអាំពីអងគធាតុរាវ ជាំហានទី៥៖ កិចចការផធេះ (២នាទី) កត់រតាកិចចករផធេះ


លផសងលទៀតលៅផធេះថាលតើវាជាលាយ
ឬអងគធាតុសុទន? និងរាយការណ្៍លៅ
លោង
៉ោ សិកាលរកាយ

30
សនលក
ឹ កិចកា
ច រ

សូលុយសយង
ុ និងអងគធាតុរាវសុទន

សលងេតការោយលាយ៖ លតោ
ើ នអវលី កត
ើ លឡង
ើ លបលើ យង
ើ ោយ...?

. ខាច់ និងទឹក៖...........................................................................................................................

. អប
ាំ ល
ិ នង
ិ ទក
ឹ ៖ ........................................................................................................................

. សេរ និងទឹក៖ ............................................................................................................................

. លមៅសូោ នង
ិ ទក
ឹ លខមេះ៖ ............................................................................................................

សាំណ្ួរគនលឹេះ

សមមតិកមម៖ មតើអនកមធវើយង្
៉ា ដូចមមេច មទើបដឹង្ថ្នវាជាសូលុយសយុង្ (លាយ) ឬអង្គធាតុរាវសុទធ)?

លតើគិតថា របសិនលបើវាជាអងគធាតុរាវសុទន វានឹង ................................................................................

លហើយរបសិនលបើវាមិនដមនជាអងគធាតុរាវសុទនលទ វានឹង ....................................................................

បលង្ព
់ ិមោធន៖៍ មតើរតូវមធវើមតសេសមមតិកមមរបស់អនកយង្
៉ា ដូចមមេច? (ចូរលរោងបលង់ពិលសាធន៍របស់អបក)

ចូរពមិ ោធន ៍ នង្


ិ កតរ់ ាលទធផល៖ បលាំ ពញតារាងខាងលរកាម

អង្គធាតុរាវ ការសមង្កត អង្គធាតុសុទធ ឬលាយ? អង្គធាតុរាវមួយណ្?

សនបិោឌន៖ លតើអបករកលឃើញអវីខលេះ?

អងគធាតុរាវដដលជាសូលុយសយងោន............................................លហ
ុ ើយអងគធាតុរាវសុទនោន..................................
ដដលអាចដញកវាបានតាមរយៈ ................................................................................................................................។

31
បលងកា
់ ថ រលខៀន

សូលុយសយង
ុ និងអងគធាតុរាវសុទន

សលងេតការោយលាយ៖ លតោ
ើ នអវលី កត
ើ លឡង
ើ លបលើ យង
ើ ោយ...?

. ខាច់ និងទឹក៖...........ខាច់មិនរោយកបងទ
ុ ឹកលទ លរកាយលពលកូរ លយើងលឃើញខាច់លៅដដល.....

. អប
ាំ ល
ិ នង
ិ ទក
ឹ ៖ .........អប
ាំ ល
ិ រោយកបងទ
ុ ក
ឹ បលងេើតជាសូលុយសយងថា
ុ ល ........................................

. សេរ និងទឹក៖ ..............សេររោយកបងទ


ុ ឹក បលងេើតជាសូលុយសយងថា
ុ ល ............................................

. លមៅសូោ នង
ិ ទក
ឹ លខមេះ៖ ......ោនពពុេះលកត
ើ ោន លមៅសូោរោយបលងេើតបានជាសូលុសយងថា
ុ ល

សាំណ្ួរគនលឹេះ ចូរលយើងដញកសោគល់សូលុយសយង
ុ និងអងគធាតុសុទន!

សមមតិកមម៖ មតើអនកមធវើយង្
៉ា ដូចមមេច មទើបដឹង្ថ្នវាជាសូលុយសយុង្ (លាយ) ឬអង្គធាតុរាវសុទធ)?

លតើគិតថា របសិនលបើវាជាអងគធាតុរាវសុទន វានឹង ......ោមនសល់អវីទង


ាំ អស់លរកាយលពលដុតរហួ
ាំ តទឹកអស់......

លហើយរបសិនលបើវាមិនដមនជាអងគធាតុរាវសុទនលទ វានឹង ....លៅសល់រកាមរងលរកាយលពលដុ


ឹ តរហួ
ាំ តទឹកអស់......

បលង្ព
់ ិមោធន៖៍ មតើរតូវមធវើមតសេសមមតិកមមរបស់អនកយង្
៉ា ដូចមមេច? (ចូរលរោងបលង់ពិលសាធន៍របស់អបក)

. ោកអ
់ ងគធាតុរាវបនថច
ិ ចូលកបងឧបករណ្
ុ ៍ រហួ
ាំ ត ដូចជាចានរហួ
ាំ ត ឬសាលបរ លោហៈ
. ដុតរហួ
ាំ ត នង
ិ សលងេតលមល
ើ ោនអវលី កត
ើ លឡង
ើ បនាធបព
់ រី ហួ
ាំ ត
. លយង
ើ អាចលរបឧ
ើ បករណ្៍ដុតកលតថលផសងៗ ដូចជាចន្រ្ េ នហាគស ចលងេៀងអាលក
់ ុល ឬលទៀន។

ចូរពមិ ោធន ៍ នង្


ិ កតរ់ ាលទធផល៖ បលាំ ពញតារាងខាងលរកាម

អង្គធាតុរាវ ការសមង្កត អង្គធាតុសុទធ ឬលាយ? អង្គធាតុរាវមួយណ្?


A ោនសល់អងគធាតុរងពណ្
ឹ ៌ស សូលុយសយង
ុ អាំបិល (រគូរបាប់)
B ោនសល់អងគធាតុរងពណ្
ឹ ៌ លតាបត សូលុយសយង
ុ សេរ (សិសសរបាប់)
C ោនសល់អងគធាតុរងពណ្
ឹ ៌ស សូលុយសយង
ុ ទឹកលខមេះនិងលមៅសូោ(រគូរបាប់)
D ោមនសលអ
់ លវី ទ អងគធាតុរងសុ
ឹ ទន ទក
ឹ (សស
ិ សរបាប់)
E ោនសល់អងគធាតុរងពណ្
ឹ ៌ស សូលុយសយង
ុ ទឹកសូោ (រគូរបាប់)

សនបិោឌន៖ លតើអបករកលឃើញអវីខលេះ?

អងគធាតុរាវដដលជាសូលុយសយងោន...អងគ
ុ ធាតុរាវ A, B, C និង E...លហើយអងគធាតុរាវសុទនោន....អងគធាតុរាវ D....
ដដលអាចដញកវាបានតាមរយៈ ........វ ិធីរហួ
ាំ តលោយដុតកលតថ................................................................។

32
កាំណ្ត់សាំោល់លមលរៀន (សរោប់រគូ )

កាំណ្ត់សាំោល់លមលរៀន (សរោប់រគូ )

១.មូលោឌនរគឹេះ

របធានបទសឋីអាំពី «សូលុយសយង
ុ និងអងគធាតុរាវសុទន» រតូវបានអភិវឌណ និងលរៀបចាំលឡើងលោយដផែកតាមលោល
ការណ្៍បលរងៀន-លរៀនតាមដបបរេះរក
ិ និងោនលោលលៅសរោប់សិសសថាបក់ទី៨ ដនមធយមសិកាបឋមភូមិ។ លទេះជា
ោង
៉ោ ណាកល៍ ោយ លដើមផីដាំលណ្ើរការលមលរៀនលនេះបាន សស
ិ សរតូវោនចលាំ ណ្េះដឹងមូលោឌនរគេះឹ ចាបា
ាំ ច់ ទង
ាំ ចលាំ ណ្េះ
នង
ិ ជាំនាញមួយចន
ាំ ួន។ ដូលចបេះ មុនលពលអនុវតថលមលរៀនលនេះ សស
ិ សរតូវលរៀនចបន
់ ូវខលម
ឹ សារលមលរៀនជាមូលោឌនដូចជា
ិ វ ិធដី ញកលាយរួចលហយ
អងគធាតុសុទន លាយ នង ើ ។

២. ការដាំលឡើងសោភរ

លមលរៀនលនេះរតូវការសោភរសមញ្ដបផ
ាំ ុត លដើមផីដាំលណ្ើរការសកមមភាពកបងថា
ុ ប ក។
់ សោភរទង
ាំ លនាេះោនដូចជា អប
ាំ ល
ិ សេរ
ដីខាច់ លមៅសូោ (NaHCO3) ទក
ឹ ទក
ឹ លខមេះ ទក
ឹ សូោ ដកវថាល កូនសាលបរ នង
ិ សាលបរ ធ។
ាំ ចន្រ្ េ នហាគសអាចរតូវ
បានលរបោ
ើ ង ៉ោ ោនរបសទ
ិ នភាពសរោបរ់ ហួាំ តលោយដុតកលតថ ដតលទៀនកអា
៍ ចលរបជា
ើ របភពកលតឋបានដដរ កបងករណ្
ុ ី
ោមនលយង ើ ោមនចរ នហាគស។ មាង ៉ោ វ ិញលទៀត រគូអាចលរជើសលរសសោ
ើ ភ រលផសងៗលទៀត លដើមផីប ា ញសស
ិ សពកា
ី រ
ោយលាយលៅតាមលទនភាពដដលអាចរកបាន។

៣. ដាំណ្ឹកនាលាំ មលរៀន

រគូរតូវដតោនលកខណ្ៈសាវហាប់កងការប
បុ ា ញ និងដាំណ្ឹកនាលាំ មលរៀន ដដលោត់អាចលរបើរបាស់វ ិធីសាស្រសថជាលរចើន
តាមរយៈការប ា ញសោភរជាក់ដសឋង លដើមផីទក់ទញចាំណាប់អារមមណ្៍សិសស។ ខាងលរកាម គឺជាវ ិធីសាស្រសថមួយ
សរោប់លមលរៀនលនេះ ៖

. លរៀបចាំោក់ទឹកកបងដកវថា
ុ ល នីមួយៗ នូវោឌលសមើោប និងោក់ពីមុខសិសសឱ្យលហើយ មុនលពលដាំលណ្ើរការលមលរៀន

. រគូរតូវលធវើបទប ា ញោ៉ោងទក់ទញបាំផុតដដលអាចលធវើបាន អាំពីការោយ បលងេើតជាលាយលៅពីមុខ


សិសស រួមជាមួយសាំណ្ួរបាំផុស និងទញអារមមសិសសឱ្យសលងេតោ៉ោងរបុងរបយ័តប និងកត់រតាចូលកបងសនល
ុ ឹក
កិចចការដដលបានដចកជូន។

. លោយសារោនលាយលរចើនរតូវបានោយ លហើយលាយទង
ាំ លនាេះភាគលរចើនោនលកខណ្ៈថាលដូចជាទឹក
ដដរ រគូរតូវលធវើជារចឡាំដកវ លោយមិនដឹងថាដកវណាជាដកវលាយណាជាទឹកលទ។ ដូលចបេះ រគូអាចសូមឱ្យ
សិសសជួយ ដញកបញ្ញជក់ថា ដកវណាជាដកវលាយណាជាទឹកសុទន។

. ភាជប់បញ្ញាលនេះលៅនឹងសាំណ្ួរគនលឹេះ « ចូរលយើងដញកសាំោល់សូលុយសយង
ុ និងអងគធាតុរាវសុទន! »។ រគូ
អាចលលើកទឹកចិតថសិសសឱ្យបលងេើតសាំណ្ួរមួយសរោប់បញ្ញាមួយលនេះលោយខលនឯង
ួ មុននឹងដាំលណ្ើរការលោេះ
ស្សាយវា។

. លោយលពលលនេះលយើងមិនដឹងថាដកវណាជា ជាដកវអវី រគូរតូវនាស


ាំ ិសសោក់ល្មេះែមីលៅលលើដកវទង
ាំ អស់
លនាេះ លដើមផី យសាំោល់លពលដាំលណ្ើរការពិលសាធន៍។ ល្មេះែមីអាចជា A, B, C, D, E...តាមលាំោប់ឮលាំលោ
យ។ រតូវចាថា
ាំ ល្មេះអងគធាតុរាវមិនសាំខាន់លទលៅកបងដាំ
ុ លណ្ើរការស្សាវរជាវលនេះ។ លោលបាំណ្ងគឺរោន់ដត

33
ចង់រកឱ្យលឃើញថា ដកវណាជាសូលុយសយង(លាយ)
ុ លហើយដកវណាជាអងគធាតុរាវសុទន(ទឹក) ដតប៉ោុលណាតេះ។
លទេះជាោ៉ោងណាក៍លោយ រគូអាចសាកលផងសួរសិសសឱ្យបញ្ញជក់ ឬរគូរបាប់លៅចុងលមលរៀន។

. លលើកទឹកចិតថសិសសឱ្យបលងេើតសមមតិកមម និងបលង់ពិលសាធន៍ លោយពួកលគផាធល់ លោយសួរសាំណ្ួរថា «លតើ


លយើងរតូវលធវើោ៉ោងដូចលមឋច?»។ រតូវលលើកទឹកចិតថចាំល េះបលង់ពិលសាធន៍ដដលសិសសរកបានលលើកលឡើង លោយ
ផឋល់ឱ្កាសឱ្យោត់បានសាកលផងនូវគាំនិតរបស់ោត់ផាធល់ តាមរយៈការផឋល់សោភរបដនទមតាមដដលអាចលធវើ
បាន។

. សរមបសរមួលសិសសកបងសកមម
ុ ភាពពិលសាធ ស្សង់ទិនបន័យ និងបាំលពញកបងសនល
ុ ឹកកិចចការ រពមទង
ាំ ឱ្យពួក
លគលធវើបទប ា ញអាំពីអវីដដលពួកលគរកលឃើញ។

. ឱ្យសិសសពរងីកការយល់ដឹងរបស់ពួកលគ លោយឱ្យលគសិកាបដនទមលទៀតលៅលលើអងគធាតុរាវ ដដលលគោន


លៅផធេះ ថាលតើណាខលេះជាលាយ និងណាខលេះជាអងគធាតុសុទន? និងលលើទឹកចិតថឱ្យពួកលគលធវើបទប ា ញពីលទន
ផលលនាេះលៅលោង
៉ោ លរកាយ។

៤.ទសសនៈរេះរក

តាមរយៈបទប ា ញរបសរ់ គូ អព
ាំ កា
ី រោយលាយ នង
ិ លធវជា
ើ រចឡដាំ កវ សស
ិ សចាសជា
់ ចាបល់ ផឋើមគត
ិ ថាលតើ
រតូវលធវកា
ើ រដញកសោ
ាំ លអ
់ ងគធាតុរាវណាជាលាយ នង
ិ ណាជាអងគធាតុរាវសុទនោង
៉ោ ដូចលមឋច។ លនេះោននយ

ថារគូទទួលបានលជាគជ័យកបងការទក
ុ ទ
់ ញអារមមណ្៍សស
ិ ស ឱ្យគត
ិ ពអ
ី ដវី ដលពួកលគរតូវលធវលើ ៅកបងលមលរៀន

លនេះលហយ
ើ ។ តាមរយៈសណ្
ាំ ួ របផ
ាំ ុសបដនទមរបសរ់ គូ សស
ិ សអាចលរៀបចប
ាំ លង់ពលិ សាធន៍ លធវព
ើ លិ សាធន៍ របមូល
ទន
ិ បនយ
័ បកស្សាយទន
ិ បនយ
័ ពភា
ិ ការួមរកុម លធវលើ សចកឋស
ី នបោ
ិ ឌ នបានលោយខលនឯង។
ួ លៅលពលលនាេះ បញ្ញា
នង
ឹ រតូវលោេះស្សាយបានលោយសស
ិ ស ដដលជាលោលលៅចមផងរបសកា
់ រលរៀនតាមដបបរេះរក។

34
កច
ិ ចដតងការបលរងៀន
១. សរោប់បលរងៀនៈ សិសសថាបក់ទី៧

២. លមលរៀនៈ ការកាំណ្ត់ដង់សុីលតអងគធាតុរាវ

៣. រយលពលៈ ៦០នាទី

៤. វតទប
ុ ាំណ្ងៈ សិសសអាច

. ពណ្៌នាពីលកខណ្ខុសោប ដនអងគធាតុរាវចាំនួនរបាតា
ាំ មរយៈការសលងេតពិលសាធប ា ញរបស់រគូបាន

រតឹមរតូវ។

. រលបៀបកាំណ្ត់រកដង់សុីលតអងគធាតុរាវ តាមរយៈការអនុវតថពិលសាធរកោស
៉ោ និងោឌអងគធាតុរាវទង
ាំ របាាំ
លោយសស
ិ សបានរតម
ឹ រតូវ។
𝑚
. បលងេើនបាំណ្ិនគណ្នាដង់សុីលតតាមរូបមនថ 𝑑= និងោនបាំរុងរបយ័តបកងលពលព
បុ ិលសាធន៍។
𝑉

៥. សោភរៈ

. ដកវលបដស៥
៉ោ សុីឡាាំងរកិត៥ ពីដប៉ោត ជញ្ជ ីងលអឡិតរតូនិច បាំពង់សាក កូនកដន្រ្នថ កាល់សយល
ូ (Casio) ទឹក
ឃមាំុ ទឹកសុឺរូ ៉ោ សាបូោងចាន (LIX) អាល់កុល ល័កខអាហារពណ្៌លខៀវ ដបតង រកហម លលឿង ដុាំឥដឌ ដុាំលឈើ ...
និងទឹក។

៦. ដាំណ្ឹកនាលាំ មលរៀន

សកមមភាពរគូ ខលឹមសារលមលរៀន សកមមភាពសិសស


ជាំហានទី១ (២នាទី)
រគប់រគងថាបក់ លរៀបចាំថាបក់ និងអវតថោនសិសស រកាការសៃប់សាៃត់
ជាំហានទ២
ី (៣នាទី)
. ដូចលមថចដដលលៅថារូបធាតុ? . វតទទ
ុ ង
ាំ ឡាយដដលោនោស
៉ោ និង
ោឌតាង
ាំ កបងល
ុ ាំហ។
. លតើភាពរូបដនរូបធាតុោនអវីខលេះ? . រូបធាតុ និងភាពរូប . ោនបគឺរង
ឹ រាវ និងឧសម័ន។
. លដើមផីលធវើចាំដណ្កថាបក់តាមជាក់ . ចាំដណ្កថាបក់រូបធាតុតាមភាពជាក់ . ភាពយឺត របឹង ភាពហូតជាល
ាំ ួស
ោក់ដនរូបធាតុោនអវីខលេះ? ោក់ ភាពដផន ភាពចមលងអគគិសនី កលតថ
ចាំនុចរោយ ដង់សុីលត
. ចូរឱ្យរូបមនថសរោប់គណ្នាដង់សុី . រូបមនថគណ្នាដង់សុីលត . រូបមនថសាំរាប់គណ្នាដង់សុីលតគឺ
m
លតអងគធាតុ៕ 𝑑= V
ជាំហានទី៣ (៤០នាទី)
. ដចកសនលឹកកិចចការឱ្យសិសស និង របធានបទ លមើលសនលឹកកិចចការ គិត និងសរលសរ
ដណ្នាាំ ការកាំណតដ
់ ង្់សុីមតអង្គធាតុរាវ ចលមលើយចូល។

35
រគូលធវព
ើ លិ សាធប ា ញទី១ លដើមផី ១. សលងគត សលងេត គត
ិ រួចលឆលយ

លរបៀបលធៀបដង់សុីលតដនអងគធាតុរង
ឹ ដុាំលឈើអដណ្ថត ឯដុាំឥដឌធៃន់ជាងទឹក
(រូបទី១ និង២) នឹងរបាប់សិសសឱ្យ លគថាឥដឌោនដង់សុីលតធាំជាងលឈើ។
លធវើការសលងេត និងចាលាំ ឆលើយសាំណ្ួរ ដុាំលឈើ
រូបទី១ .សលងេតលមើលលោយរបុងរបយ័តប
៣៤
លោយយកដកវមួយោក់ទឹក ដុាំ ឆៃល់គិតលឆលើយ...
លឈើ១ និងដុាំឥដឌមួយដដលោនទាំហាំ
ដុាំឥដឌ
ប៉ោុនោប រាងដូចោប រួចផថល់សាំណ្ួរលៅ . លធវើពិលសាធន៍ លោយរបុងរបយ័តប
សិសស លតើលពលដដលអបករគូទាំោក់ដុាំ នឹងយកចិតថទុកោក់។
ឥដឌលនេះចូលលៅកបងដកវទ
ុ ឹក លតើោន ដុាំលឈើ
អវីលកើតលឡើង? ចូលបកស្សាយ។
សាំណ្ួរភាជប់លយើងអាចលរបៀបលធៀប រូបទី២

ដង់សុីលតអងគធាតុរងទ
ឹ ងាំ ពីរដដល ដុាំឥដឌ
ខុសោបបានលោយលរបទ
ើ ក
ឹ ចុេះលបើ
លយង
ើ ចង់លរបៀបលធៀបអងគធាត
ាំ ុរាវ
ដដលជាសារធាតុខុសោបវ ិញ លតើ
លយង
ើ រតូវលធវដ
ើ ូចលមថច?
. ប ា ញបព
ាំ ង់សាកដដលោនោក់ . របដហលមកពីវា ជាសារធាតុទង
ាំ
E
អងគធាតុរាវទង
ាំ ៥ រូបទ៣
ី របាប់ របាលាំ នេះោនដង់សុីលតខុសោប។
សស
ិ សឱ្យលធវកា
ើ រសលងេតនង
ិ លលក
ើ ជា D
រូបទី៣
សណ្
ាំ ួ រ លតលើ ហតុអបា
វី នជាលាយដន
សូលុយសយងរបា ាំ លនេះអាចសទិតលៅ C

ជាស្សទប់ៗដូចលនេះ? . សរលសរសាំណ្ួរគនលឹេះចូលកបងរក

B
. សរលសរសាំណ្ួរគនលឹេះលលើកាថរលខៀន។ ោស់កិចចការ រាងៗខលន។

មតើដង្់សមុី តអង្គធាតុទង្
ាំ របា
Aខ្
ាំ ុសគានដូចមមេចមទើបវាអាចបមង្កើតពណ៌បានដូចកនុង្រូប?
មតើរតូវមធវើដូចមមេចមដើមបីកណ
ាំ តដ
់ ង្់សមុី តរបសវា
់ ?
២. សាំណ្ួរគនលឹេះ (២នាទី)
ឱ្យសស
ិ សលធវកា
ើ រគត
ិ នង
ិ ទសសនទ
៍ យ ដង់សលុី តវាខុសោប . លធវកា
ើ រទសសនទ
៍ យសាមនតាមគន
ាំ ត

លទនផល ជារកុម (៣នាទ)ី ដូចតលៅ
. កតរ់ តាចលមលយ
ើ រ ងទុករបស់ ៣. សមមតក
ិ មម (៥នាទី)
សស
ិ សលលកា
ើ ថ រលខៀន . គថា
ឺ DA > DB > DC > DD > DE . ដង់សលុី តវាតូច ធជា
ាំ ងោប
. កណ្
ាំ តោ
់ ស
៉ោ អងគធាតុរាវនម
ី ួយៗ . កណ្
ាំ ត់ ោស
៉ោ អ
់ ងគធាតុរាវ កណ្
ាំ ត់
គត
ិ ជារកាម ោឌ....
. កណ្
ាំ តោ
់ ឌអងគធាតុរាវនម
ី ួយៗគត

ជាមល
ី ល
ី រី ត
m
. គណ្នាដង់សុីលតវាតាមរូបមនថ . គណ្នាតាមរូបមនថ 𝑑 = V
m
𝑑= V

36
៤. បលង់ពលិ សាធន៍ ពលិ សាធន៍ នង

. ឱ្យសិសសគិតបលង់ពិលសាធន៍ ដូច លទនផល (២៥នាទី) . គិតពិភាកា តាមរកុមរួចគូសបលង់
លមថចលដើមផីបកស្សាយបានពីសមមតិ លៅតាមគាំនិតលរៀងៗខលន
ួ និងរកុម
កមម ដដលលគបានទយទុក។ លបើ . សាថប់ លមើល សលងេត ការដននាាំ
សិសសលធវើមិនលកើតរគូពនយល់ប ា ញ របស់រគូ និងទទួលសាំភារ
និងដណ្នា។
ាំ . អនុវតថពិលសាធ ជារកុម តាមការ
. ប ា ញសិសសពីរលបៀបលរបើរបាស់ កាំណ្ត់របស់រគូ នឹងតាមលាំោប់លាំ
សោភរៈពិលសាធ បរមុងរបយ័តប និង លោយ។
ឱ្យសិសសលធវើការសលងេត ឱ្យតាំណាង
រកុមលឡើងមកយកសោភរ
. ផថល់ឱ្កាសឱ្យសិសសលធវើពិលសាធន៍ . ស្សង់ទន
ិ បនយ
័ ោកក
់ ងតារាង
បុ
និងស្សង់លទនផលោក់កងតារាង
បុ
. របាប់សិសសពីរលបៀបគណ្នារក ោ៉ោស(g)
សុី
. រកោស
៉ោ ោឌ និងគណ្នាតាមរូប
សុី ដង់សុី ល្មេះ
ោស
៉ោ ោឌ តាមពិលសាធន៍ និងដង់ អ
.ធ
ឡាាំង
ឡាាំង
រកិត

.ធ
ោឌ
(ml)
លត អ.ធ មនថនូវលទនផលពិលសាធន៍បាន រួច
រកិត (g/ml) រាវពីរ
សុីលតតាមរូបមនថ សៃត

ោន
អ.ធ
រាវ
បកស្សាយ និងពនយល់បាតុភូត ដន
លតើលរកាយលពលពិលសាធបែនសលងេ
ូ ត A
ពិលសាធន៍។
B

លឃើញោនអវីលកើតលឡើង? ចូរពនយល់ C
D

ឱ្យសិសសគណ្នាោស
៉ោ រួចអនុវតថ E

តាមរូបមនថ គណ្នាដង់សុីលតសារ . ការគណ្នា (Calculation) . ស្សង់ទិនបន័យដង់សុីលតតាមរកុម


ធាតុនីមួយៗតាមរកុមរួចឱ្យពួកលគ m(syrup)=162.3-130.2 = 32.1g ពភា
ិ កាគិតគណ្នា
32.1
ស្សង់លទនផលរួមោក់តារាង និង D(syrup)= 27 =1.188g/mL m(syrup)=162.3-130.2 = 32.1g
ចាំដណ្កអងគធាតុរាវទង
ាំ របាតា
ាំ មភាព 32.1
D(syrup)= 27 =1.188g/mL
លកើនលឡើងដនដង់សុីលតរបស់សារធាតុ . ចាំណាត់ថាបក់តាមលាំោប់លកើនលឡើង លធវច
ើ ណា
ាំ តថា
់ ប កតា
់ មលោ
ាំ បល់ កន

នីមួយៗ។ ដនតដមលដង់សុីលត លឡង
ើ ដនតដមលដង់សលុី តអងគធាតុរាវ
DA > DB > DC > DD > DE Acoholl <cooking oil < water
< dishwashing Liquid < syrup
៥. សនបិោឌន (៥នាទី)
. ឱ្យសិសសលធវើការសនបិោឌនតាមរកុម អងគធាតុរាងទង
ាំ របាោ
ាំ នដង់សុីលត . លធវើសនបិោឌនលរៀងខលន
ួ និងពិភាកា
និងលឡើងសរលសរលទនផលលលើកាថរ តូចធាំជាងោប លទើបវាបលងេើតជាលាំោប់ ជារកុមរួចលឆលើយ។
លខៀន។ ពណ្៌នាបាន និងលដើមផីកាំណ្ត់ដង់សុី
. លតើអងគធាតុរាវទង
ាំ ៥ោនដង់សុីលត លតវាលគរតូវលធវើពិលសាធន៍ ែលឹងោស
៉ោ
ខុសោបដូចលមថច? លតើរតូវលធវើដូចលមថច និងវាស់ោឌវាមថងមួយៗ រួចយក
លដើមផីកាំណ្ត់វា? តដមលវាលធវើផលលធៀបោប។
ជាំហានទី៤ (៤នាទី)
. សិសសសាកលផងចាក់អងគធាតុលៅ ពរងឹងពុទនិ លរបៀបលធៀបលៅនឹងប ា ញទី២
កបងប
ុ ាំពង់សាក លមមបនថោបតាម (រូបទី៣) ខាងលលើ។

37
លោ
ាំ បជា
់ កដ់ សថងដូច(រូបទ៣
ី ) តាម អនុវតថពលិ សាធនជា
៍ កដ់ សថងពកា
ី រ សាថបគ
់ ត
ិ នង
ិ សាកលផងចាកអ
់ ងគ
បាំរាប់តាមរកុមលោយប ា ញដប ោយពណ្៌អងគធាតុទង
ាំ ៥ ធាតុរាវទង
ាំ របាម
ាំ ថងមួយៗ តាមលាំ
ដដលោនោក់សូលុយសយងអងគ
ុ ធាតុ ោក់ និងស្សបតាមតដមលដង់សុីលតវា
ទង
ាំ របារាំ ួចរបាប់ពួកលគឱ្យវាល់ោឌ ជាបនថបនាធប់ និងែបមៗ
លសមើៗោប ដនអងគធាតុនីមួយៗ លោយ
លរបើពីដប៉ោតោក់ែបមៗ លៅកបងប
ុ ាំពង់
សាកតាមលាំោប់លាំលោយទឹកសុីរូ ៉ោ
រកហម ទឹកសាបូោងចាន ទឹក
ោយល័កខដបតង លរបង្និងចុង
លរកាយគឺអាល់កុលោយល័កខ្ម
រជូក(លបើោនលពល)។
ជាំហានទី៥ (២នាទី)
១. ចូរគណ្នាដង់សុីលត អងគធាតុ កិចចការផធេះ . សិសសកត់រតា ។
មួយដដលោនោស
៉ោ ១.៦រកាម
និង ោឌ ៣០ មីលីលីរត
២. ចូរគណ្នា ោនោស
៉ោ អងគធាតុ
ដដល ១.៦ រកាម ដង់សុីលត និងោឌ
០.៩៨ រកាម/មីលីលីរត

38
សនលឹកកិចចការសិសស លទនផល (ចូរស្សង់តដមលសារធាតុនីមួយៗោក់កងតារាងខាងលរកាម)
បុ
របធានបទ៖ ការកាំណតដ
់ ង្់សុីមតអង្គធាតុរាវ ោស
៉ោ (g)
១. សលងេត (៥នាទី) ល្មេះអងគ សុីឡាាំង ដង់សុីលត
សុឡា
ី ងាំ ោឌ (mL)
ក. លរកាយលពលដដលោក់ដុាំឥដឌ និងដុាំលឈើចូរលៅកបងដកវទ
ុ ឹក លតើោនអវីលកើត ធាតុរាវ រកិតោន អងគធាតុរាវ (g/mL)
រកត
ិ សៃត

លឡើង? ............................................................................... អងគធាតុ

ខ. លហតុអវីបានជាអងគធាតុរាវទង
ាំ ៥ ដនលាយខាងលលើលនេះសទិតលៅជាស្សទប់
ដូចលនេះ?............................................................................

២. សាំណ្ួរគនលឹេះ (៣នាទី)

 ការគណ្នា
៣. សមមតិកមម ចូរបែនលធវ
ូ ើការទសសន៍ទយ នូវសាំណ្ួរខាងលលើ (៥នាទី) ................................................................................................................
................................................................................................................
. ......................................................................................... ................................................................................................................
. .........................................................................................
. ........................................................................................  ចូលលធវច
ើ ណា
ាំ តថា
់ ប កតា
់ មលោ
ាំ បល
់ លាំ ោយលកន
ើ លឡង
ើ ដនតដមលដង់សលុី តអងគធាតុ
រាវទង
ាំ ៥ ខាងលលើ
៤. បលង់ពលិ សាធន៍ នង
ិ ដាំលណ្ើរការពលិ សាធន៍ (២០នាទ)ី ........................<.......................<.......................<.......................<.................

៥. សនបោ
ិ ឌ ន (៥នាទ)ី
................................................................................................................
................................................................................................................

39
៣.១.ជីវវិទា

៣.១.១ ការពិនិរយទរមង់រ ូបផគំរុ គាប់តភាកថនិកសរវ

(រជូក)

៣.១.២ ការរ ំភ្នយចំហាយទឹករបស់រ ុកខជារិ

៣.១.៣ ចលនាតាមឆនេែះ និងររផលិច

៣.១.៤ ការសរងេរទរមង់របដាប់បនតពូជរបស់រ ុកខជារិ

ម្ភនផ្កេ
កិចចដតងការបលរងៀន

របធានបទៈ ការពន
ិ ត
ិ យទរមង្់រូបផគុរាំ គាបផ់ ភនកថនក
ិ សតវ (រជូក)

មុខវ ិជាជៈ ជីវវ ិទា

់ ៈី ៩ (កមមវ ិធស
ថាបកទ ី ក
ិ ាចាស់)

ជាំពូកទី៥ តរមូវសារ ងគកាយ

ី វ ិញ្ញដណ្
លមលរៀនទី១២ សររាងគ

I. ដភបក

១. ទរមង់ដភបក

២. គាំលហើញ

រយៈលពលៈ ៥០នាទី

វតទប
ុ ាំណ្ង៖

 ចលាំ ណ្េះដឹង
- កាំណ្ត់ពីដផបកលផសងៗរបស់រោប់ដភបកបានរតឹមរតូវតាមរយៈការសលងេតរោប់ដភបកលោយផាធល់។
- កណ្
ាំ តព
់ ម
ី ុខ ររបសដ់ ផបកនម
ី ួយៗរបសរ់ ោបបា
់ នរតម
ឹ រតូវតាមរយៈការសលងេត នង
ិ សណ្
ាំ ួ របផ
ាំ ុស។
 បណ្
ាំ ិ ន
- វេះកាត់ និងសលងេតរោប់ដភបកបានរតឹមរតូវតាមរយៈការអនុវតថផាធល់ និងការដណ្នារាំ បស់រគូ។
 ឥរោបែ

- ស្សោញ់ និងដែរការោប់ដភបកបានរតឹមរតូវ។

សោភរៈ

រោប់ដភបករជូក (រគូបកសាច់ដុាំលចញជាលស្សច) រជុញ កាប


ាំ ិត (ឬឡាម) កដន្រ្នថ ដលងេៀប ដកវពរងីក...។

ដាំណ្ឹកនាលាំ មលរៀន៖

សកមមភាពរគូ ខ្លឹមោរមមមរៀន សកមមភាពសិសស


ជាំហានទី១ លាំនឹងថាបក់ (១២នាទី)
-រតួតពិនិតយថាបក់ និងមតិសាវគម រដឌបាលថាបក់ -សាថប់ និងសាវគម
-រតួតពិនិតយអវតថោន - របធានថាបក់រាយការណ្៍
ជាំហានទី២ រល
ាំ ក
ឹ លមលរៀនចាស់ (៤៦នាទី)
-លយង
ើ ធាលបបា
់ នលរៀនរួចមក ី វ ិញ្ញដណ្ោន៥៖ ដភបក
-សររាងគ រចមុេះ រតលចៀក អណា
ថ ត ដសផក។

43
លហយ
ើ អព
ាំ ស ី វ ិញ្ញដណ្ លតស
ី ររាងគ ើ ររាងគ
ី (ចកខវុ ិញ្ញដណ្) រចមុេះ (ឃាន
វ ិញ្ញដណ្របស់មនុសសោនអវីខលេះ ? វ ិញ្ញដណ្) រតលចៀក (លសាតថវ ិញ្ញដណ្)
-សូមពនិតយលមើលមុខមិតថភ័កថិរបស់ ថ ត (ជីវាាវ ិញ្ញដណ្) និងដសផក
អណា -ពិនិតយលមើលមុខោប លហើយលឆលើយនឹង
លតើោនចាំណ្ុចអវីគួរកត់សាំោល់លៅ (កាយវ ិញ្ញដណ្)។ សាំណ្ួរតាមការលមើលលឃើញរបស់ពួក
លលើមុខរបស់ោត់? លគ។
-លតើអបកដឹងលោយសារអវី? -ដភបកសរោប់លមើល។ -ដឹងលោយសារដភបក។
ជាំហានទ៣
ី លមលរៀនរបចាដាំ ែៃ (៣០៣៥នាទី)
ី វ ិញ្ញដណ្
លមលរៀនៈ សររាងគ
របធានបទៈ ពន
ិ ត
ិ យទរមង់ រោបដ់ ភបក
-សូមពន
ិ ត
ិ យលមល
ើ រោបដ់ ភបក របស់ របសរ់ ជូក -សស
ិ សគូររូបលៅលលរើ កោស លរៀង
មត
ិ ថអបក លហយ
ើ គូររូបរោបដ់ ភបកលោយ លាំនាប
ាំ ញ្ញា សាំណួរគនលឹះឹ សមមតក
ិ មម ខលន
ួ លហយ
ើ សស
ិ សពរី នាក់ លឡង
ើ គូររូប
ោកល់ ្មេះ ដផបកនម
ី ួយៗ។ រោបដ់ ភបកលៅលលកា
ើ ថ រលខៀន។
-បត
ិ រូបទរមង់រោបដ់ ភបកលលកា
ើ ថ -ពន
ិ ត
ិ យលមល
ើ រូបរគូបត
ិ លលកា
ើ ថ រលខៀន
រលខៀន លហយ
ើ ពភា
ិ កាជាមួយសស
ិ ស លហយ
ើ លផធៀងផាធតរ់ ូបភាពដដលលគគូរ
ទង
ាំ អស់។ ជាមួយ នឹងរូបដដលរគូបិត។

សូមបញ្ញាក់ពទ
ី រមង្់រូបផគុផាំ ភនករជូកទង្
ាំ អសគា
់ ន !!!

-ឱ្យសិសសអានសនលឹកកិចចការ រួច ដាំមណើរការសមង្កត -អានសនលឹកកិចចការលហើយ សាថប់រគូ


ពនយល់ពីដាំលណ្ើរការវេះកាត់និងពិនិ ដូចនៅក្នុងសន្លឹក្ក្ិចចការ ពនយល់។
តយ។
លទធផល
-ដចកសោភរឱ្យសិសស (រោប់ដភបកពីរ -លឡើងយកសោភរតាមរកុម
កបង១រកុ
ុ ម) សរដសរបសាទអុបទិក
ករលន
-ឱ្យសិសសគូររូបទរមង់ខាងលរត និង -សិសសគូររូបកបងសនល
ុ ឹកកិចចការលរៀង
ទរមង់ខាងកបងរោប
ុ ់ដភបក លហើយោក់ ខលន
ួ លហើយសិសសពីរនាក់លឡើងគូររូប
ល្មេះដផបកលផសងរបស់វា។ ទង
ាំ ពីរលៅលលើកាថរលខៀន។
-ឱ្យសិសសលរបៀបលធៀបលទនផលលៅ ដកវដភបក របស្សី -សិសសលរបៀបលធៀបលទនផល ដដល
នឹងសមមតិកមមរបស់ពួកលគ។ បានពិនិតយលៅនឹងសមមតិកមមរបស់
ពួកលគ។

លរទីន រននរបស្សី

44
សននដ្ឋ
ិ ឋន
-ឱ្យសិសសលធវើការសនិបោឌនលោយ ទរមង់រូបផគាំរុ ោប់ដភបកោន៖ -សិសសពិភាកាតាមរកុម លហើយសរ
ដផែកលៅលលើលទនផល សលងេតរបស់ ក្រនន្េៈជាភាបសថាលសទិតលៅខាងមុខ លសរសនបិោឌនលលើសនលឹកកិចចការ
ពួកលគ លហើយលៅសិសសលឡើងសរ របស្សីដភបក។ លហើយសិសស ៣នាក់លឡើងសរលសរ
លសរសនបិោឌនលលើកាថរលខៀន។ ប្រស្សីភ្ននក្េៈ ជាលិការងវង់ោនពណ្៌ សនបិោឌនលលើកាថរលខៀន។
លមម។
រន្ធប្រស្សីេៈ របលហាងរាងមូលលៅ
កណា
ថ លរបស្សីដភបក។
ភ្ក្វភ្ននក្េៈ ដុាំោនរាងមូលថាល។
នរទីន្េៈ ជាស្សទប់លកាសិកាសទិតដផបក
ខាងលរកាយដនរោប់ដភបក (រួសនឹង
ពនលឺ)។
សរសសប្រសាទអុរទិក្េៈ ជាបាច់
សរដសតភាជប់ពីរោប់ដភបក
ជាំហានទី៤ ពរងឹងពុទនិ (៨១០នាទី)
-សួរសាំណ្ួរបាំផុសទក់ទងលៅនឹង ករមនេៈោននាទីការ ររបស្សី ដភបក -សិសសលឆលើយសាំណ្ួរលៅនឹង លោយ
មុខ ររបស់ដផបកលផសងៗរបស់រោប់ និងឱ្យពនលឺចូលកបងដភប
ុ កបាន។ ដផែកលលើការសលងេត ដផបកនីមួយៗ
ដភបក។ របស្សីផភនកេៈ ោននាទីរតួតពិនិតយ និងតាមរយៈ សាំណ្ួរបាំផុសរបស់
-បិតរូបភាពពីការតរមូវពនលឺរបស់ បរោណ្ពនល
ិ ឺចូលកបងដភប
ុ ក។ រគូ។
ដភបកែនិកសតវ រនធរបស្សីេៈឱ្យពនលឺឆលងចូលកបងដភប
ុ ក។
ផកវផភនកេៈោននាទីលធវើឱ្យោនកាំនុាំរូប
ភាពលលើលរទីន។
មរទីនេៈ ោននាទីរបមូលផថាំរុ ូបភាព
សរោប់បញ្ជូ នព័ត៌ោនលៅខួរកា
ល។
សរថ្សរបោទអុបទិកេៈ ោននាទី
(លរបើដកវពរងីកលដើមផីបញ្ញជក់ពីមុខ រ បញ្ជូ នព័ត៌ោនលៅខួរកាល។
របស់រោប់ដភបក)

ជាំហានទី៥ បណា
ថ លាំ ផញើ និងកិចចការផធេះ (៣៤នាទី)
-សូមពិនិតយលមើលរននរបស្សីលៅ -រននរបស្សីរកធ
ី ាំលៅទីងងឹត លហើយរនន -កត់រតា លហើយយកលៅលធវើពិលសាធ
កដនលងដដលោនពនលឺខាលង
ាំ និងកដនលង របស្សីរួមតូចលៅលពលោនពនលឺខាលង
ាំ ។ លៅផធេះ។
ដដលោនពនលឺលខាយ លហើយកត់រតា
ពីទាំហាំរបស់វា។
-សូមចាំលងលមលរៀនសលងខបោក់កង
បុ -នាទីដផបកនីមួយៗរបស់រោប់ដភបក -សាថប់លោយយកចិតថទុកោក់និង
លសៀវលៅ។ យកលៅអនុវតថ

45
-សូមអានលសៀវលៅបដនទម រតង់ -ខលម
ឹ សារកបងលសៀវលៅអតទ
ុ បទ ថាបក់ -សាថបល់ ោយយកចត
ិ ថទុកោក់ នង

ចាំណ្ុចដដលលយើងបានលរៀនលៅដែៃ ទី៨។ យកលៅអនុវតថ។
លនេះ (ទរមង់ដភបក និងគាំលហើញ)។

46
សន្លឹកកិច្ចការ

របធានបទ៖ ការពន
ិ ត
ិ យទរមង្់រូបផគុរាំ គាបផ់ ភនកថនក
ិ សតវ (រជូក)
១. សាំណួរគនលឹះឹ (និង្សមមតក
ិ មម)

សូមបញ្ញាក់ពីទរមង្់រូបផគុាំផភនករជូកទង្
ាំ អស់គាន!!!
២. ដាំមណើរការពិមោធ

ក. ពិនត
ិ យផផនកទរមង្់ខាង្មរៅ

ពិនត
ិ យដផបកខាងលរតរោប់ដភបកលដើមផីពិនត
ិ យដផបកមួយចាំនន ិ យ (របសីដភបក រននរបស្សី កញ្ច ក់ដភបក)
ួ ដដលអបកបានពិនត

ខ្. ពិនត
ិ យផផនកទរមង្់ខាង្កនុង្

១. លរបើឡាម ឬកាាំបិតកាត់ទទឹង ២. ដញកបាំដណ្កទាំងពីរ ដនរោប់ដភបក

រោប់ដភបកចាំ ក់កណា
ថ លដូច ដូចប ា ញកបងរូ
ុ ប។ លរបើដកវពរងីកលដើមផី
កបុងរូប។ លមើលដផបកនីមួយៗឱ្យចាស់។

ដកវដភបក

ករលន
៣. រកដផបកលផសងៗដនរោប់ដភបក ោនលៅកបងរូ
ុ ប សរដសរបសាទអុបទិក

ភាពលនេះលហើយសរលសរដផបកនីមួយៗលៅកបងរូ
ុ ប
រននរបស្សី
ភាពកបងលទន
ុ ផល។

លរទីន
៣. លទធផល របស្សី

គូររូប និងោក់ល្មេះដផបកលផសងៗរបស់រោប់ដភបក

ទរមង់លរតរោប់ដភបកលមើ លពីចលាំ ហៀង ទរមង់កងរោប់


បុ ដភបក

៤. សននដ្ឋ
ិ ឋន

......................................................................................................................................................................................
......................................................................................................................................................................................
......................................................................................................................................................................................

47
ប្លងកា
់ ា រខ ៀន្

របធានបទ៖ ការពន
ិ ត
ិ យទរមង្់រូបផគុរាំ គាបផ់ ភនកថនក
ិ សតវ (រជូក)

សាំណួរគនលឹះឹ , សមមតក
ិ មម
លទធផល

រូបសិសសគូរ រូបសិសសគូរ
ស រសស ប្រ សាទ អុ រ ទិក
ក រនេ កក វកនែក ប្រ ស្សដភប
ី ក
ទរមង់ខាង លរតរោប់ដភបក ទរមង់ខាង កបងរោប់
ុ ដភបក

នរ ទី េ រេធប្រ ស្សី

សននដ្ឋ
ិ ឋន
ក្រនន្េៈជាភាបសថាលសទត
ិ លៅ ខាងមុខរបស្សីដភបក។
សូមបញ្ញាកព
់ ទ
ី រមង្់រូបផគុាំផភនករជូកទង្
ាំ អសគា
់ ន !!! ប្រស្សីភ្ននក្េៈ ជាលិការងវងោ
់ នពណ្៌លមម។
រន្ធប្រស្សីេៈ របលហាងរាងមូល លៅកណា
ថ លរបស្សីដភបក។
ដាំមណើរការពិមោធ ភ្ក្វភ្ននក្េៈ ដុាំោនរាងមូលថាល។
(លមើលសនលក ច រ)
ឹ កិចកា នរទីន្េៈ ជាស្សទប់លកាសិកា សទិតដផបកខាងលរកាយដនរោប់ ដភបក (រួសនឹងពនលឺ )។

ក. ពិនត
ិ យផផនកទរមង្់ខាង្មរៅ សរសសប្រសាទអុរទិក្េៈ ជាបាច់សរដសតភាជប់ពីរោប់ដភបក

ខ្. ពិនត
ិ យផផនកទរមង្់ខាង្កនុង្

48
ដផបកលផសងៗរបស់រោប់ដភបក និងនាទីរបស់វា(លមលរៀនសលងខប)

ផផនករគាបផ់ ភនក លកខណេៈរគាបផ់ ភនក មុខ្ងាររបស់វា


ករលន ជាភាបសថាលសទត
ិ លៅខាងមុខរបស្សី ការ ររបស្សដី ភបកនង
ិ ឱ្យពនលច
ឺ ូល
ដភបក។ កបងដភប
ុ កបាន។
របស្សីដភបក ជាលិការងវង់ោនពណ្៌លមម។ រតួតពិនិតយបរោណ្ពនល
ិ ឺចូលកបង

ដភបក។
រននរបស្សី របលហាងរាងមូលលៅកណា
ថ ល ឱ្យពនលឺឆលងចូលកបងដភប
ុ ក។
របស្សីដភបក។
ដកវដភបក ដុាំោនរាងមូលថាល។ លធវើឱ្យោនកាំនុាំរូបភាពលលើលរទីន។

លរទីន ជាស្សទប់លកាសិកាសទិតដផបកខាង របមូលផថាំរុ ូបភាពសរោប់បញ្ជូ ន


លរកាយដនរោប់ដភបក (រួសនឹង ព័ត៌ោនលៅខួរកាល។
ពនលឺ)។
សរដសរបសាទអុបទិក ជាបាច់សរដសតភាជប់ពីរោប់ដភបក បញ្ជូ នព័ត៌ោនលៅខួរកាល។

49
កិច្ចតែងការបងរងៀន

របធានបទ្យ ការរ ំភាយច្ំហាយទ្យឹករប ់រុកខជាែិ

មុខ្វ ិជាជជីវវ ិទ្យា

ថ្ននក់ទ្យី៨ ងបាឹះពុមាឆ្នំ ២០១១ (ទ្យំព័រ ១៨៨ -១៩១)

ងមងរៀនទ្យី២ : ដំណឹកនាំកុងរុ
ន កខជាែិម្ពន រផ្ នាំ

១ .ដំណឹកនាំកុងរុ
ន កខជាែិោមន រផ្ នាំ

២ .ដំណឹកនាំកុងរុ
ន កខជាែិម្ពន រផ្ នាំ

២ .១.របព័នធដឹកនាំរប ់រក
ុ ខជាែិម្ពន រផ្ នាំ

២ .២ . ំរប
ូ ទ្យឹកនិងអំបិលខ្និជកនុងរុកខជាែិម្ពន រផ្ នាំ

ក . ំរប
ូ ទ្យឹក

ខ្ . ំរប
ូ អំបិលខ្និជ

រ . រ ំភាយច្ំហាយទ្យឹក (ច្ំណុច្រែូវបងរងៀន)

រយ:ងពល: ៦០នាទ្យី

១. វែថុបំណង:

- ច្ំងណឹះដឹង: ពនយល់ពីការរ ំភាយច្ំហាយទ្យឹករប ់រក


ុ ខជាែិបានរែឹមរែូវតាមរយ :ការពិងស្កធ។

- បំណិន: បំណិនកនុងការ ងងាែបាែុភូែរ ំភាយច្ំហាយទ្យឹកបានរែឹមរែូវ។

- ឥរ ិយាបង: តងរការុកខជាែិងោយងច្ឹះងស្រស្កច្ទ្យឹកបានរែឹមរែូវតាមងពលងវលា។

២. ម្ពារ:បងរងៀន

- ង ៀវងៅជីវវ ិទ្យាថ្ននក់ទ្យី៨

- នលឹកកិច្ចការពិងស្កធ

- ម្ពារ:ពិងស្កធម្ពន : ងដើមផ្ទារ ំងយាល ងង់បាល ិច្


ទ ថ្នលទ្យំ ំ (៤០ .ម x ៦០ .ម) ងៅ ូកង ឬ ុែ។

50
៣. កមមភាពបងរងៀន

កមភាពររូ ខ្លឹមស្ករ កមមភាព ិ ស


ជំហានទ្យី១ រដឋបាលថ្ននក់
រែួែពិនិែយវ ិន័យ ណា
ត ប់ធានប់ និងអ របមូលអារមមណ៍ ិ សមុនងពលចប់ របធានថ្ននក់ង ើងរាយការណ៍
វែតម្ពន ិ ស ងផតើមងមងរៀន
ជំហានទ្យី២ រ ំលឹកងមងរៀន
១. ងែើរបព័នធដឹកនាំរប ់រក
ុ ខជាែិម្ពន ១. របព័នធដឹកនាំរប ់រក
ុ ខជាែិម្ពន
អវីខ្លឹះ? ុីត ម និងផលូតអម។

២. ងែើ ុីត មម្ពននាទ្យីអីវ? ២. ុីត មម្ពននាទ្យីដឹកនាំទ្យឹកពីឬ


ងៅកាន់តផនកងផសងៗរប ់រក
ុ ខជាែិដូច្
ជាងដើម តមក និង ឹក
ល ជាងដើម។
ជំហានទ្យី៣ ងមងរៀនងមី
(បងាាញកូនរុកខជាែិោំកុងងផើ
ន ងតដល ងមងរៀន: - ិ ស ងងាែ
បានងរៀបច្ំរច្
ួ ឱ្យ ិ ស ងងាែ)
.ច្ូរបអូន ងងាែងមើល ថ្នងែើទ្យឹកងៅ -រែូវបានស្រ ូបងោយឬ ដឹកនាំងៅ
កនុងដីរែូវបានស្រ ូបងោយស្ករអវី? ដំណឹកនាំកុងរុ
ន កខជាែិ ឹក

ដឹកនាំងៅតផនកណា? ម្ពន រផ្ នាំ
.ងែើទ្យឹកងៅដល់ ឹក
ល រែូវបានរកាទ្យុក
- ទ្យឹកភាយងច្ញងៅកនងខ្យល់
ងៅកនុង ឹក ល ឬ?
.ងែើទ្យឹកតដលភាយងច្ញងៅកនុងខ្យល់
- រ ំភាយច្ំហាយទ្យឹក
ងរងៅថ្នអវី?
ដូច្ងនឹះផ្ងៃងនឹះងយើង ិកាពី
(ការរ ំភាយច្ំហាយទ្យឹករប ់រុកខជាែិ) រ.រ ំភាយច្ំហាយទ្យឹក
.រប ិនងបើ ខ្្ុំយកងង់បាល ិច្
ទ ថ្នលមក - ងៅកនុងងង់នឹងម្ពនែំណក់ទ្យឹក
ងស្រស្កបងលើតមករប ់វាតដលម្ពន ១. ំនួររនលឹឹះ ងកើែង ើង ។
ទំង ឹក
ល ងែើវានឹងម្ពនអវីងកើែង ើង
ងៅកនុងងង់បាល ិច្
ទ ?
ងែើរ ំភាយច្ំហាយទ្យឹករុកខជាែិរបរពឹែតងៅងៅតផនកណា?

២. មមែិកមម

ច្ូរស្កកលបងរិែងមើលថ្ន ងែើនឹង A - អាច្ម្ពនច្ងមលើយងផសងៗោន


ម្ពនអវីងកើែង ើងងៅកនុងងង់ទំងពីរ?
ង ែុអីវ?

- ងៅកនុងងង់ Aម្ពនែំណក់ទ្យឹក ងកើែ


A
- ងៅកនុងងង់ Aម្ពនែំណក់ទ្យឹក ងកើែ ង ើង ពីងរោឹះម៉ាូងលរុលទ្យឹកងច្ញពី
ង ើង ពីងរោឹះម៉ាូងលរុលទ្យឹកងច្ញពី ងកា ិកា ឹក
ល រុកខជាែិតាមរយ: ូម្ព៉ា

ងកា ិកា ឹក
ល រុកខជាែិតាមរយ: ូម្ព៉ា
ត ែ។
ែ។

51
B B

- ងៅកនុងងង់ Bោមនែំណក់ទ្យឹក ងកើែ - ងៅកនុងងង់ Bោមនែំណក់ទ្យឹកងកើែ


ង ើងងទ្យ ពីងរោឹះោមន ឹក
ល រុកខជាែិ ង ើងងទ្យ ពីងរោឹះោមន ឹក
ល រុកខជាែិ
ម៉ាូងលរុលទ្យឹកមិនអាច្ ភាយងច្ញតាម ម៉ាូងលរុលទ្យឹកមិនអាច្ភាយងច្ញ
ងដើមងទ្យ។ តាមងដើមងទ្យ។
៣.ដំងណើរការពិងស្កធ ិ សអាន នលឹកកិច្ចការ ិ ស
ឱ្យ ិ សអាន នលឹកកិច្ចការតណនាំ
ក. ម្ពារ: ស្កតប់ ិ សងធវើពិងស្កធ
ិ សឱ្យបំងពញ នលឹកកិច្ចការ
ងដើមផ្ទារ ំងយាល ងង់បាល ិច្
ទ ថ្នល ទ្យំ ំ
(៦០ .មx៤០ .ម) ុែឬងៅ
ា ូ កង
ខ្. ដំងណើរការ

-យកងង់បាល ិច្
ទ ថ្នលពីរងៅងស្រស្កប ងៅ
ងលើតមកផ្ទារ ំងយាលងដើមតែមួយ ងោយ
ងង់មួយងស្រស្កបងៅងលើតមក តដលម្ពន
ឹក
ល ។ ច្ំតណកងង់មួយ ងទ្យៀែងស្រស្កប
ងៅងលើតមករុកខជាែិ តដលោមន ឹក
ល ។
ងង់ទំងពីររែូវ ច្ងម្ពែ់វាជាប់នឹងតមក
- ទ្យុករយ:ងពលរបត ល១៥នាទ្យី។
ងងាែងមើលងៅកនុងងង់ទំងពីរម្ពន
លកខណ:ដូច្ងមតច្?

៤.លទ្យធផល
ច្ូរកែ់រតាលទ្យធផលងៅកនុងតារាងខ្នង ិ ស ងងាែ និងកែ់រតាលទ្យធផល
ឱ្យ ិ ស ងងាែ និងបំងពញកនុង
ងរកាម ោក់កុងតារាង

តារាងលទ្យធផល
ែង់A ែង់B
ែង់
៥ ១០ ១៥ ៥ ១០ ១៥
នាទី នាទី នាទី នាទី នាទី នាទី

-តាមលទ្យធផលខ្នងងលើងែើអនកបាន លកខណ្:
តិច លរចើន លរចើន ោមន ោមន ោមន

ងងាែង ើញយា៉ា ងដូច្ងមតច្?


តាមរយ:លទ្យធផល ងងាែង ើញថ្នងៅ
កនុងងង់Aតដលម្ពន ឹក
ល ផ្ទារ ំងយាលម្ពន ិ សពនយល់ពីលទ្យធផលតដលងរ
ែំណក់ទ្យឹកែូច្ៗជា ងរច្ើនងកើែង ើង ទ្យទ្យួលបាន

ជាប់នឹងងង់ ច្ំតណកងង់ Bតដលោមន


ឹក
ល ផ្ទា រ ំងយាលមិនម្ពនែំណក់ទ្យឹក
ងកើែង ើងងទ្យ។

52
ស្សទប់គុយទីន
លអពីដឌមស្សទប់លលើ
ពនយល់ពីទ្យរមង់ខ្ននែ់រប ់ ឹក

លកាសិកាលមសូភល

លអពីដឌមស្សទប់លរកាម
កលរបា
៉ោូ ល ស
(តាមការពនយល់ពីទ្យរមង់ ឹក
ល ឱ្យ
លកាសិកាលាត
ិ សងធវើការ ននិោឋន)
សថោ៉ោ
ូ ត

៥. ង ច្កតី ននិោឋន
ច្ូរងធវើការ ននិោឋន
ដូច្ងនឹះងយើងអាច្ទញង ច្កតី ិ សងធវើការ ននិោឋន
ននិោឋនថ្ន រ ំភាយច្ំហាយទ្យឹកងកើែ
ង ើងតាមរយ: ូម្ព៉ា
ត ែរប ់ ឹក

ងរកាមកងតតផ្ងៃ។
ជំហានទ្យី៤ ពរងឹងពុទ្យិធ

រ ាំភាយចាំហាយទឹក និងការស្សូបទឹក

តាមរកាបងនឹះ ងែើរ ំភាយច្ំហាយទ្យឹក រ ំភាយច្ំហាយទ្យឹកងៅងពលផ្ងៃងរច្ើន


រ ាំភាយចាំហាយទឹក
ងពលផ្ងៃ និងងពលយប់ដូច្ោនតដរ ជាងងៅងពលយប់។
អរតារ ាំភាយចាំហាយទឹក (9/h)

ទឹករស្សូប

ឬងទ្យ?

៦រពឹក ដែៃរតង់ ៦ោៃច កណា


ថ លយប់

ងែើអនករួរងស្រស្កច្ទ្យឹករុកខជាែិងៅ ងយើងរួរងស្រស្កច្ទ្យឹកងៅងពលរពឹក
ងពលណាងទ្យើប មស្រ ប? ច្ូរពនយ រ ូែដល់មុនងពលផ្ងៃរែង់ ពីងរោឹះ
ល់។ ចប់ពីងពលរពឹកងលបឿនផ្នការរ ំភាយ
ច្ំហាយទ្យឹកចប់ងផតើមងកើនង ើង ម្ពន
ន័យថ្នការស្រ ូបទ្យឹកក៍ងកើនង ើងតដរ
ជំហានទ្យី៥ បណា
ត ំ ងផ្ើ

កិច្ចការផទឹះ
ច្ូរបអូនស្កកលបងងធវើពិងស្កធងនឹះងលើ ស្កតប់ និងយកងៅអនុវែត
រុកខជាែិងផសងងទ្យៀែ។

53
នលឹកកិច្ចការ

របធានបទ្យ ការរ ំភាយច្ំហាយទ្យឹករប ់រក


ុ ខជាែិ

វែថុបំណង: ងងាែការរ ំភាយច្ំហាយទ្យឹករប ់រក


ុ ខជាែិ។

១. ំណួររនលឹឹះ - ទ្យុករយ:ងពលរបត ល១៥នាទ្យី។ ងងាែងមើលងៅកនុងងង់ទំងពីរម្ពនលកខណ :ដូច្

ងមតច្?
ងែើការរ ំភាយច្ំហាយទ្យឹករប ់រក
ុ ខជាែិរបរពឹែតងៅងៅតផនកណា?

៤.លទ្យធផល
២. មមែិកមម
ច្ូរកែ់រតាលទ្យធផលងៅកនុងតារាងខ្នងងរកាម
តាមរយ :ការ ងងាែខ្នងងលើ ច្ូរស្កកលបងរិែថ្ន ងែើម្ពនអវីងកើែង ើងងៅកនុងងង់

ទំងពីរ?
ែង់A ែង់B
-ងៅកនុងងង់ A:…………………………………………………………………….…… ែង់
៥នាទី ១០នាទី ១៥នាទី ៥នាទី ១០នាទី ១៥នាទី
………………………………………………………………………………… លកខណ្:
-ងៅកនុងងង់ B:………………………………………………………………………
……………………………………………………………………………………… ៥. ង ច្កតី ននិោឋន

តាមលទ្យធផលខ្នងងលើងែើអនកបាន ងងាែង ើញយា៉ា ងដូច្ងមតច្? ច្ូរងធវើការ ននិោឋន។


៣.ដំងណើរការពិងស្កធ ……………………………………………………………………………………
ក. ម្ពារ: ……………………………………………………………………………………….……
ងដើមរុកខជាែិ ងង់បាល ិច្
ទ ថ្នលទ្យំ ំ (៦០ .មx៤០ .ម) ុែ
ា ឬងៅ ូកង។ …………………………………………………………………………………………….
ខ្. ដំងណើរការ

- យកងង់បាល ិច្
ទ ថ្នលពីរងៅងស្រស្កបងៅងលើតមករុកខជាែិងដើមតែមួយ ងោយងង់

មួយងស្រស្កបងៅងលើតមកតដលម្ពន ឹក
ល ។ ច្ំតណកងង់មួយងទ្យៀែងស្រស្កបងៅងលើតមករុកខ

ជាែិតដលោមន ឹក
ល ។ ងង់ទំងពីររែូវច្ងម្ពែ់វាឱ្យជិែ ង ើយជាប់នឹងតមក។

54
បលង់កាតរងខ្ៀន

របធានបទ្យ ការរ ំភាយច្ំហាយទ្យឹករប ់រុកខជាែិ

ងៅកនុងងង់ងនឹះអនក ងងាែង ើញអវី? តាមរយ:លទ្យធផល ងងាែង ើញថ្នងៅកនុងងង់ Aតដលម្ពន ឹក


ល ផ្ទារ ំងយាលម្ពន

១. ំណួររនលឹឹះ ែំណក់ទ្យឹកែូច្ៗជាងរច្ើនងកើែង ើងជាប់នឹងងង់ ច្ំតណកងង់ B តដលោមន ឹក


ល ផ្ទារ ំងយាល

មិនម្ពនែំណក់ទ្យឹកងកើែង ើងងទ្យ។
ងែើការរ ំភាយច្ំហាយទ្យឹករប ់រក
ុ ខជាែិរបរពឹែតងៅងៅតផនកណា?

៥. ង ច្កតី ននិោឋន
២. មមែិកមម

តាមរយ :ការ ងងាែខ្នងងលើ តាមលទ្យធផលខ្នងងលើងែើអនកបាន ងងាែង ើញយា៉ា ងដូច្ងមតច្?

-ងៅកនុងងង់ Aម្ពនែំណក់ទ្យឹកងកើែង ើង ពីងរោឹះម៉ាូងលរុលទ្យឹកងច្ញពីងកា ិកា តាមលទ្យធផលពិងស្កធន៍ងយើងអាច្ទញង ច្កតី ននិោឋនថ្ន រ ំភាយច្ំហាយទ្យឹក

ងកើែង ើងតាមរយ: ូម្ព៉ា


ត ែរប ់ ឹក
ល ងរកាមកងតតផ្ងៃ។
ឹក
ល រុកខជាែិតាមរយ: ូម្ព៉ា
ត ែ។

-ងៅកនុងងង់ Bោមនែំណក់ទ្យឹកងកើែង ើងងទ្យ ពីងរោឹះោមន ឹក


ល រុកខជាែិម៉ាូងលរុល
ស្សទប់គុយទីន
លអពីដឌមស្សទប់លលើ
ទ្យឹកមិនអាច្ភាយងច្ញតាមងដើមងទ្យ។

៣.ដំងណើរការពិងស្កធ
លកាសិកាលមសូភល

ងរៀបរាប់ ម្ពារ :ពិងស្កធ និងងរៀបច្ំពិងស្កធ

៤.លទ្យធផល លអពីដឌមស្សទប់លរកាម

កលរបា
៉ោូ ល ស
ច្ូរកែ់រតាលទ្យធផលងៅកនុងតារាងខ្នងងរកាម

ងង់A ងង់B
ងង់ សថូោ៉ោត
លកាសិកាលាត
៥នាទ្យី ១០នាទ្យី ១៥នាទ្យី ៥នាទ្យី ១០នាទ្យី ១៥នាទ្យី

លកខណ: ម្ពន ម្ពន ម្ពន ោមន ោមន ោមន

ែំណក់ទ្យឹក ែំណក់ទ្យឹក ែំណក់ទ្យឹក

ែិច្ៗ ងរច្ើន ងរច្ើន

55
កិចផច តង្ការ

របធានបទៈ ចលនាាមឆនទេៈ នង្


ិ មរផលច

មុខវ ិជាជៈ ជីវវ ិទា

ថាបក់ទី៩ (កមមវ ិធីសិកាែមី)

ជាំពូកទី២: របព័ននរបសាទ

លមលរៀនទី៣: បរមណ្
ិ ឍ លរបសាទ

២. ចលនាតាមឆនធៈ និងលរផលិច

រយៈលពលៈ ៤៥នាទី

វតថុបាំណង្េៈ

 ចាំលណ្េះដឹង

ពនយលព
់ ច
ី លនាលឆលយ
ើ តបលោយឆនធៈ នង
ិ លរផលិចបានរតម
ឹ រតូវតាមរយៈការលធវស
ើ កមមភាព នង
ិ សណ្
ាំ ួ របផ
ាំ ុស។

 បាំណ្ិន

 កណ្
ាំ តបា
់ នពរី បភពទទួលពត
័ ោ
៌ ន នង
ិ ការបញ្ជូ នពត
័ ោ
៌ នបានរតម
ឹ តាមរយៈការគូសរពួញលៅលលរើ ូបភាព។

 ឥរោបែ

 ោនទោលបហា
់ តល់ រៀនលដើមផីលធវឱ្
ើ យចលនាលឆលយ
ើ តបលោយឆនធៈោនភាពរហស
័ លែ។

សមាារេៈ

បនាធត់ (៥០សម) ហវឺត សនលឹកកិចចការ...

ដាំណឹកនាមាំ មមរៀនេៈ

សកមមភាពរគូ ខ្លឹមោរមមមរៀន សកមមភាពសិសស


ជាំហានទ១
ី លន
ាំ ង
ឹ ថាបក់ (២៣នាទ)ី
.សាវគមន៍សិសស .សាវគមន៍ និងរដឌបាលថាបក់ .សាវគមន៍រគូ
.ពនត
ិ យលមល
ើ កបងថា
ុ ប ក់ .សាថបរ់ គូ
ជាំហានទី២ រល
ាំ ឹកលមលរៀនចាស់ (៣៥នាទី)
.លតើលៅកបងបរ
ុ មណ្
ិ ឍ លរបសាទោន កាយវ ិភាគវ ិទាបរមណ្
ិ ឍ លរបសាទ .របព័ននរបសាទសូោទ
៉ោ ិច និងរបព័នន
របព័ននរបសាទអវីខលេះ? របសាទសវ័យរបវតថិ

56
លតើរបព័ននរបសាទសូោទ
៉ោ ិចោនមុខ រ របព័ននរបសាទសូោទ
៉ោ ិច .របព័ននរបសាទសូោទ
៉ោ ិចរតួតពនិតយនិង
អវី? សរមបសរមួលចលនាឆនធៈរបស់
សារ ងគកាយ។
.លតើរបព័ននរបសាទសវ័យរបវតថិោនមុខ របព័ននរបសាទសវ័យរបវតថិ .របព័ននរបសាទសវ័យរបវតថិ តរមូវចលនា
រអវី? អឆនធៈដនសររាងគ
ី កង។បុ

.លតើអបកអាចឱ្យឧទហរណ្៍ពីរបព័នន របព័ននរបសាទសូោទ
៉ោ ិចសទិតលៅលរកាម .សូោទ
៉ោ ិច៖ ការរត់ ការនិោយ
របសាទសូោទ
៉ោ ិច និងរបសាទ ការរតួតរតារបស់ខួរកាល ការលរចៀង....។
សវ័យរបវតថបានលទ? របព័ននរបសាទសវ័យរបវតថិមិនលៅលរកាម សវ័យរបវតថ៖ ច វ ក់លបេះដូង ការរោយ
ាំ
ការរតួតពិនិតយពីខួរកាលលទ អាហារ..។

ជាំហានទី៣ លមលរៀនរបចាដាំ ែៃ (២៥៣០នាទី)


ចាំណា៖ាំ រគូនឹងប ា ញពីគរមូដនចលនា ឃាលភាជប់លមលរៀន
លោយលរផលិចដល់សិសសជាលាំនាប
ាំ ញ្ញា។
លដើមផីប ា ញពីចលនា លឆលើយតបលោយ
លរផលិច រគូរតូវលធវើឱ្យសិសសភាញក់លផែើលដូច
ជារតូវនិោយដូចខាងលរកាមលនេះ៖
រគប់ចលនាទង
ាំ អស់រតូវការថាមពល
ដូចលនេះមុននឹងតលមលរៀនដែៃលនេះរគូនឹង
ដចកនាំឱ្យលយើងញុាំ លដើមផីទទួលបានថា
មពល។
. រគូោក់រតីឬកដងេបកបងែង់
ុ ឫរបអប់នាំ របធានបទៈ ចលនាាម ឆនទេៈ និង្មរផលិច .សិសសនារោ
ី ប ក់លឡើងមកលូកដដចូលកបង

(លរៀបចាំជាលស្សច លោយមិនឱ្យសិសស លាំនាប
ាំ ញ្ញា និងចលនាលឆលើយតបលោយ ែង់លហើយដស្សកលឡើងរពមទង
ាំ ដកដដ
លឃើញ) លហើយនិោយថារគូនឹងដចកនាំ លរផលិច លចញោ៉ោងរហ័ស។
ឱ្យលយង
ើ ញុអ
ាំ បកណាសមរ័ គចត
ិ ថលឡង
ើ យក
មុនលគ?

ចណា
ាំ ៖ាំ តាមដបបអនាធកខា
់ ងលលស
ើ ស
ិ ស
នង
ឹ ភាញកល់ ផែើល លហយ
ើ ដកដដរបសល់ គ
លចញពក
ី ញ្ច បន
់ ោ
ាំ ង
៉ោ រហស
័ ។ របសន

លបអ
ើ នាធកល់ នាេះទទួលបានលជាគជ័យវាជា
គរមូមួយដន ចលនាលឆលយ
ើ តបលោយលរផលិ
ច។ រគូអាចលរៀបចាំតាមវ ិធីលផសងលទៀត
លដើមផីលធវើឱ្យសិសសោនលរផលិច។
របសិនលបើរគូពុាំទទួលបានលជាគជ័យ
តាមវ ិធីលនេះរគូអាចលធវើសកមមភាពលផសង

57
លទៀត ឬក៍អាចរបាប់សិសសពីសមមភាព
លផសងៗដូចជាការដកដដលចញោ៉ោង
រហ័សលៅលពលដដលប៉ោេះនឹងវតទលត
ុ ថ និង
រពិចដភបកលៅលពលោនអវីមួយមកជិត
ដភបក។

.លហតុអបា
វី នជានាងដកដដលចញោង
៉ោ (១) ប៉ោេះលោយរោមដដ (ទទួពត
័ ោ
៌ ន) .នាងបេះ៉ោ អវម
ី ួយដដលលធវឱ្
ើ យនាងភាញក។

(២) ពីរោមដដលៅខួរឆែង
ឹ ខបង
រហស
័ ?
ឹ ខបងលៅដដវ ិញលដើមផី លឆលយ
(៣) ពីខួរឆែង ើ តបជា
.លតព
ើ ត
័ ោ
៌ នទទួលបានតាមណា លហយ
ើ លរផលច

.គូសពទ
ី ស
ិ លៅទទួលបញ្ជូ ននង
ិ ការ
រតូវបានបញ្ជូ ន នង
ិ លឆលយ
ើ តបោង
៉ោ ដូច (៤) ដកដដលចញ (លឆលយ
ើ តបព័ត៌ោន) លឆលយ
ើ តបព័ត៌ោនលៅលលកា
ើ ថ រលខៀន។
លមថច? សូមគូសពទ
ី ស
ិ លៅពត
័ ោ
៌ នលៅ (៥) ព័ត៌ោនពីខួរឆែង
ឹ ខបងក៍បញ្ជូ នលៅខួរកាល

លលើរូបភាព។ (បិតរូបលលើកាថរលខៀន) កបងលពលដតមួ


ុ យផងដដរ។

.លតើព័ត៌ោនដដលទទួលបាន រតូវឆលង .ព័ត៌ោនដដលទទួលបានតាម .ព័ត៌ោនមិនឆលងកាត់ខួរកាលលទ។ វា


កាត់ខួរកាលឫលទមុននឹងលឆលើយតប? រយៈរោមដដរតូវបញ្ជូ នតាម ដដរហូត ឆលងកាត់ខួរឆែឹងខបង លហើយបញ្ជូ នរតលប់
ដល់ខួរឆែឹងខបង លហើយព័ត៌ោនរតូវ ពីខួរឆែឹងខបងមកដដដតមថង។
បញ្ជូ នរតលប់ពីខួរឆែឹងខបង មកដដវ ិញ
លដើមផីលឆលើយតប។ លហើយព័ត៌ោនក៍រតូវ
បានបញ្ជូ នលៅខួរកាលកបងលពលដត

មួយផងដដរ។
.លតើោនចលនាអវីដដលសទិតលៅលរកាម .ោនចលនាលរចើនណាស់ដដលសិទតលៅ .ោនលរចើន
ការបញ្ញជពីខួរកាលឫលទ? លរកាមការបញ្ញជពីខួរកាល។

បងាាញពីការទ្យំលាក់បនាទែ់ងៅច្ងនាលឹះ ំណួររនលឹឹះ
រម្ពមផ្ដ និងងមផ្ដ។
មតើមយើង្អាចចបប
់ នាទតបា
់ នមៅរតង្់ចាំណុចAផដរឬមទ?

58
សរលសរអកសរA លៅរតង់ចាំណ្ុច០cm B សមមតិកមម .សិសសសរលសរសមមតិកមម លៅលលើកាថ
លៅរតង់ចាំណ្ុច២៥ cm និងC លៅ រតង់ .លយើងអាចចាប់បានរតង់ រលខៀន លហើយពនយល់ពីអតទន័យ និងមូល
ចាំណ្ុច៥០cm លៅលលើបនាធត់ដដលោន ចាំណ្ុច A (០-៥សម) ឬក៍លទ លហតុលគទសសន៍ទយ។
របដវងសរុប ៥០cm ោក់បនាធត់លៅ លយើងមិនអាចចាប់បានរតង់ (ចាប់បានរតង់ចាំណ្ុចA ោនន័យថាការ
ចលនាលេះលមដដ និងរោមដដ លហើយសួរ ចាំណ្ុចA លទដូលចបេះលយើងអាច៖ លឆលើយតប ព័ត៌របស់របព័ននរបសាទោន
សិសសថារបសិន លបើរគូោក់បនាធត់លៅ -ចាប់បានរតង់ចាំណ្ុច B ភាពរហ័ស។ ចាប់បានរតង់ចាំណ្ុចB
ចលនាលេះដដបែនដបបលនេះលត
ូ ើបនន
ែូ ឹងចាប់ (២០-២៥សម) ោនន័យថារបព័ននរបសាទរតូវការលពល
បានលៅរតង់ចាំណ្ុចណា? -ចាប់បានរតង់ចាំណ្ុច C លវោលដើមផីលឆលើយតប។ ចាប់បានរតង់
ឱ្យសិសសពនយល់ពីមូលលហតុនិងអតទ (៤៥-៥០សម) ចាំណ្ុចC ោនន័យថារបព័ននរបសាទរតូវ
ន័យដនការលរជើសលរសរបស
ើ ់ពួកលគ។ ការលពលលវោលរចើនលដើមផីលឆលើយតប។)
ដាំមណើរការពិមោធ
ដូចលៅកបងសនល
ុ ក
ឹ កច
ិ ចការ

.លរជើសលរសលទន
ើ ផលរបសស
់ ស
ិ ស៥រកុម លទធផល
លហយ
ើ ឱ្យោតល់ ឡង
ើ សរលសរលៅលលកា
ើ ថរ
លខៀន។

លលើកទី១ លលើកទី២ លលើកទី៣ លលើកទី៤ លលើកទី៥ សរុប មធយម

រកុម១ ១៨សម ១៩សម ២២សម ១៩សម ២៣សម ១០១ ២០.២


រកុម២ ២៥សម ២៨សម ២៤សម ២២សម ២០សម ១១៩ ២៣.៨
រកុម៣ ៣០សម ២៨សម ២៥សម ៣០សម ២៨សម ១៤១ ២៨.២
រកុម៤ ២៥សម ១៨សម ២២សម ២០សម ២០សម ១០៥ ២១
រកុម៥ ១៩សម ២៣សម ២៥សម ២២សម ២៦សម ១១៥ ២៣
ចាំណា៖ាំ ជាទូលៅសិសសអាចចាប់បានលៅរតង់ចណ្
ាំ ុ ច១៥ លៅ២០សម។ សិសសមិនអាចចាប់បានរតង់ចណ្
ាំ ុ ចតិចជាង
១០សមលទ។ របសិនលបើសិសស អាចចាប់បានរតង់ចណ្
ាំ ុ ចតិចជាង១០សម ោនន័យថាសិសសគិតទុកមុនថាបនាធត់នង

រតូវទាំោក់ គឺោនដមនលឃើញបនាធត់ធាលក់លហើយលទើបចាប់លទ។ របសិនលបើលគអាចចាប់បានរតង់ចណ្
ាំ ុ ច២០សម ោនន័យ
ថារបព័នរន បសាទរតូវការលពល០,២វ ិនាទីលដើមផីលឆលយ
ើ តប។

សននដ្ឋ
ិ ឋន .សរលសរសនបោ
ិ ឌ នលៅលលើ សនលក
ឹ កច
ិ ចការ
ឱ្យសស
ិ សលរបៀបលធៀបលទនផលលៅនង
ឹ តាមរយៈលទនផលពលិ សាធ លយង
ើ អាច លហយ
ើ សស
ិ សមួយ ឫពរី នាកល់ ឡង

សមមតក
ិ មមរបសព
់ ួកលគ លហយ
ើ ទញសនបិ សនបោ
ិ ឌ នបានថា របពន
័ ធរបោទរតូវការ សរលសរសនបោ
ិ ឌ នលៅលលកា
ើ ថ រលខៀន។
ោឌន។ មពលមវលាមួយមដើមបីមឆលើយតប នង្
ឹ ពត
័ មា
៌ ន
ផដលទទួលបាន។
.បត
ិ រូបលៅលលកា
ើ ថ រលខៀន លហយ
ើ ឱ្យសស
ិ ស . ពត
័ ោ
៌ នរតូវបានទទួលតាមរយៈដភបក
លឡើងគូរពីដផបកដដលទទួល និងបញ្ជូ ន រួចបញ្ជូ នលៅខួរកាលមុននឹងបញ្ជូ ន
ព័ត៌ោន។ លៅដដលដើមផីលធវើចលនា។

59
ដូចលនេះការលឆលយ
ើ តបរបសរ់ បពន
័ ន
របសាទទង
ាំ ឡាយដដលសទត
ិ លៅលរកាម
ការរតួតពន
ិ ត
ិ យលោយខួរកាលជា
ចលនាលឆលយ
ើ តបលោយឆនធៈ។
ពនយលព ើ វ ិញ
់ រី ូបទី១លឡង
ដូចលនេះការលឆលយ
ើ តបរបសរ់ បពន
័ ន .ព័ត៌ោនរតូវបានបញ្ជូ នតាម រយៈដដលៅ
របសាទទង
ាំ ឡាយ ដដលមន
ិ សទត
ិ លៅ ខួរឆែឹងខបង លហើយរតលប់មកដដវ ិញ
លរកាមការរតួតពន
ិ ត
ិ យលោយខួរកាលជា លោយមិនឆលងកាត់ខួរកាលលទ។
ចលនាលឆលើយតបលោយលរផលិច។
ជាំហានទ៤
ី ពរងឹងពុទនិ (៤៥នាទី)
ឱ្យសិសសបាំលពញរបលឡាេះលៅ កបងសនល
ុ ឹក កយបាំលពញកបងរបលឡាេះ
ុ សិសសបាំលពញលរៀងៗខលនកប
ួ ងសនល
ុ ឹកកិចច
កិចចការ លហើយដកលមែចលមលើយសិសស ខួរឆែឹងខបង, ខួរកាល, សររាងគ
ី ចលករ, ការ
ខួរកាល, យឺតជាង, លរផលិច
ជាំហានទ៥
ី កិចចការផធេះ (២៣នាទី)
លយើងដឹងលហើយថាចលនាលឆលើយតប ការហវឹកហាត់លធវើឱ្យចលនា លឆលើយតប សិសសសាថប់ និងយកលៅអនុវតថ
លោយឆនធៈភាពខុសោប ពីមនុសសោបក់ លោយឆនធៈោនភាពរហ័ស។
លៅមនុសសោបក់លទៀតចលនាឆនធៈអាច
លលឿន លៅបានលុេះរតាដតោនការហាត់
លរៀន។ ដូចសូមបែនលធវ
ូ ើ ការហាត់លរៀនឱ្យ អានលមលរៀនបដនទម លដើមផីលផធៀងផាធត់និង
បានលរចើន លដើមផីហវឹកហាត់ចលនាឆនធៈ ពរងីកចាំលណ្េះដឹងបដនទម
របស់លយើង។ លហើយសូមបែន
ូ លមើលលម
លរៀនបដនទមលទៀតលៅទាំព័រ១៩២ លៅកបង

លសៀវលៅសិការបស់លយើង។

60
សនលឹកកិចចការ
របធានបទៈ ចលនាតាមឆនធៈ និងលរផលិច
១. សាំណួរគនលឹឹះ
លតើលយើងអាចចាប់បនាធត់បានលៅរតង់ចាំណ្ុច A ដដររលទ?

២. សមមតក
ិ មម
......................................................................................................................................................................................................
......................................................................................................................................................................................................
................................................................................................................................................
៣. ដាំមណើរការពិមោធ
លធវើការជាដដគូ (ោបក់ជាអបកចាប់ លហើយោបក់លទៀតជាអបកទោលក់ )
- គូសបនាធត់ចាំ ក់កណា
ថ លរកចកលមដដរបស់អបកចាប់ លហើយអបកចាប់រតូវអងគយលៅលល
ុ ើលតអី លោយោក់កាំភូនដដលលើ
ដគមតុ (លមល
ើ រូបភាព)។
- អបកទោលក់ោក់បនាធត់លៅចលនាលេះលមដដ និងរោមដដ
(របដហល២សម) របស់អបកចាប់លោយោក់ចាំណ្ុចសូនយលៅលលើបនាធត់
ឱ្យលសមើនឹងគាំនូសលៅលលើរកចកលមដដ និងរោមដដរបស់អបកចាប់ ន ើយមិន្ប្រូវឱ្យ
រន្ទាររ
់ ៉ះន្ង
ឹ នមសដ ន្ង
ិ ប្ាមសដនសេងនទៀរររសអ
់ នក្ចារន់ ទ។
- អបកចាប់រតូវសាំឡឹងលមើលបនាធត់រតង់លលខសូនយ។
លហើយរង់ចាចា
ាំ ប់បនាធត់លៅលពលអបកទោលក់ ទោលក់បនាធត់ចុេះ។
(មិនរតូវចាប់មុនអបកទោលក់បនាធត់ចុេះលទ)
- កត់រតាចោៃយដដលោត់ចាប់បាន លៅកបងតារាងលទន
ុ ផល។
លធវើតាមរលបៀបខាងលលើលនេះចាំនួន៥ដង លហើយគណ្នាចោៃយរបស់វា។

៤. លទធផល
បលាំ ពញលទនផលលៅកបងតារាងខាងលរកាម៖

ទោលក់ លលើកទី១ លលើកទី២ លលើកទី៣ លលើកទី៤ លលើកទី៥ សរុប មធយម
ចោៃយលលប
ើ នាធត់
(សម)

61
៥. សននិដ្ឋឋន
........................................................................................................................................................................................
........................................................................................................................................................................................
........................................................................................................................................................................................
........................................................................................................................................................................................
........................................................

ពរង្ឹង្ពុទធិ

ចូរបាំលពញរបលឡាេះឱ្យបានរតឹមរតូវ៖
១. ចាំល េះចលនាលឆលើយតបលោយលរផលិចព័ត៌ោន (អាង
ាំ ភលចរបសាទ)
ុ ដដលទទួលបានរតូវឆលងកាត់...................លហើយ
ី ចលករវ ិញលោយមិនឆលងកាត់ ..................លឡើយ។ អាង
បញ្ជូ នលៅសររាងគ ាំ ភលចរបសាទដដលបញ្ជ
ុ ូ នលៅកាន់ខួរកាល
និងលៅកាន់.........................លៅកបងលពលដតមួ
ុ យ។
២. ចាំល េះចលនាលឆលើយតបលោយឆនធៈព័ត៌ោន (អាង
ាំ ភលចរបសាទ)
ុ ដដលទទួលបានរតូវឆលងកាត់..................សិនមុន
នង
ឹ បញ្ជូ នលៅសររាងគ
ី ចលករ។ លហតុលនេះលហយ
ើ បានជាចលនាលឆលយ
ើ តមលោយឆនធៈោនភាព............................ជាង
ចលនាលឆលើយតបលោយ...................។

62
បលង្កា
់ េ រមខ្ៀន
របធានបទៈ ចលនាតាមឆនធៈ និងលរផលិច

១. សាំណួរគនលឹឹះ
លតើលយើងអាចចាប់បនាធត់បានលៅរតង់ចាំណ្ុច A ដដររលទ?

២. សមមតិកមម
លយើងអាច ចាប់បានរតង់ចាំណ្ុច A (០-៥សម) ឬក៍លទ លយើងមិនអាចចាប់បានរតង់ចាំណ្ុចA លទដូលចបេះលយើងអាច៖
-ចាប់បានរតង់ចាំណ្ុច B (២០-២៥សម)
-ចាប់បានរតង់ចាំណ្ុច C (៤៥-៥០សម)
៣. ដាំមណើរការពិមោធ
ដូចលៅកបងសនល
ុ ឹកកិចចការ
៤. លទធផល

លលើកទី១ លលើកទី២ លលើកទី៣ លលើកទី៤ លលើកទី៥ សរុប មធយម

រកុម១ ១៨សម ១៩សម ២២សម ១៩សម ២៣សម ១០១ ២០.២


រកុម២ ២៥សម ២៨សម ២៤សម ២២សម ២០សម ១១៩ ២៣.៨
រកុម៣ ៣០សម ២៨សម ២៥សម ៣០សម ២៨សម ១៤១ ២៨.២
រកុម៤ ២៥សម ១៨សម ២២សម ២០សម ២០សម ១០៥ ២១
រកុម៥ ១៩សម ២៣សម ២៥សម ២២សម ២៦សម ១១៥ ២៣

៥. សននិដ្ឋឋន

តាមរយៈលទនផលពិលសាធ លយើងអាចសនបិោឌនបានថា របពន


័ ធរបោទរតូវការមពលមវលាមួយមដើមបីមឆលើយតប នឹង្ពត
័ មា
៌ ន
ផដលទទួលបាន។

63

(5
)
(3
) (1
)

(2

)


➁ ➃
(4
)

ករណីចលនាមឆលយ
ើ តបមដ្ឋយមរផលច
ិ ករណ្ីការលឆលយ
ើ តបលោយឆនធេះ

➀ ប៉ោេះអវម
ី ួយចដមលកលៅចុងរោមដដ (1) លមើលលឃើញបនាធត់ធាលក់លោយដភបក
(ទទួលព័តោ
៌ ន) (ទទួលព័ត៌ោន)

➁ ព័ត៌ោនបញ្ជូ នតាមរបព័នរន បសាទ លចញពី (2) ព័ត៌ោនលចញពីដភបកលៅដផបកទទួលគាំលហើ


រោមដដចូលលៅកាន់ខួរឆឆង
ឹ ខបង។ ញដនខួរកាល លហើយទទួលដឹងវា។

➂ ព័ត៌ោនបញ្ជូ នរតឡប់ពីខួរឆឆង
ឹ ខបងលៅកាន់ (3) ព័ត៌ោនបញ្ជូ នពីតាំបន់គាំលហើញលៅតាំបន់
សាច់ដុាំដដលដើមផីលឆលយ
ើ តប។ ចលនាកបងខួ
ុ រកាល។

➃ សាច់ដុាំដដដកដដោ៉ោងរហ័ស (លឆលយ
ើ តប (4) លចញពីតាំបន់ចលនាព័ត៌ោនរតូវបាន
ព័ត៌ោន) បញ្ជូ នតាមរយៈខួរឆែង
ឹ ខបងលៅរោមដដ
លដើមផីចាប់បនាធត់។
➄ ព័ត៌ោនពីរោមដដក៍បញ្ជូ នលៅខួរកាលផង
ដដរ លដើមផីទទួលអារមមណ្៍។
(5) ចាប់បនាធត់ដដលធាលក់លោយរោមដដ។
(លឆលយ
ើ តបព័ត៌ោន)

64
65
ការមរៀបចាំសកមមភាពគរមូ (Jisoyoteiji) មៅចុង្ជាំហានទី២ថ្នមមមរៀន។

ការមរៀបចាំកញ្ច បន
់ ាំជាមុន
រគូរតូវលរៀបចាំែង់នាំរួចជាលស្សច ដដលលៅកបងលនាេះោត
ុ ់ោក់វតទដបល
ុ កឧទហរណ្៍
ដូចជា រតី កដងេប ទឹកលតថអុនៗ ឬទឹកកក លហើយទុកវាលៅដដលសិសសមិន
លឃើញ ដូចជាោក់លៅលរតបនធប់ជាលដើម។ សូមលរជើសលរសមួ
ើ យ ដដលអបកគិត
ថាសមស្សបបាំផុត លហើយលរៀបចាំឲ្យបានមុនចាប់លផថើមបលរងៀន។ លៅកបងការ

បលរងៀន រគូឲ្យសស
ិ សោបកល់ ឡង
ើ មកយកនព
ាំ ក
ី ងែង់
បុ ។ ជាការពត
ិ សស
ិ សមន
ិ ដឹង
ថាវាជាអវីលនាេះលទ។ រគូរតូវដតលធវើឲ្យសិសសទុកចិតថ ដូលចបេះរគូរតូវដតយកនាំញុាំ
លហើយឲ្យសិសសលូកយកនាំ។
រគូរតូវដតរបុងរបយ័តបលោយមិនឲ្យសិសសអវីដដលលៅកបងែង់
ុ លនាេះលទ។

សិសសលូកយកនាំ
លៅលពលលរៀបចាំលធវើសកមមភាពលនេះ រគូរតូវលរជើសលរសទ
ើ ីតាង
ាំ ដដលលែលដើមផី
ប ា ញពីសកមមភាពលឆលើយតបរបស់នាងលៅកាន់សិសសទង
ាំ អស់លៅកបងថា
ុ ប ក់។

បនាធប់មកនាងក៍លផញើកលផែើល លហើយដកដដោ៉ោងរហ័ស ពីលរ េះនាងបានប៉ោេះអវី


ដដលចដមលកលៅកបងែង់
ុ ។

របសិនលបើសកមមភាពលនេះលកើតលឡើងោ៉ោងរហ័ស លោយមិនបានដឹងជាមុនអាំពី
អវលី ៅកបងកញ្ច
ុ ប់ សកមមភាពលនេះគទ
ឺ ទួលបានលជាគជ័យលហយ
ើ ។ លនេះគជា

ឧទហរណ្៍មួយដនការលឆលើយតបលោយលរផលិច។

របសិនលបើនាងបានដឹងអាំពីអវីលៅកបងកញ្ច
ុ ប់(ដូចជា រត)ី លហើយនាងដកដដលច
ញ។ លនេះមិនដមនជាឧទហរណ្៍ដនការលឆលើយតបលោយលរផលិចលទ។ រគូគួរដតបត់
ដបនការបលរងៀនលៅតាមសកមមភាពលឆលើយតបរបស់នាង។ (រតូវលធវើសកមមភាព
មួយលផសងលទៀត លដើមផីបញ្ញជកព
់ ច
ី លនាលឆលយ
ើ តបលោយលរផលិច ដូចជាយកដដ
លៅចាក់ចលងេេះសិសសណាោបក់ជាលដើម)។

66
កិចចតរងការបររងៀន

របធានបទ៖ ការសង្កេតទម្រក់ម្រដាប់បន្តពូជរបស់រុក្ខជាតមា
ិ ន្ផ្កេ

ថ្ននក់ទី ៧ ៖ កមមវ ិធីសិកាែមី


ជាំពូកទី២ ៖ រុកខជាតិ
មមមរៀនទី២ ៖ រុកខជាតោ
ិ នផាេ

២.របោប់បនថពូជ

រយេៈមពល៖ ៥០ នាទី
វតថុបាំណង្៖

-កាំណ្ត់ពីដផបកលផសងៗរបស់ផាេបានចាស់ោស់តាមរយៈការសលងេតផាេផាធល់។
-កណ្
ាំ តព
់ ទ
ី រមង់របោបប
់ នថពូជរបសរ់ ុកខជាតោ
ិ នផាេបានរតម
ឹ រតូវតាមរយៈការពន
ិ ត
ិ យលមល

សររាងគ
ី បនថពូជរបស់រុកខជាតិ លោយផាធល់។

-លរៀបរាប់ពីវដថដនការបនថពូជរបស់រុកខជាតិោនផាេ បានរតឹមរតូវតាមរយៈការសលងេត។
-ស្សោញ់ និងដែរកា ការ ររុកខជាតិ
សមាារេៈ ៖ ផាេរកខុប ដកវពរងីក ឡាមលកាពុកោត់
លនា
ាំ ថ្ាំ នការបមរង្ៀន

សកមមភាពរគូ ខ្លឹមោរមមមរៀន សកមមភាពសិសស

ជាំហានទ១
ី ៖ រដឌបាលថាបក់

-រតួតពិនិតយថាបក់។ ថ ប់ធាបប់,វ ិន័យនិងអ -សរមួលឥរោបែ


រតួតពិនិតយសណា ិ
-រតួតពិនិតយ អវតថោនសិសស វតថសិសស។ -របធានថាបក់រាយការណ្៍

ជាំហានទ២
ី ៖ រល
ាំ ឹកលមលរៀនចាស់

-កាលពលី ោង
៉ោ មុនលយង
ើ បានលរៀន រួច របលភទរបសស
់ ក
លឹ ោន៖ -ោនសលក
ឹ លពញ សលក
ឹ ដឆក សលក

មកលហយ
ើ អព
ាំ រី បលភទសលក
ឹ ។ សលក
ឹ លពញ សលក
ឹ ដឆក សលក
ឹ គនាលក់ គនាលក់ សលក
ឹ លធមញ....
+ចូរលរៀបរាបព
់ រី បលភទលផសងៗ របស់ សលក
ឹ លធមញ....
សលក
ឹ ?
-ចុេះសលក
ឹ រកខុបជារបលភទសលក
ឹ អវី ? -របលភទសលឹក្លស់

ជាំហានទ៣
ី ៖ លមលរៀនរបចាដាំ ែៃ

ប ា ញដផលរកខុបរួចសួរ៖ លមលរៀនទ២
ី ៖ រុកខជាតោ
ិ នផាេ
លឆលើយតាមការដដលពួកលគគិតលឃើញ
លតដើ ផលលូតោសល់ ចញពដី ផបកណាដន ២. របោបប
់ នថពូជ
គឺលចញពីផាេ។
រុកខជាតិ?

67
ង ើយតផលវ ិវែតពីតផនកណាផ្នផ្ទា? . រ ីរាងគបនតពូជ
រុបច្ងមលើយ ិ សង ើយងលើកជា
ំណួររនលឹឹះ ំណួររនលឹឹះ
ងែើទ្យរមង់របោប់បនតពូជរប ់រក
ុ ខ
ជាែិម្ពនផ្ទាម្ពនលកខណៈដូច្ងមតច្?

មមែិកមម

ងបាលកលំអង ិច្
ទ ម្ព៉ា ែ ស្រ ទប់
. ោក់ង្មឹះតផនកងផសងៗរប ់តាម
. បងាាញទ្យរមង់រប
ូ ផគំុផ្ទា រួច្ឱ្យ ិ ស ការរិែ។
ោក់ង្មឹះតផនកងផសងៗរប ់វា។ រែបក អូតវ

. តច្ករូបផ្ទា ឱ្យ ិ សរិែ ង ើយ + តផនកតដលជា រ ីរាងគបនតពូជញី . ងងាែពីទ្យរមង់ផ្ទា ង ើយ រង រ


ងររកនុង នលឹកកិច្ចការពី៖ ......កញ្ុច ំងក រញី........... ងៅតាមរំនិែរប ់ពួកងរ
+ តផនកណាជា រ ីរាងគបនតពូជញី + តផនកតដលជា រ ីរាងគបនតពូជ + រ ីរាងគបនតពូជញីម្ពន៖
+ តផនកណាជា រ ីរាងគបនតពូជ ង្មល ិច្
ទ ម្ព៉ា ែ អូតវ......
ង្មល ......កញ្ុច ំងក រង្មល........ + រ ីរាងគបនតពូជង្មលម្ពន ងបាលក
លំអង ទ្យងងក រ....
ដំងណើរការ ងងាែ
(លំនាំផ្នការ ងងាែ ដូច្ម្ពន
ងរៀបរាប់ងៅកនុង នលឹកកិច្ចការ)
. ឱ្យ ិ សងធវើការជារកុម (១រកុមោន . ច្ូលតាមរកុម
៤នាក់) . ិ សទ្យទ្យួលយក ម្ពារៈ រួច្ងធវើការ
. តច្ក ម្ពារៈងផសងៗ ងងាែពី៖
. តណនាំពីដំងណើរការ និង + ទ្យរមង់ទ្យូងៅរប ់ផ្ទា
កមមភាពតដលរែូវងធវើដល់ ិ ស។ + ទ្យរមង់រប ់ងក រញី និងងក រ
ង្មល
+ ខ្ននែ់ទ្យទ្យឹងរប ់អូតវ
. រួច្រូរប

68
. លៅសិសស៣នាក់ មកគូរលលើកាថរ លទនផល . លឡើងគូររូបលលើកាថរលខៀនលោយ
លខៀន តាមសនលឹកកិចចការរបស់ពួក ោក់ល្មេះដផបកលផសងៗ របស់សររា

លគ។ ងគបនថពូជនីមួយៗ
. ឱ្យសិសសលរៀបរាប់ពីដផបកលផសងៗ
ដនសររាងគ
ី បនថពូជញី និងសររាងគ
ី បនថ
ពូជល្មល លហើយលធវើការរលរបៀប
លធៀបលៅនិងសមមតិកមមរបស់ពួក . លធវើការលរបៀបលទនផលលៅសមមតិ
លគ។ កមមរបស់ខន។
លួ
. ពនយល់ និងដណ្នាប
ាំ ដនទមដូចជាទី
កញ្ចុ ាំលកសរល្មល កញ្ចុ ាំលកសរញី
តាង
ាំ របស់ទងលកសរល្មល និង
លកសរញី។
អាំរបីយ៉ោុងរោប់

អូវុល

ខាបតទ
់ ទង
ឹ អូដវ
សនបោ
ិ ឌន
. ឱ្យសស
ិ សសរលសរសនបោ
ិ ឌ ន លៅ របោបប
់ នថពូជរបសរ់ ុកខជាត៖ិ . តាមលទនផលដនការសលងេតសស
ិ ស
កបងសនល
ុ ក
ឹ កច
ិ ចការរបសខ
់ ន។
លួ . របោបប
់ នថពូជល្មលោន៖ សរលសរលសចកថស
ី នបោ
ិ ឌ នោកក
់ ង
បុ
. លៅសស
ិ សោបកម
់ កសរលសរ លបាលកលអ
ាំ ង ោនផធកលៅលោយ
ុ សនលក
ឹ កច
ិ ចការរបសខ
់ ន។
លួ
លសចកថីសនបិោឌនលលើកាថរលខៀន។ រោប់លាំអង។ . តាំណាងសិសសោបក់សរលសរ
. ដលលមែចលមលើយសិសស ទងលកសរល្មល សរោប់ភាជប់លៅ សនបិោឌនលលើកាថរលខៀន។
នឹងលបាលកលាំអង។ . ដកលមែចាំលលើយរបស់ខន។
លួ
. របោប់បនថពូជញីោន៖
សធិចោត
៉ោ , បាំពង់អូដវ
អូដវ ោនផធកអូ
ុ វុល
ជាំហានទ៤
ី ៖ ពរងឹងពុទនិ
. លយង
ើ បានសលងេតរួចលហយ
ើ ពី វដថបនថពូជរបសរ់ ុកខជាតិ . សស
ិ សពន
ិ ត
ិ យលមល
ើ រូប រួចពភា
ិ កា
ទរមង់របោបប
់ នថពូជរបសរ់ ុកខជាត។
ិ ជាដដគូ លហយ
ើ បកស្សាយ។
ចូរបែនព
ូ ន
ិ ត
ិ យលមល
ើ រូបរួចលធវកា
ើ របក
ស្សាយពវី ដថបនថពូជរបសវា
់ ។

ជាំហានទ៥
ី ៖ បណា
ថ លាំ ផញើ និងកិចចការផធេះ
.លតើវតថបនថពូជរបស់រុកខជាតិោន វដថបនថពូជរបស់រុកខជាតិោនផាេោន សិសសកត់រតា យកលៅស្សាវរជាវ
ប៉ោុនាមនដាំណាក់កាល? អវីខលេះ? ៣ដាំណាក់កាលគឺ ដាំណាក់ផាេ លៅផធេះ។
បញ្ញជក់? ដាំណាក់បងេកាំលណ្ើត និងអាំរបីយ៉ោុង។

69
របធានបទៈ សរងេរទរមង់របដាប់បនតពូជរបស់រ ុកខជារិម្ភនផ្កេ លទ្យធផល៖ ច្ូររូររូប

. ទ្យរមង់ទ្យូងៅរប ់ងក រង្មល និងទ្យរមង់ទ្យូងៅរប ់ងក រញី


សាំណ្ួរគនលឹេះ៖ លតើទរមង់របោប់បនថពូជរបស់រុកខជាតិោនផាេោនលកខណ្ៈ
ដូចលមថច?
លកខណ្ៈដូចលមថច?
សមមតិកមម ៖ តាមរយៈរូប លតើដផបកណាខលេះជាសររាងគ
ី បនថពូជញី? លហើយដផបកណាខលេះ
ជាសររាងគ
ី បនថពូជល្មល?

. ខ្ននែ់ទ្យទ្យឹងរប ់អូតវ

លាំនាដាំ នការសលងេតផាេ៖
. សោភរៈ ៖ ផាេរតខុប ដកវពរងីកឡាមលកាពុកោត់
ននិោឋន៖ តាមលទ្យធផលតដលអនកបាន ងងាែង ើញ ច្ូរងធវើការ ននិោឋន។
. ដាំលណ្ើរការសលងេត៖ សលងេតលមើលការតលរមៀបរបស់ដផបកនីមួយៗរបស់ផាេ។ រួចបកយក
រតបក និងស្សទប់ផាេរលចញ បនាធប់មកពិនិតយលមើល៖
. លកខណ្ៈរបស់លកសរល្មល៖ លបាលកលាំអង រោប់លាំអង លោយលរបើដកវពរងីក។
. លកខណ្ៈរបសល់ កសរញ៖ី សធច
ិ ោត
៉ោ អូដវ នង
ិ លធវខា
ើ ប តទ
់ ទង
ឹ លលអ
ើ ូដវ
. ពន
ិ ត
ិ យលមល
ើ ពកា
ី រលូតោសរ់ បសអ
់ ូដវ នង
ិ អូវុល លោយលធវខា
ើ ប តទ
់ ទង
ឹ ដផល

70
ប្លងកា
់ ា រខ ៀន្ . ខាបត់ទទឹងរបស់អូដវ

ប្ប្ធាន្បទ៖ សខងេតទប្រង់ប្ប្ដាប់បន្ាព ូជរបស់រក


ុ ជា
ខ តិមាន្ផ្កេ

សាំណ្ួរគនលឹេះ
លតើទរមង់របោប់បនថពូជរបស់រុកខជាតិោនផាេ ោនលកខណ្ៈ
ដូចលមថច?
សមមតិកមម៖
+ សររាងគ
ី បនថពូជញោ
ី ន៖ សធច
ិ ោត
៉ោ អូដវ...
+ សររាងគ
ី បនថពូជល្មលោន៖ លបាលកលាំអង ទងលកសរ....
លាំនាកា
ាំ រសលងេតផាេ៖
សនបិោឌន៖ របោប់បនថពូជរបស់រុកខជាតិ
. សោភរ៖ ផាេរតខុប ដកវពរងីក ឡាមលកាពុកោត់
. របោប់បនថពូជល្មលោន៖
. ដាំលណ្ើរការសលងេត៖
+ លបាលកលាំអងោនផធកលៅលោយ
ុ រោប់លាំអង។
លទនផលៈ ចូរគូររូប
+ ទងលកសរល្មល សរោប់ភាជប់លៅ និងលបាលកលាំអង។
ទរមង់ទូលៅរបស់ល្មល នឹងទរមង់ទូលៅរបស់លកសរញី
. របោប់បនថពូជញីោន៖
+ សធិចោត
៉ោ , បាំពង់អូដវ
+ អូដវ ោនផធកអូ
ុ វុល

71
៤.១. តផនដីវិទា

៤.១.១ វគគរពែះចនេ(១)

៤.១.២ វគគរពែះចនេ(២)

៤.១.៣ ដំរណើររបច ំថ្ថៃរបស់រពែះអាទិរយ

៤.១.៤ វដតទឹក
មុ ខ្វ ិជាជៈ តផនដី វ ិទ្យា ថ្ននក់ ទ្យី៨ ងមងរៀន វរគរពឹះច្នទ (១)
រយៈលពល 2 លោ៉ោង
ជាំពូក3: រពឹះចនទ ផផនដី និង្រពឹះអាទិតយ
- លមលរៀនទី1: ចលនារបស់ដផនដី និងរពេះចនធ
- លមលរៀនទី2: វគគរពេះចនធ (ទាំព័រទី280)
- លមលរៀនទី3: ចនធរោស និងសូរយរោស
- លមលរៀនទី4: ជាំលនារសមុរទ
វតថុបាំណង្ជាំពូក
- រ ាំលឹកពីទីតាង
ាំ និងគនលងរបស់រពេះចនធ និងដផនដី
- ពណ្៌នាពីលទនផលដនរងវិលជុាំ និងរងវិលខាញល់របស់ដផនដី និងរពេះចនធ
- ពនយល់ពីវគគរពេះចនធ
- ពណ្៌នាពី្បច
ាំ នធគតិ និងរលបៀបកាំណ្ត់លពលកបងវគគ
ុ រពេះចនធ
- ពនយល់ពីបាតុភូតចនធរោស និង សូរយរោស
- ពណ្៌នាពីសូរយរោសដដលលកើតលឡើងកបងរបលទកមព
ុ ជា

- ពនយល់ពីទាំនាក់ទាំនងរវាងរពេះចនធនិងជាំលនារសមុរទ
1. វតថុបាំណង្មមមរៀន (មួយមមា៉ាង្)
ចាំលណ្េះដឹង
- ពនយល់ពីវគគរពេះចនធបានចាស់ោស់តាមការពិភាការកុម។
បាំណ្ិន
- បលងេើតគរមូរូបភាពវគគរពេះចនធបានរតឹមរតូវតាមលាំោប់ តាមរយៈសកមមភាពរកុម។
អភិវឌណបុគគលិកលកខណ្ៈ
- យកចិតថទុកោក់សលងេតវគគរពេះចនធ ដសវងយល់ពីទាំនាក់ទាំនងរវាងរពេះចនធ និងដផនដី និងការរស់លៅរបចាាំដែៃ។
2. សមាារ
លសៀវលៅដផនដីវ ិទាថាបក់ទ៨
ី ឯកសារលមលរៀនរបស់គលរោង Stepsam2 និង VVOB
បាល់តូច១២ និងបាល់ធាំ១ រោប់លប៉ោងប៉ោុង រូបភាពវគគរពេះចនធ ផាធង
ាំ លសាបគរមូរូបភាពវគគរពេះចនធ
3. ដាំណឹកនាាំមមមរៀន
កមមភាពររូ ខ្លឹមស្ករ កមមភាព ិ ស
ររូ ួរ៖ តផនករដឋបាលថ្ននក់ ជំហានទ្យី១(២នាទ្យី)
រែួែពិនិែយវ ិន័យ ណា
ត ប់ធានប់ របធានថ្ននក់ង ើងរាយការណ៍

- ងែើអនកធាលប់ ងងាែរូបរាងរពឹះច្នទ ជំហានទ្យី២ ំណួរបំផុ + ធាលប់, រូបរាងរពឹះច្នទតរបរបួល។


តដរឬងទ្យ? ងែើរពឹះច្នទម្ពនរូបរាង ដូច្ (១៥នាទ្យី) ងពលខ្លឹះពនលឺងពញវង់ និង ងពលខ្លឹះពនលឺ
ងមតច្តដរ? បំផុ ំណួរងដើមបីរ ំលឹកនូវរូបរពឹះច្នទ តែមួយច្ំណិែតខ្។
- ូមរូររូបរាងរពឹះច្នទតដលអនកធាលប់ តដលងរធាលប់បានង ើញ។ + រូររូបរាងរពឹះច្នទ ង ើយពនយល់
ង ើញ ង ើយពនយល់ផង!. ឲ្យ ិ សងធវើផលងធៀបរវាង ទ្យំ ំពិែ + រពឹះច្នទទ្យទ្យួលពនលឺពីរពឹះអាទ្យិែយ
រប ់តផនដី រពឹះច្នទ និងច្ម្ពៃយរប ់ ងយើងង ើញវាងពញវង់។
អងគទំងពីរងធៀបងៅនឹង ទ្យំ ំររមូផ្ន + ងយើងង ើញពនលឺរពឹះច្នទតែ១ច្ំណិែ
តផនដីរពឹះច្នទ និងច្ម្ពៃយរប ់វា។ ពីងរោឹះម្ពនអវីបាំងវា។

75
- ងែើអនកដឹងងទ្យថ្ន ទ្យំ ំរពឹះច្នទ និង ងដើមបីឲ្យ ិ សត ង
វ យល់ពីការ +តផនដីធំជាងរពឹះច្នទ៤ដង
តផនដីខ្ុ ោនដូច្ងមតច្? ងងាែងមើលវែថុងៅជិែខ្លួន និងងៅ
+ងបើអងាែ់ផិចែតផនដីង ើ៤
ម ម ងនាឹះ
- ងបើអងាែ់ផិចែតផនដីរបតវង៤ ម ងែើ ឆ្ៃយពីខ្ួនដូ
ល ច្ជារពឹះច្នទជាងដើម។
អងាែ់ផិចែរពឹះច្នទរឺ ១ ម
អងាែ់ផិចែរពឹះច្នទង ើប
ម ៉ាុនាមន?
+ច្ម្ពៃយពីតផនដីងៅរពឹះច្នទ មធយម
- ងែើច្ម្ពៃយពីតផនដីងៅរពឹះច្នទម្ពន
៣៨៤,៤០៣រម
របតវងប៉ាុនាមន?
+អងាែ់ផិចែតផនដីម្ពនរបម្ពណ
- ងែើអងាែ់ផិចែតផនដីម្ពនរបតវង
១២,៧៥៦រម
ប៉ាុនាមន?
+ងបើងធៀបនឹងអងាែ់ផិចែតផនដី ច្ម្ពៃយ
រវាងរពឹះច្នទ-តផនដីរឺងរច្ើនជាង
- ដូច្ងច្នឹះងែើម្ពនភាពខ្ុ ោនដូច្ងមតច្
របម្ពណ៣០ដង។
រវាង ច្ម្ពៃយពីតផនដីងៅរពឹះច្នទ និង
+ច្ម្ពៃយររមូពីតផនដីងៅរពឹះច្នទ រឺ
អងាែ់ផិចែ រប ់តផនដី ?
៤X៣០= ១២០ ម
- ដូច្ងនឹះងបើអងាែ់ផិចែររមូតផន
ដីរបតវង៤ ម ងែើរែូវោក់ច្ម្ពៃយពី
តផនដី -រពឹះច្នទ របតវងប៉ាុនាមន?
ំណួររនលឺឹះ ៖ ង ែុ អី បា
វ នជារពឹះច្នទបូររូ
ត បរាង?
- ួរ ិ សឲ្យបងងាើែ មមែិកមម៖ ពី ជំហានទ្យី៣ ងមងរៀនរបចំផ្ងៃ៖ បងងាើែ មមែិកមម
មូលង ែុតដលនាំឲ្យងយើងង ើញរពឹះ វរគរពឹះច្នទ (៦០នាទ្យី) -ងយើងអាច្ង ើញរពឹះច្នទផ្ទល ់បូររូ
ត ប
ច្នទតរបរបួលរូបរាង។ ងអាយ ិ សរិែរក មមែិកមម ងច្ញពី រាង ងោយស្កររពឹះច្នទ ងធវើច្លនា
ការយល់ដឹងផ្ទទល់ខ្ួន
ល ង្លើយែបនឹង តាមរនលង ិែ
ថ ងលើទ្យីតាំងងផសងៗោន។
បញ្ហ
ា តដលររូោក់ឲ្យ និងងធវើការ - រពឹះច្នទផ្ទល ់បូររូ
ត បរាង ងោយ ស្ករ
ច្ូរពនយល់តាមការយល់ ង ើញរប ់ ពនយល់ពីរំនិែរប ់ខ្ួន។
ល ស្រ ងម្ពលរប ់តផនដីររបពីងលើវា។
អនក។ រំនិែរប ់ ិ សរឺ ំខ្នន់បំផុែរំនិែ - ងោយស្ករទ្យីតាំងរពឹះច្នទ
យល់រច្ ំងផសងៗ តដលអាច្ងកើែ - ងោយស្កររពឹះច្នទរាងមូល
ម្ពនងទឹះ ិ សមិនបានរកនឹកង ើញ - ងោយស្ករ តផនដីរាងមូល
ក៏ងោយ ដូច្ជា រូបភាពច្នទរោ រូប - ងោយស្ករពពកររបងលើ
រពឹះច្នទរឹះ ឬលិច្ជាងដើម - ងោយស្ករ ងយើង ិែ
ថ ងលើមុំខ្ុ ោន
- ងោយស្ករងយើងអាច្ង ើញតែតផនក
តដលឈមនឹងរពឹះអាទ្យិែយ។
ងទ្យ, មិនអាច្ងទ្យ។ វាងៅម្ពនរូបរាង

- ងែើអនកធាលប់ង ើញ រពឹះច្នទបូររូ
ត បរាង ដតដលងៅមុនងពល និងងរកាយងពល
រកា មមែិកមម ងដើមបីងផទៀងផ្ទទែ់នឹង
ងៅងរកាយងពលតដលពពកបាំង រួច្ ពពកបាំង។ ដូច្ងនឹះរពឹះច្នទ បតូររូបរាង
លទ្យធផលងរកាយងធវើ កមមភាព។
ពពករស្កែ់ងច្ញងៅវ ិញតដរឬងទ្យ? មិនតមនងោយស្ករតែពពកបាំងងនាឹះ
បដិង ធងចលនូវអវីតដលមិនោក់ព័នធ
-ងែើអនកច្ង់ម្ពនន័យដូច្ងមតច្ ច្ំងោឹះ ងទ្យ។
នឹង កមមភាព
ោកយ +ងយើង ិែ
ថ ងៅទ្យីតាំងខ្ុ ោនងលើតផន

ថ្ន ងយើង ិែ
ថ ងៅងលើមុំខ្ុ ោន? ដី

លអ ូមរការំនិែងរៀងខ្លួន ង ើយរង់ +ងយើង ិែ


ថ ងៅកតនលងតែមួយតែរពឹះ

ចំងមើលលទ្យធផល ច្នទបូរទ្យី
ត តាំង។

76
ច្ូរងរបើររមូតាងរពឹះច្នទ តផនដី និងរពឹះអាទ្យិែយ ងដើមបីពិនិែយងមើលការតរបរបួលរូបរាងរពឹះច្នទទំងអ ់ោ
! ន (៤០នាទ្យី)
-ជាដំបូងងយើង ងងាែងមើលរូបរាងរពឹះ តាមរយៈររមូអងគទំង៣ ិ សអាច្ - ងងាែ កមមភាព ង ើយរិែដល់
ច្នទ ពីខ្នងងរៅលំ ងយើងនឹងង ើញ ត ង
វ យល់ពីរប
ូ ពិែរប ់រពឹះច្នទតដល តផនកផ្ងៃផ្នរពឹះច្នទ និងតផនដី ជាតផនក
រពឹះច្នទម្ពនរូបរាងយា៉ា ងដូច្ងមតច្ ្លុឹះពនលឺពីរពឹះអាទ្យិែយ និងរូបរាងរពឹះ តដលឈមនឹងរពឹះអាទ្យិែយ និងតផនក
ច្នទតដលងមើលពីតផនដីងៅ ងទឹះបីជា តដលមិនឈមជាងពលយប់។
អនក ងងាែ ិែ
ថ ងៅទ្យីតាំងខ្ុ ោនងលើ - ងងាែ និងរូររូបវរគរពឹះច្នទតាម
តផនដីក៏ងោយ ។ ច្លនារងវិលជុំរប ់រពឹះច្នទជុំវ ិញតផន
ដីផុយនឹ
ទ ងរទ្យនិច្នា ិកា
1 2 3 4

- ូមរែ ប់ងៅកាន់ភពរប ់ងយើង - ែំណាងរកុមបិទ្យរូបងលើកាតងខ្ៀន

វ ិញ តខ្....... តខ្....... តខ្....... តខ្....... ពនយល់ពីវរគរពឹះច្នទ៖ ច្នទងពញវង់ តខ្

- តច្ក ិ សជា៣រកុម ងងាែ និងរូរ ោច្់ តខ្៨ងកើែ និងតខ្៨ងរាច្។


5 6 7 8
រូបររមូរពឹះច្នទោក់តាមងលខ្ងរៀងកនុង - វរគ៤ងផសងងទ្យៀែម្ពនង្មឹះថ្ន តខ្

តារាង រួច្យកងៅកាែ់បិទ្យងលើផ្ទទំង ងដើមរងនាច្ តខ្ច្ុងរងនាច្ តខ្ងដើមងខ្នើែ


តខ្....... តខ្....... តខ្....... តខ្.......
រកោ ធំ។ និងតខ្ច្ុងងខ្នើែ។

- របាប់ែំណាងរកុម រូររូបវរគរពឹះច្នទ - វរគរពឹះច្នទម្ពន៨វរគ រឺតខ្ងពញវង់ តខ្

តាមលំោប់លំងោយ យកមកបងាាញ ងដើមរងនាច្ តខ្៨ងរាច្ តខ្ោច្់ តខ្ងដើម

ងលើកាតងខ្ៀនង ើយពនយល់។ ងខ្នើែ តខ្៨ងកើែ និងតខ្ច្ុងងខ្នើែ

- ងែើវរគ៤ងផសងងទ្យៀែ ម្ពនង្មឹះអវីខ្លឹះ?
- ដូងច្នឹះងែើវរគរពឹះច្នទម្ពនប៉ាុនាមន?
ជំហានទ្យី៤ (២០នាទ្យី)
ដូងច្នឹះ រែ ប់ងៅរក មមែិកមមរប ់ + រូបភាពងនឹះ ច្តមលកខ្ុ ពីរប
ូ វរគ
ងច្ញពីលទ្យធផលផ្នការ ងងាែ
អនក ទំងអ ់ោនវ ិញ ងែើម្ពនរូបភាព រពឹះច្នទ។ វាដូច្ជារូបច្នទរោ តផនក
កមមភាពររមូវរគរពឹះច្នទ ិ សអាច្
ណាតដលដូច្និងម្ពនរូបភាពណា តដលងងឹែជាស្រ ងម្ពលរប ់តផនដី
ងធវើង ច្កតី ននិោឋនរួម ពីមូលង ែុ
តដលខ្ុ ពីអីវតដល អនកងទ្យើបបាន បាំងបិទ្យងលើរពឹះច្នទ។
ពិែតដលងយើងអាច្ងមើលង ើញ រូប
ងងាែងមីៗងនឹះ? + ងមើលពីតផនដីងៅរពឹះច្នទ ហាក់ដូច្
រាងរពឹះច្នទម្ពនការផ្ទល ់បូរត រឺមិន
- លអណា ់, ច្ូរងធវើការពិភាការក ជាបតូររូបរាង។ ការពិែវាមិនបតូររូបរាង
តមនងោយស្កររពឹះច្នទ ផ្ទល ់បូររូ
ត ប
ច្ងមលើយរួម៖ ងែើង ែុអីវបានជាងយើង ងទ្យ ងោយស្ករតែរពឹះច្នទ ិែ ថ ងៅទ្យី
រាងខ្លួនវាងនាឹះងទ្យ។
ង ើញរពឹះច្នទបូររូ
ត បរាង? ងែើរពឹះច្នទ តាំងខ្ុ ៗោន ងធវើឲ្យងមើលង ើញរពឹះ
មា៉ា ងងទ្យៀែជួយឲ្យងរយល់ច្ា ់និង
ពិែជាផ្ទល ់បូររូ
ត បរាងរប ់វាតមនឬ? ច្នទកុងរូ
ន បភាពខ្ុ ោន តដរ។ ងនឹះជា
តលងម្ពនការយល់រច្ ំងលើរប
ូ វរគរពឹះ
វរគរពឹះច្នទ តដលម្ពនច្ំនួន៨វរគ។
ច្នទ និងច្នទរោ ។
កិច្ចការផទឹះ ជំហានទ្យី៥ (៣នាទ្យី)
- ច្មលង ំណួរ និងស្កតប់តាមការ
-បងាាញរូបរបរកែិទ្យិនច្នទរែិ តដល ការផតល់ដំបូនាមនឲ្យ ិ សងច្ឹះងធវើកិច្ច
តណនាំរប ់ររូ។
ម្ពនរូបភាពរពឹះច្នទរឹះតាមយប់នីមួយ ការផទឹះនិងស្កលា ត ង
វ យល់ពី
ឲ្យ ិ ស ងធវើការ ងងាែងមើលរពឹះច្នទ ច្ំណាប់អារមមណ៍រប ់ ិ សងលើងម
និងរិែដល់ រូបរាងរពឹះច្នទងៅមុន ងរៀនរបចំផ្ងៃ ងដើមបីបានដឹងពីអីវតដល
និងងរកាយងពលច្នទរឹះ និងលិច្។ ងរងៅមិនទន់យល់ច្ា ់ និងងលើក
ទ្យឹកច្ិែត ិ សឲ្យងច្ឹះ ័យ
វ ិកាងលើ
- ងែើម្ពនអវីតដលរួរឲ្យចប់អារមមណ៍
ងមងរៀនបនាទប់។
ច្ំងោឹះងមងរៀនងនឹះ?

សូមអបករគូ លោករគូដកតរមូវកិចដច តងការលនេះ តាមលពលលវោជាក់ដសថងដនលោ៉ោងបលរងៀន

77
ឹរ
ល សារបន្ន្ែរ

ងោយស្កររងវិលខ្ន្ល់រប ់តផនដី ងយើងអាច្ងមើលង ើញរពឹះច្នទរឹះងៅទ្យិ ខ្នងងកើែ និងលិច្ងៅវ ិញងៅទ្យិ ខ្នងលិច្


ដូច្រពឹះអាទ្យិែយតដរ។ ងមើលច្ំពីងលើប៉ាូលខ្នងងជើងផ្នតផនដី រពឹះច្នទងធវើច្លនាជុំវ ិញតផនដី ងពលតដលតផនដីកំពុង ងធវើច្លនាជុំ

វ ិញរពឹះអាទ្យិែយតាមទ្យិ ផទុយនឹងរទ្យនិច្នា ិការដូច្ោនតដរ។ វរគរពឹះច្នទតដលងយើងងមើលង ើញពីតផនដីអាស្រ ័យងលើទ្យីតាំងរពឹះ


ច្នទ និងតផនដីងធវើច្លនា ងលើរនលងរប ល ំវ ិញរពឹះអាទ្យិែយ។
់ខ្ួនជុ
វរគផ្នរពឹះច្នទតដលនិយាយពីងខ្នើែនិងរងនាច្ងនាឹះ ងររាប់ចប់តាំងពីផ្ងៃមួយងកើែរ ូែដល់ ៨ងកើែ ងរងៅងដើមងខ្នើែ និង
ចប់ពី៨ងកើែរ ូែដល់ផ្ងៃ១៥ងកើែងរងៅថ្នច្ុងងខ្នើែ។ ច្ំងោឹះរងនាច្ក៏ដូច្ោនតដរ
ងនឹះជារូបច្នទងពញវង់( full moon) តដលងយើងអាច្ងមើលង ើញ ងពញមួយច្ំង ៀងរពឹះច្នទ។ តផនកភលឺ ផ្នរពឹះច្នទឈមមុខ្នឹង
តផនដីងពលយប់ ម្ពនន័យថ្នតផនដី រពឹះអាទ្យិែយ និងរពឹះច្នទ ង ើរទ តែ ិែ
ថ ងលើតខ្ស បនាទែ់តែមួយ តដលម្ពនតផនដីងៅច្ំកណា

ល។ ពនលឺរពឹះច្នទតដលងយើងងមើលង ើញជាពនលឺតដលទ្យទ្យួលបានពីរពឹះអាទ្យិែយប៉ាុងណាណឹះ។
ងនឹះជារូបតខ្ោច្់ (new moon) តដលងយើងមិនអាច្ងមើលង ើញរពឹះច្នទទល់តែងស្កឹះ។ តផនកមួយ ច្ំង ៀងរពឹះច្នទតដល

ទ្យទ្យួលពនលឺពីរពឹះអាទ្យិែយ បានតបរងច្ញពីតផនដីងៅងពលយប់ ម្ពនន័យថ្ន តផនដី រពឹះអាទ្យិែយ និងរពឹះច្នទង ើរទ តែ ិែ


ថ ងៅងលើតខ្ស
បនាទែ់តែមួយដូច្ច្នទងពញវង់តដរ តែរពឹះច្នទ ថ ងៅច្ងនាលឹះរវាងរពឹះអាទ្យិែយនិងតផនដីវ ិញ។
ិែ
ងនឹះជារូបតខ្៨ងកើែ(First Quarter Moon) តដលងយើងអាច្ងមើលង ើញរូបរពឹះច្នទភឺលោក់កណា
ត លខ្នងស្កតំង ើយតផនកោក់
កណា
ត លងទ្យៀែងយើងមិនអាច្ងមើលង ើញង ើយ។ ងៅកំ ុង ងពលងនឹះ ចប់ពីផ្ងៃតខ្ោច្់ ពនលឺចប់ងផតើមងលច្ងច្ញបនតិច្មតងៗ
ងច្ញជាច្ំណិែតខ្ ងរៀងរាល់ផ្ងៃរ ូែមកដល់តខ្៨ងកើែងនឹះពនលឺបានងពញោក់កណា
ត លងពញវង់។ ង ើយពនលឹងនឹះងៅតែបនតរ ីកធំ
ងទ្យៀែរ ូែ ដល់តខ្ងពញវង់។
ងនឹះជាតខ្៨ងរាច្(Third or Last Quarter Moon) តដលងយើងអាច្ងមើលង ើញពនលឺរពឹះច្នទរែឹម មួយច្ំង ៀង។មួយច្ំង ៀង
ខ្នងង្វងម្ពនពនលឺរពឹះច្នទនិងមួយច្ំង ៀងខ្នងស្កតំជាតផនកងងឹែ។ ងៅកំ ុងងពលងនឹះ ចប់ពីតខ្ងពញវង់ ពនលឺច្នទបានបាែ់បនតិច្
មតងៗជាច្ំណិែងរៀងរាល់ផ្ងៃរ ូែដល់ តខ្៨ ងរាច្ ពនលឺបាែ់អ ់ោក់កណា
ត លង ើយពនលឺងនឹះងៅតែបនតបាែ់រ ូែអ ់ពនលឺងៅតខ្
ោច្់។

បនាទប់មកងទ្យៀែ វរគងនឹះងៅតែបនតជារ ូែ។ ងរៅពីវរគធំៗទំង៤ខ្នងងលើ ងៅម្ពនវរគ៤ងទ្យៀែ តដល ិែ


ថ ងៅច្ងនាលឹះវរគ
ទំង៤ងនាឹះ ម្ពនដូច្ជា៖

ងនឹះជាតខ្ងដើមងខ្នើែ (Waxing Crescent) ពនលឺច្នទបានងលច្ងច្ញបនតិច្ មិនងពញដូច្ច្នទងពញ វង់តដលងយើងង ើញពនលឺ


ងពញមួយច្ំង ៀងបំភឺលងោយរពឹះអាទ្យិែយងនាឹះងទ្យ។ តផនកផ្នរពឹះច្នទ កំពុងតែ ម្ពនពនលឺធំង ើងៗ។
ងនឹះជាតខ្ច្ុងងខ្នើែ (Waxing Gibbous) ពនលឺច្នទរ ីកធំជាងោក់កណា
ត ល ប៉ាុតនតងៅមិនទន់ងពញវង់ងៅង ើយងទ្យ។ វានឹង

ងពញវង់រ ូែដល់ផ្ងៃទ្យី១៥ងកើែងពញបូរមី។
ងនឹះជាតខ្ងដើមរងនាច្(Waning Gibbous) ងច្ញពីច្នទងពញវង់ ពនលឺងៅ ល់ងលើ ជាងោក់ កណា
ត លច្នទងពញវង់

ង ើយពនលឺងនឹះនឹងបនតបាែ់បនតិច្មតងៗរ ូែ ល់ោក់កណា
ត លរឺតខ្៨ងរាច្។
ងនឹះជាតខ្ច្ុងរងនាច្(Waning Crescent)ងច្ញពីតខ្៨ងរាច្ ពនលឺ ល់ែិច្ជាង៨ងរាច្ ង ើយពនលឺងៅតែបនតលិច្បាែ់បនតិច្
មតងៗ រ ូែបាែ់អ ់ពនលឺដល់តខ្ោច្់។ ូមចំថ្ន រងនាច្ពនលឹងៅខ្នងស្កតំ ងខ្នើែពនលឺងៅខ្នងលិច្។
ច្លនារពឹះច្នទវ ិលជុំវ ិញខ្លួនឯងរឺរងវិលខ្ន្ល់ និងវ ិលជុំវ ិញតផនដីរឺរងវិលជុំ។ តខ្ច្នទរែិ = វរគរពឹះច្នទមួយជុំ ចប់ពីតខ្
ោច្់រ ូែដល់តខ្ោច្់វ ិញកនុងតខ្បនាទប់។រងវិលខ្ន្ល់មួយជុំ =29.5 ផ្ងៃរងវិលជុំ ជុំវ ិញតផនដី មួយជុំ = 27.3ផ្ងៃ រពឹះច្នទរឹះនិងលិច្
ងរកាយោនតាមលំោប់ ៥០នាទ្យីងរៀងរាល់ផ្ងៃ។

78
សន្លឹក្ក្ិចចការ នមនរៀន្វគ្គប្រ៉ះចន្ា
ចូរលរបើគរមូតាងរពេះចនធ និងរពេះអាទិតយ លដើមផីពិនិតយលមើលការដរប
វរគរពឹះច្នទ របួលរូបរាងរពេះចនធទង
ាំ អស់ោប! (២០នាទ)ី
ងែើអនកបាន ងងាែរូបរាងរពឹះច្នទឬងទ្យ? ងែើរពឹះច្នទម្ពនរូបរាងដូច្ងមតច្?

7
8
6

1
............................................................................................................. 5
.............................................................................................................
2 4
.............................................................................................................
3

សាំណ្ួរគនលឹេះ៖
1 2 3 4
.............................................................................................................
............................................................................................................. តខ្....... តខ្....... តខ្....... តខ្.......

.............................................................................................................
5 6 7 8

តខ្....... តខ្....... តខ្....... តខ្.......


កាំណ្ត់សោគល់៖
សនបិោឌន
.............................................................................................................
.............................................................................................................
.............................................................................................................

79
រលង់កាារន ៀន្ ចូរលរបើគរមូតាងរពេះចនធ និងរពេះអាទិតយ លដើមផីពិនិតយលមើលការដរប
វរគរពឹះច្នទ របួលរូបរាងរពេះចនធទង
ាំ អស់ោប! (២០នាទី)

រូបភាពរប ់ ិ ស
7
8
6

- រងនាច្
- តខ្ងពញវង់ ៧-៨ ផ្ងៃ អាច្ជារងនាច្ឬ 1 5
- ពនលឺែូច្
-ង ើញពនលឺ ងរកាយតខ្ ជាងខ្នើែ ងរោឹះ
ច្នទងពញ ងពញវង់ ង ើញពនលឺខ្លឹះអែ់
ង ើញខ្លឹះ 2 4

សាំណ្ួរគនលឹេះ

លហតុអវីបានជារពេះចនធបររូ
ថូ បរាង?
2 3 4

តខ្....... តខ្....... តខ្.......


សមមតិកមម
 ទីតាង
ាំ រពេះចនធ 5 6 7 8

 រពេះចនធរាងមូល
 ដផនដីរាងមូល តខ្....... តខ្....... តខ្....... តខ្.......

 ពពកបាង
ាំ រពេះចនធ សនបិោឌន
 លយើងសទិតលៅលលើមុាំខុសោប
 លយើងអាចលមើលលឃើញដតដផបកដដលឈមនឹងរពេះអាទិតយ លមើលពីដផនដីលៅរពេះចនធ ហាក់ដូចបថររូ
ូ បរាង។ ជាការពិតវាមិនបថររូ
ូ បរាង
 វគគរពេះចនធផាលស់បរលោយសារដតស្សលោលដផនដី
ថូ លទ លោយសារដតរពេះចនធសទិតលៅទីតាង
ាំ ខុសៗោបលធវើឲ្យលមើលលយើងលឃើងរពេះចនធកង
បុ
រូបភាពខុសោបដដរ។ រូបភាពខុសៗោបលនេះជាវគគរពេះចនធដដលោនចាំនួន៨វគគ។

80
វរគរពឹះច្នទ (ច្នទរឹះនិងលិច្) (២)

រយៈងពល 1ងម្ព៉ា ង (៥០នាទ្យី)


4. វែថុបំណង
- ច្ំ ងណឹះដឹ ង
+ បកស្រស្កយពីងពលងវលាច្នទរឹះនិងលិច្ងៅវរគងដើមងខ្នើែបានច្ា ់លា ់តាមរយៈការពិភាការកុម
- បំ ណិន
+ បងងាើែររមូច្នទរឹះ និងលិច្ងៅវរគងដើមងខ្នើែ កនុងថ្ននក់បាន មស្រ ប តាមរយៈ កមមភាពរកុម
- អភិ វឌ្ឍបុ រគលិកលកខណៈ
+ យកច្ិែតទ្យុកោក់ ងងាែច្នទរឹះនិងលិច្ និងទ្យំនាក់ទ្យំនងរវាងរពឹះច្នទ និងតផនដីកុងជី
ន វភាពរ ់ងៅ

5. ម្ពារ
រកោ ផ្ទទំងធំ រូបភាពវរគរពឹះច្នទ របរកែិ ទ្យិនច្នទរែិ រូបភាពផ្ងៃរឹះនិ ងលិច្

6. ដំងណើរការបងរងៀន
កមមភាពររូ ខ្លឹមស្ករ កមមភាព ិ ស

ររូ ួរ ជំ ហានទ្យី1 (2 MN)

រែួែពិ និែយវ ិន័ យ បទ្យបញ្ហ


ជ ផ្ផទកុង
ន របធានថ្ននក់ ង ើងរាយការណ៍

- ឱ្យ ិ សរូ រវរគរពឹះច្នទ ជំ ហានទ្យី2បំ ផុ ំណួរ + រូ ររូបវរគរពឹះច្នទ ង ើយពនយល់

រ ំលឹកងមងរៀន (10MN) អំ ពីរប


ូ រាងនិ ងវរគរពឹះច្នទ ផ្ងៃនិងយប់

ផ្នរពឹះច្នទដូច្តផនដី តដរ ដូ ងច្នឹះោក់


បំ ផុ ំណួរនិ ងរ ំលឹកងមងរៀនមុ ន
កណា
ត លរពឹះច្នទងងឹ ែនិ ងោក់ ក

9 3 ងដើ មបីឱ្យងរនឹ កង ើញនូ វ រូបភាពរពឹះ


ណា
ត លងទ្យៀែភលឺ។ ប៉ាុ តនតងយើងងមើ ល
ច្នទ ងពលងរ ងងាែពី លំ និ ង
3
a 9
ង ើញរពឹះច្នទ ផ្ទល ់បូររូ
ត បរាង
ងងាែពី តផនដី ងមើ ល រូបរាងរពឹះច្នទ
a តាមវរគនីមួយៗ
ងោយស្កររពឹះច្នទងធវើច្លនាតាម

p
ូ ភាពតផនដី ៈផ្ងៃ យប់p
រនលងរប ់ខ្ួននាំ
ល ឱ្យវា ិែ
ថ កនុងទ្យី តាំង
-ពនយល់ពីរប
m m ងផសងោន

-ងែើ រពឹះអាទ្យិ ែយរឹះ និ ងលិច្m m + ជាធមមតា រពឹះអាទ្យិ ែយរឹះ ងៅងម្ព៉ា ង


ងៅងពល
6am និ ងលិច្ងៅងម្ព៉ា ង6pm.
ណា?
12 M
-ពនយល់ពីងពលរពឹះច្នទចប់ រឹះ រឹះ 6pm 6am
ខ្ា ់ និ ងលិច្ងៅវ ិញ!
ងជើងងម ខ្នងងកើែ ងជើងងម ខ្នងលិច្
រពេះចនធ រេះ រេះខពស់ លិច
តខ្ងពញវង់រឹះងរកាយ តខ្ងពញវង់លិច្ងៅងពល
៦ោៃច ១២រពឹក ៦រពឹក
ងពលរពឹះអាទ្យិែយលិច្ភាលម រពឹះអាទ្យិែយរឹះនារពឹកងមី

81
ំណួររនលឺឹះ៖ ងែើ ងពលណាតដល
ងយើងអាច្ ងងាែវរគងដើ មងខ្នើែបាន ងែើ ងពលងនាឹះវា ិែ
ថ ងៅទ្យី តាំងកតនលងណា?
ច្ា ់?
- ឱ្យ ិ សបងងាើែ មមែិកមម៖ ជំហានទ្យី3 ងមងរៀនរបចំផ្ងៃ 25min បងងាើែ មមែិកមម
វរគរពឹះច្នទ (រពឹះច្នទរឹះនិងលិច្) + ងយើងង ើញតខ្ងដើ មងខ្នើែ
-ងពលណាអនកង ើញតខ្ងដើ មងខ្នើែ បាន មមែិកមមពី ិ សង ើយ ងៅងម្ព៉ា ង7pm ។
បានច្ា ់? ិ សបកស្រស្កយ មមែិកមមរប ់
ខ្លួន។
- ងពលងនាឹះ វាម្ពនរូបរាងដូ ច្ + វាម្ពនរូបរាង
រកា មមែិកមម ទ្យុ កងរបៀបងធៀបនិ ង
ងមតច្?
លទ្យធផលបដិ ង ធងចលនូ វច្ងមលើយ
រូបរាងតបបងនឹះ ? តបបងនឹះ
តដលមិ នោក់ ព័នធនិង កមមភាព។
? ?

ឬតបបងនឹះ ?

- ងែើ ងពលណាងយើងអាច្ង ើញរូប


+ វាម្ពនរូបរាងតបបងនឹះ
រាងវាតបបងនឹះ ?
ងៅមុ នងពលរពឹះអាទ្យិ ែយលិច្។

- ងែើ ងពលណាងយើងអាច្ង ើញរូប


+ វាម្ពនរូបរាង ងៅ
រាងវាតបបងនឹះ ?
ងរកាយងពលរពឹះអាទ្យិ ែយលិច្។

- ងែើ ងយើងធាលប់ ង ើញ រពឹះច្នទ + ងយើងមិ នអាច្ងមើ លង ើញ


ច្ា ់ងពលផ្ងៃតដរឬងទ្យ? រពឹះ ច្នទច្ា ់ងពលផ្ងៃង ើយ
ងោយស្ករពនលឺរពឹះអាទ្យិ ែយជឹះ
រកាច្ងមលើយរប ់អនក ងយើងនិ ងពិ និ មកខ្នលំង។
ែយង ើងវ ិញងរកាយងពលបានលទ្យធ
ផល

ច្ូ រងរបើ រំរច្


ូ នទ តផនដីនិ ងរពឹះអាទ្យិ ែយ ងដើ មបីពិនិែយងមើ លងពលរពឹះច្នទរឹះ និ ងលិច្ រពមជាមួ យរូបរាងរប ់វា
ផង! (15 mn)
- តច្ក ិ សជា៣រកុម តច្ក ងធវើ កមមភាពបំ ងពញកនុងតារាង
រកោ ធំ និ ងឱ្យ ិ សតាមរកុម និ ងពនយល់លទ្យធផលរកុម។
បំ ងពញកនុងតារាង និ ង នលឺកកិ ច្ច
+ ងពលងវលារពឹះច្នទរឹះនិ ងលិច្
ការ
អាស្រ ័យងលើទ្យីតាំងផ្នវរគរពឹះ
ពនយល់ពីលទ្យធផលរប ់អនកតាម
ច្នទនីមួយៗ។ រពឹះច្នទងរបើ ងពល
រកុម ង ើយងរបៀបងធៀបជាមួ យ
12 ងម្ព៉ា ងចប់ ពីរឹះរ ូែដល់
រកុមងផសងងែើ លទ្យធផលរកុមអនករែូវ
លិច្ង ើយងៅោក់ កណា
ត ល
ឬរកុមដ៍ ផ្ទ្យរែូវ?
ងពលងនឹះ

82
ជាងពលតដលច្នទរឹះខ្ា ់ងលើ
ងម . ឧទ រណ៍ តខ្ងដើ មងខ្នើែ
រពឹះច្នទរឹះងៅងម្ព៉ា ង9am រឹះខ្ា ់
បំ ផុែងៅងម្ព៉ា ង3pm និ ងលិច្
ងៅងម្ព៉ា ង9pm.
ងពលងដើ មងខ្នើែងយើងអាច្ងមើ ល
ង ើញរពឹះច្នទតែមួ យច្ំ ណិែ
ែូ ច្ ងពលវារឹះតផនកផ្នពនលឺនិង
ិែ
ថ ងៅខ្នងងលើ ងរោឹះ
ពនលឺរពឹះអាទ្យិ ែយងៅរឹះងៅង ើយ
ងពលងនាឹះតដរងយើងមិ នអាច្
ង ើញរពឹះច្នទបានច្ា ់ងទ្យ។
ងពលច្នទរឹះខ្ា ់ រប
ូ រាងវារឺ
ប៉ាុ តនតងយើងងៅ មិ នអាច្ង ើញវា
ច្ា ់ងនាឹះង ើយ រ ូែងៅ
ភារខ្នងលិច្ រពឹះអាទ្យិ ែយលិច្
បាែ់ ងយើងអាច្ង ើញរពឹះច្នទ
កាន់ តែច្ា ់រ ូែវាលិច្

- ូមពិ ភាកាោនកនុងរកុម ងលើ ជំ ហានទ្យី4:ពរងឹ ងពុ ទ្យី ធ (10mn) + ននិោឋន៖
ំណួររនលឹឹះ ង ើយងធវើង ច្កតី ងយើងអាច្ងមើ លង ើញរពឹះច្នទ
ពរងឹ ងនូ វអវីតដលងរងទ្យើ បតែបាន
ននិោឋន។ ងៅវរគងដើ មងខ្នើែច្ា ់ ងៅ
ងរៀន ង ើយនិ ងងធវើឱ្យ ិ សកាន់ តែ
ងរកាយងពលរពឹះអាទ្យិ ែយលិច្
យល់ច្ា ់ងមងរៀន និ ងបំ បាែ់ ភាព
ភាលម ងៅងលើងម ខ្នងលិច្។
ភ័ នតរច្ ំ កាលពី មុនដូ ច្ជាងពល
+ ពិ និែយ មមែិកមមង ើងវ ិញ
ច្នទរឹះ រឹះខ្ា ់ និ ងលិច្ងៅវ ិញរឺ មិន
ពិ និែយ មមែិកមមរប ់អនក + លទ្យធផល និ ងទ្យិ ននន័យងច្ញពី
ដូ ច្ោននឹ ងងពលរឹះ និ ងលិច្ ផ្នរពឹះ
បងាាញរបរកែិ ទ្យិនច្នទរែិ ង ើយ របរកែិ ទ្យិនច្នទរែិ ម្ពនការខ្ុ
អាទ្យិ ែយង ើយ។ ជាងងនឹះងទ្យៀែ ច្នទ
ឱ្យ ិ សងធវើការងរបៀបងធៀបងមើ ល ោនែិ ច្ែួ ច្ ងរោឹះ កមមភាព(
រឹះ រឹះនិ ងលិច្ជួ នកាលមិ ន ិែ
ថ ងៅ
ងែើ លទ្យធផលរប ់ងរ រែូវតាម ម្ពារៈពិ ងស្កធ) រប ់ងយើង និ ង
កនុងផ្ងៃតែមួ យងនាឹះងទ្យ។
ងពលងវលាកនុងរបរកែិ ទ្យិនតដរ ងពលងវលាតដល ងងាែរយៈ

ឬងទ្យ? ងពលខ្លី ដូ ច្ងនឹះងយើងមិ នអាច្


បងាាញពី ងពលងវលារែឹ មរែូវ
ពិ ែ របាកដដូ ច្ទ្យិ ននន័យងច្ញពី
របរកែិ ទ្យិនង ើយ។
+ ងពលរពឹះច្នទរឹះនិ ងលិច្

83
កិ ច្ចការផទឹះ ជំ ហានទ្យី 5 (3 mn)
- ងែើ រពឹះច្នទរឹះនិ ងលិច្ងៅងពល ររូផតល់ដំបូនាមនដល់ ិ ស ោក់ - កែ់ រតា ស្កតប់ និ ងងធវើតាមការ
ណា កនុងវរគងផសងៗដ៏ ផ្ទ្យងទ្យៀែ? កិ ច្ចការផទឹះឱ្យ ិ ស និ ងងលើកទ្យឹ ក តណនាំរប ់ររូ។
ច្ិ ែត ិ សឱ្យងច្ឹះ ័យ
វ ិកា និ ង
ងច្ឹះ ងងាែងលើបាែុ ភូែវ ិទ្យា
ស្កស្រ តត ដលងកើ ែម្ពនង ើយងៅ
ជុំ វ ិញខ្លួន។

84
របែិទ្យិនច្នទរែិ តខ្ែុលាឆ្នំ 2011
Sunday Monday Tuesday Wednesday Thursday Friday Saturday
25 28
26 29 30 1
27
NEW
18:10
Sun Rise: 05:49 Sun Rise: 05:49
Sun Rise: 05:49 Sun Rise: 05:49 Sun Rise: 05:49 Sun Rise: 05:49
Set : 17:55 Sun Rise: 05:49 Set : 17:53
Set : 17:54 ិែ
ថ ងៅជិែតផនដី Set : 17:52 Set : 17:51 Set : 17:51
Set : 17:53
Moon Rise: 03:21 Moon Rise: 05:18 Moon Rise: 06:18
Moon Rise: 04:19 Moon Rise: 07:20 Moon Rise: 08:23 Moon Rise: 09:26
Set : 16:01 Set : 17:37 Set : 18:27
Set : 16:49 Set : 19:20 Set : 20:16 Set : 21:14
ច្នទលិច្ Moon Perigee
2 3 4 5 6 7 8
1stQ
10:17

Sun Rise: 05:49 Sun Rise: 05:49 Sun Rise: 05:49 Sun Rise: 05:49 Sun Rise: 05:49 Sun Rise: 05:49 Sun Rise: 05:49
Set : 17:50 Set : 17:49 Set : 17:49 Set : 17:48 Set : 17:47 Set : 17:47 Set : 17:46

Moon Rise: 10:28 Moon Rise: 11:26 Moon Rise: 12:19 Moon Set : 00:07 Moon Set : 01:00 Moon Set : 01:51 Moon Set : 02:39
Set : 22:12 Set : 23:10 Rise: 13:08 Rise: 13:53 Rise: 14:34 Rise: 15:13
12
9 10 11 13 14 15
FULL
09:08

Sun Rise: 05:50


Sun Rise: 05:50 Sun Rise: 05:50 Sun Rise: 05:50 Sun Rise: 05:50 Sun Rise: 05:50 Sun Rise: 05:50
Set : 17:44
Set : 17:45 Set : 17:45 Set : 17:44 ិែ
ថ ងៅឆ្ៃយពីតផនដី Set : 17:43 Set : 17:42 Set : 17:42
Moon Set : 05:44
Moon Set : 03:26 Moon Set : 04:12 Moon Set : 04:58 Moon Set : 06:31 Moon Set : 07:19 Moon Set : 08:09
Rise: 17:44
Rise: 15:51 Rise: 16:28 Rise: 17:05 Rise: 18:24 Rise: 19:07 Rise: 19:52
Moon Apogee
16 17 18 19 20 21 22
3rdQ
10:32

Sun Rise: 05:50 Sun Rise: 05:50 Sun Rise: 05:50 Sun Rise: 05:51 Sun Rise: 05:51 Sun Rise: 05:51 Sun Rise: 05:51
Set : 17:41 Set : 17:41 Set : 17:40 Set : 17:40 Set : 17:39 Set : 17:39 Set : 17:38

Moon Set : 08:59 Moon Set : 09:50 Moon Set : 10:40 Moon Set : 11:29 Moon Set : 12:18 Moon Rise: 00:12 Moon Rise: 01:07
Rise: 20:40 Rise: 21:31 Rise: 22:24 Rise: 23:17 Set : 13:04 Set : 13:50
26
23 24 25 27 28 29
NEW
02:57
Sun Rise: 05:52
Sun Rise: 05:51 Sun Rise: 05:51 Sun Rise: 05:52 Sun Rise: 05:52 Sun Rise: 05:52 Sun Rise: 05:52
Set : 17:36
Set : 17:38 Set : 17:37 Set : 17:37 Set : 17:36 Set : 17:36 Set : 17:35
Moon Rise: 04:58
Moon Rise: 02:02 Moon Rise: 02:59 Moon Rise: 03:58 Moon Rise: 06:01 Moon Rise: 07:06 Moon Rise: 08:10
Set : 17:03
Set : 14:36 Set : 15:23 Set : 16:12 Set : 17:58 Set : 18:57 Set : 19:57
Moon Perigee
30 31 1 2 3 4 5
1stQ
23:39

Sun Rise: 05:53 Sun Rise: 05:53 Sun Rise: 05:53 Sun Rise: 05:53 Sun Rise: 05:54 Sun Rise: 05:54 Sun Rise: 05:54
Set : 17:35 Set : 17:34 Set : 17:34 Set : 17:34 Set : 17:34 Set : 17:33 Set : 17:33

Moon Rise: 09:12 Moon Rise: 10:10 Moon Rise: 11:02 Moon Rise: 11:50 Moon Rise: 12:33 Moon Set : 00:36 Moon Set : 01:23
Set : 20:58 Set : 21:57 Set : 22:53 Set : 23:46 Rise: 13:13 Rise: 13:51
All times are displayed in Standard Time. If Daylight Savings time is in effect at the designated location, add one
hour to all event times.

85
ការបរ ិចឆទ៖................. សនលឹកកិចចការ
ល្មេះ .................. មមមរៀនរពឹះចនទ (ចនទរឹះ នង្
ិ លច
ិ ) ច្ូ រងរបើ រំររូ ពឹះច្នទ តផនដី និ ងរពឹះអាទ្យិ ែយ ងដើ មបីពិនិែយងមើ ល ងពលងវលា

រពឹះច្នទរៈនិ ងលិច្ រពមជាមួ យរូបរាងរប ់វាផង! (15 minutes)

9 3
a3 9
រពេះចនធ រេះ រេះខពស់ លិច ងជើងងម ខ្នងងកើែ ងជើងងម ខ្នងលិច្
a
p
m p
m
សនបោ
ិ ឌ ន៖

...........................................................................................................................................................
m m ...........................................................................................................................................................
.........................................................................................................................................................
ងជើងងម ខ្នងងកើែ ងជើងងម ខ្នងលិច្

សាំណ្ួរគនលេះឹ ៖
កាំណ្ត់សោគល់៖
.....................................................................................................................................................
...................................................................................................................................................? ...........................................................................................................................................................
...........................................................................................................................................................
សមមតិកមម៖ កិចកា
ច រផធេះ លតើចនធរេះ រេះខពស់ និងលិចលៅវគគលផសងៗលទៀត លោ៉ោងប៉ោុនាមន? ោនរូបរាងដូចលមថច?
.....................................................................................................................................................
...................................................................................................................................................

86
បលង់កាថរលខៀន(វគគរពេះចនធ(ចនធរេះ និងលិច)

ចូរលរបើគាំរូរពេះចនធ ដផនដី និងរពេះអាទិតយ លដើមផីពិនិតយលមើលលពលលវោ

9 3 រពេះចនធរេះ និងលិច រពមជាមួយរូបរាងរបស់វាផង! (១៥ minutes)

a3 9
3 PM
រពេះចនធ
6PM
រេះ រេះខពស់
12M
លិច
6AM
a
p
m
12 M
p
m
9AM 9 PM

ងជើងងម ខ្នងងកើែ ងជើងងម ខ្នងលិច្


6 PM
m m 6 AM ងយើងមិនអាច្ងមើលង
ងរោឹះពនលឺរពឹះអាទ្យិែយ
ើញច្ា ់ ងយើងអាច្ ង ើញច្ា ់
ងរោឹះរពឹះអាទ្យិែយលិច្
ងជើងងម ខ្នងងកើែ ងជើងងម ខ្នងលិច្
បាែ់
តខ្ងពញវង់រឹះងរកាយ តខ្ងពញវង់លិច្ងៅងពល

សាំណ្ួរគនលេះឹ ងពលរពឹះអាទ្យិែយលិច្ភាលម រពឹះអាទ្យិែយរឹះនារពឹកងមី

សាំណួរគនលឹះឹ ៖ មតើមពលណ្ផដលមយើង្អាចសមង្កតវគគមដើមមខ្នត
ើ បានចាស់? មតើមពលមនាឹះវាសថិតមៅទី
ាាំង្កផនលង្ណ្?

លសចកថីសនបោ
ិ ឌ ន៖
សមមតិកមម៖
លយើងអាចលមើលលឃើញរពេះចនធលៅវគគលដើមលខបើតចាស់លៅលរកាយលពលរពេះអា
 លយើងលឃើញដខលដើមលខបត
ើ ចាស់លៅលោ៉ោង 7pm ។ វាោនរាង តិទយលិចភាលម លៅលលើលមឃខាងលិច។
 វាោនរាង ដបបលនេះលៅមុខលពលរពេះអាទិតយលិច។
កិចកា
ច រផធេះ
 វាោនរាង លៅលរកាយលពលរពេះអាទិតយលិច។
លតើចនធរេះ រេះខពស់ និងលិចលៅវគគលផសងៗលទៀត លោ៉ោងប៉ោុនាមន? ោនរូបរាងដូចលមថច?
 លយើងមិនអាចលមើលលឃើញ រពេះចនធចាស់លពលដែៃលឡើយ លរ េះពនលរឺ ពេះអាទិតយ

87
dMeNIrcrRbcaMéf¶rbs;RBHGaTitü
ry³eBl ! em:ag ¬%0naTI¦
!>vtßbú MNg
-segát nigkt;Rta)anBIclnaRBHGaTitü
-Bnül;)anBImlU ehtuénkarERbRbYlRsemalRBHGaTitü.
@>smÖar eQIcak;eFµj sáút bnÞat; Rkdas; A4 nigKRmbRbGb;Rkdas; A4.
#>skmµPaBbeRgon
skmµPaBRKU xøwmsar skmµPaBsisS
CMhanTI ! ¬@ naTI¦
CMhanTI @ ¬!0 naTI¦
-etIeyIgGacemIleXIjRsemalenA -emIleXIjenAeBléf¶
eBlNa? -RsemalekIteLIgedaymanvtßúGVI
-etIRsemalGacekIteLIg)anya:g mYymk)aMgBnø.W
dUcemþc? -BnøWeFVdI MeNIrRtg;
-etIBnøWeFVIdMeNIrya:gdUcemþc? -RBHGaTitü CaRbPBBnøWEdlmanBnøW -RBHGaTitü
-etIRbPBBnøWGVIEdlmanfamBlxøaMg nig famBlxøaMgCageK -RsemaleBlRBwkenAxaglic
CageK? eBll¶ac enAxagekIt eBlRBwk
-etIRsemalEdl)anBIBnøWRBHGaTitü RBlwmeXIjRsemalEvg eBll¶ac
manlkçN³dUcemþc? eXIjRsemalxøI
CMhanTI # ¬@%naTI¦
-etIRBHGaTitü rH niglic enATisxag -rHenATisxagekIt niglicenATisxag
Na? lic
sMNYrKnøwH etIRBHGaTitücrenAtampøÚvNaénEsVremX enAeBlrH nigeBllic?
-etIRBHGaTitüeFVIdMeNIreTANa -bnÞab;BIéf¶rHvaeFVIdMeNIrx<s;eTA
bnÞab;BIrH nigeBléf¶Rtg;? elIemX
-etIeyIgmanmeFüa)ayGVIedIm,Idwg -bnÞab;BIéf¶Rtg;vaeFVIdMeNIrcuHmk
BIclnarbs;RBHGaTitü? Tisxaglic
-[sisSsakl,gedAcMNuc -sisSeqøyI tamkaryl;eXIjrbs;
RsemalRBHGaTitü enAéf¶enH BYkeK
nigem:agenH. N -sisS@ bJu#nak;sakl,gedAcMNuc
E W
S

88
-EcksisSCabIRkum¬bInak;kgñú mYy N

Rkum¦ W
9am
12am E

cUrsegátkt;RtadMeNIrRbcaMéf¶énRBHGaTitü.
EcksmÖar[sisStamRkum enAeBl -Bnül;sisSBIreboberobcMsmÖar -sisSerobcMsmÖarehIyecjeRkA
ecjeRkAfñak;RtUvENnaMsisS[³ kareRbIR)as;RtIvis½yedIm,IrkTisxag fñak;. erIsTIkEnøgrabesµIdak;]bkrN_
-rkkEnøgl¥ rabesµeI dIm,Idak;smÖar eCIg nigrebobkt;Rta bJuedAcMNucén nigeRbIRtIvis½yedIm,IkMNt;Tisxag
-eRbIR)as;RtIvsi ½yedIm,IrkTisxageCIg Rsemal. eCIg.
-kt;RtacMNucTImYy % N
-kt;RtaRsemaltamkarENnaMrbs;RKU
bJu!0naTIeRkaykt;RtacMNucTIBIr. w
N
E w
-[RkumsisSbgðajlT§plrbs;BkY eK S E
S

-bgðajBIlT§plEdlBYkeK)ankt;Rta
CMhanTI $ ¬!0 naTI¦
-etIeyIgGacniyay)anya:gdUcemþc -RBHGaTitürH licnigrHx<s;enAelI -enAeBlRBwkRsemalenATisxaglic
GMBIdMeNIr RbcaMéf¶rbs;RBHGaTitü emX TaMgGs;enHGacniyay)anfa kan;EtxøIeTA².
nigrUbragRsemal? RBHGaTitü eFVIdMeNIrCMuvijEpndI. enAeBll¶acRsemalenATisxagekIt
EtkarBitEpndIvilCMuvijxøÜnÉg kan;EtEvgeTA².
-etIRBHGaTitüeFVIdMeNIrbJuEpndIvil EdlCaehtuemIleTAeXIjhak;dUcCa -RBHGaTitürHBITsi xagekIt licenA
CMuvijxøÜnÉg? RBHGaTitürH niglic. Tisxaglic nigrHx<s;enAelIemX
enAeBléf¶Rtg;.
CMhanTI % ¬# naTI¦
-[sisSrkSasmÖarehIyENnaMBYk -sisSeFVkI ic©karpÞHtamkarENnaMrbs;
eK[kt;RtaRsemalenAéf¶Rtg; RKU.
nigema:gmYyresol.

89
snøkw kic©kar
eQµaH>>>>>>>>>>>>>>>>>>>>>>>>>>>>>>>>>>>>>>>>>>>>>>>>

!> etIRsemalGacekIteLIg)anya:gdUcemþc?
>>>>>>>>>>>>>>>>>>>>>>>>>>>>>>>>>>>>>>>>>>>>>>>>>>>>>>>>>>>>>>>>>>>>>>>>>>>>>>>>>>>>>>>>>>>>>>>>>>>>>>>>>>>>>>>>
>>>>>>>>>>>>>>>>>>>>>>>>>>>>>>>>>>>>>>>>>>>>>>>>>>>>>>>>>>>>>>>>>>>>>>>>>>>>>>>>>>>>>>>>>>>>>>>>>>>>>>>>>>>>>>>>
>>>>>>>>>>>>>>>>>>>>>>>>>>>>>>>>>>>>>>>>>>>>>>>>>>>>>>>>>>>>>>>>>>>>>>>>>>>>>>>>>>>>>>>>>>>>>>>>>>>>>>>>>>>>>>>>
>>>>>>>>>>>>>>>>>>>>>>>>>>>>>>>>>>>>>>>>>>>>>>>>>>>>>>>>>>>>>>>>>>>>>>>>>>>>>>>>>>>>>>>>>>>>>>>>>>>>>>>>>>>>>>>>
>>>>>>>>>>>>>>>>>>>>>>>>>>>>>>>>>>>>>>>>>>>>>>.
@> etIRBHGaTitüeFVIdMeNIreTANabnÞab;BrI H nigbnÞab;BIeBléf¶Rtg;?
>>>>>>>>>>>>>>>>>>>>>>>>>>>>>>>>>>>>>>>>>>>>>>>>>>>>>>>>>>>>>>>>>>>>>>>>>>>>>>>>>>>>>>>>>>>>>>>>>>>>>>>>>>>>>>>>>>>>>>>>>>>>>>>>>>>>>>>>>>>>>>>>>>>>>>>>>>>>>>>>>>>>>>>>>>>>>>>>>>
>>>>>>>>>>>>>>>>>>>>>>>>>>>>>>>>>>>>>>>>>>>>>>>>>>>>>>>>>>>>>>>>>>>>>>>>>>>>>>>>>>>>>>>>>>>>>>>>>>>>>>>>>>>>>>>>>>>>>>>>>>>>>>>>>>>>>>>>>>>>>>>>>>>>>>>>>>>>>>>>>>>>>>>>>>>>>>>>>>
>>>>>>>>>>>>>>>>>>>>>>>>>>>>>>>>>>>>>>>>>.
#> cUrsegát kt;RtadMeNIrRbcaMéf¶énRBHGaTitü.
-etIeyIgGacniyay)anya:gdUcemþcGMBdI MeNIr RbcaMéf¶rbs;RBHGaTitü nigrUbragRsemal?
>>>>>>>>>>>>>>>>>>>>>>>>>>>>>>>>>>>>>>>>>>>>>>>>>>>>>>>>>>>>>>>>>>>>>>>>>>>>>>>>>>>>>>>>>>>>>>>>>>>>>>>>>>>>>>>>>>>>>>>>>>>>>>>>> N

>>>>>>>>>>>>>>>>>>>>>>>>>>>>>>>>>>>>>>>>>>>>>>>>>>>>>>>>>>>>>>>>>>>>>>>>>>>>>>>>>>>>>>>>>>>>>>>>>>>>>>>>>>>>>>>>>>>>>>>>>>>>>>>>> w
E
>>>>>>>>>>>>>>>>>>>>>>>>>>>>>>>>>>>>>>>>>>>>>>>>>>>>>>>>>>>>>>>>>>>>>>>>>>>>>>>>>>>>>>>>>>>>>>>>>>>>>>>>>>>>>>>>>>>>>>>>>>>>>>>>> S

>>>>>>>>>>>>>>>>>>>>>>>>>>>>>>>>>>>>>>>>>>>>>>>>>>>>>>>>>>>>>>>>>>>>>>>>>>>>>>>>>>>>>>>>>>>>>>>>>>>>>>>>>>>>>>>>>>>>>>>>>>>>>>>.
$> etIRBHGaTitüeFVIdMeNIr bJuEpndIvilCMuvijxønÜ Ég?
>>>>>>>>>>>>>>>>>>>>>>>>>>>>>>>>>>>>>>>>>>>>>>>>>>>>>>>>>>>>>>>>>>>>>>>>>>>>>>>>>>>>>>>>>>>>>>>>>>>>>>>>>>>>>>>>>>>>>>>>>>>>>>>>>>>>>>>>>>>>>>>>>>>>>>>>>>>>>>>>>>>>>>>>>>>>>>>>>>
>>>>>>>>>>>>>>>>>>>>>>>>>>>>>>>>>>>>>>>>>>>>>>>>>>>>>>>>>>>>>>>>>>>>>>>>>>>>>>>>>>>>>>>>>>>>>>>>>>>>>>>>>>>>>>>>>>>>>>>>>>>>>>>>>>>>>>>>>>>>>>>>>>>>>>>>>>>>>>>>>>>>>>>>>>>>>>>>>>
>>>>>>>>>>>>>>>>>>>>>>>>>>>>>>>>>>>>>>>>>>>>>>>>>>>>>>>>>>>>>>>>>>>>>>>>>>>>>>>>>>>>>>>>>>>>>>>>>>>>>>>>>>>>>>>>>>>>>>>>>>>>>>>>>>>>>>>>>>>>>>>>>>>>>>>>>>>>>>>>>>>>>>>>>>>>>>>>>>
%>sMNYr
>>>>>>>>>>>>>>>>>>>>>>>>>>>>>>>>>>>>>>>>>>>>>>>>>>>>>>>>>>>>>>>>>>>>>>>>>>>>>>>>>>>>>>>>>>>>>>>>>>>>>>>>>>>>>>>>>>>>>>>>>>>>>>>>>>>>>>>>>>>>>>>>>>>>>>>>>>>>>>>>>>>>>>>>>>>>>>>>>>
>>>>>>>>>>>>>>>>>>>>>>>>>>>>>>>>>>>>>>>>>>>>>>>>>>>>>>>>>>>>>>>>>>>>>>>>>>>>>>>>>>>>>>>>>>>>>>>>>>>>>>>>>>>>>>>>>>>>>>>>>>>>>>>>>>>>>>>>>>>>>>>>>>>>>>>>>>>>>>>>>>>>>>>>>>>>>>>>>>
>>>>>>>>>>>>>>>>>>>>>>>>>>>>>>>>>>>>>>>>>>>>>>>>>>>>>>>>>>>>>>>>>>>>>>>>>>>>>>>>>>>>>>>>>>>>>>>>>>>>>>>>>>>>>>>>>>>>>>>>>>>>>>>>>>>>>>>>>>>>>>>>>>>>>>>>>>>>>>>>>>>>>>>>>>>>>>>>>>
>>>>>>>>>>>>>>>>>>>>>>>>>>>>>>>>>>>>>>>>>>>>>>>>>>>>>>>>>>>>>>>>>.

90
វដថទឹក
រយេះលពល១លោង
៉ោ (៥០នាទី)
!>vtßbú MNg
-Bnül;)anBIdMeNIrénTwkenAkñúgFmµCati
-B’Nna)anBIsar³sMxan;énvdþTwkcMeBaHCIvitenAelIEpndI.
@>smÖar El,gsikSa ¬katrUbPaB nigxøwmsar rbs; SEAL nigkatrUbPaBrbs; STEPSAM 2 ¦
#>skmµPaBeron nigbeRgon
skmµPaBRKU xøwmsar skmµPaBsisS
CMhanTI ! ¬@ naTI¦
CMhanTI @ ¬ naTI¦
-EcksisSCaBIrRkum ENnaMssi Selg -esckþIepþIménemeronvdþTkw -sisSGgÁúytamRkumelgEl,gsikSa
El,gsikSaBIvdþTkw
-El,gsikSacb; sYrsisS -El,gvdþTwk edayeRbIRKab;LúkLak;
÷etIBkY eKyl; nigdwgGVIxøHeRkay nig katrUbPaBEdlmanGkSr ÷dwgBIdMeNIrénTwkenAelIEpndI
eBlelg El,gsikSaenHehIy? BIclnavilcuHeLIgénTwk>>>
÷etIEl,genHcg;bgðajGñkBIGVI? ÷BIdMeNIrénTwkenAelIEpndI>>>
CMhanTI # ¬ naTI¦
sMNYrKnøwH etIdMeNIrénTwkenAkñgú FmµCatimanlMnaMdUcemþc? etImansar³sMxan;dUcemþcxøHcMeBaHCIvitenAelIEpndI?
-bgðajpÞaMgrUbPaBvdþTwk
edaybitcMNgeCIg ehIysYrsisS
÷etIpÞaMgrUbPaBenHbgðajBIGVI? ÷bgðajBIdMeNIrénTwk karbþrÚ rUbragén
cUrBnül;. TwkBI rwg rav ]sµ½n>>>
÷etIpÞaMgrUbPaBmanTak;TgGVICamYy ÷dMeNIrkarénTwkkñúgFmµCati
El,gsikSamun?
cUrElgEl,gmYyEdlTak;TgeTAnwgdMeNIrkarénTwk
-Bnül;sisSBIdMeNIrkarEl,gsikSa
-EcksisSCaRkum mYyRkum %nak;
Eckkat rUbPaB[RkumsisS sisSmñak;
TTYl)ankat %snøwk
-RkumsisScab;epþImelgEl,gRKU -sisSelgEl,gkatrUbPaBGñkEdl
segáttamRkum Gs;katmuneKCaGñkQñH

91
-Ecksnøkw kic©karmYysnøwkmYyRkum -sisSbMeBjsnøkw kic©karedayEp¥kelI
El,gkatrUbPaBEdl)anelgrYcehIy
CMhanTI $ ¬ naTI¦
-eRCIserIsxømw sarsMxan;²enAkñgú snøwk Tenø dI
kic©karBIdMeNIrénTwk sresrelIkaþ r mnusS
rukçCati BBk
exonehIy sYrsisS
÷Twkkñgú BBk)anBINa bwg ÷ stV mhasmuRT
Tenø
ehIyRtLb;eTANa? bwg
rukçCati
÷Twkkñgú Tenø)anBINa BBk Tenø
mhasmuRT
ehIyRtLb;eTANa? dI
GayEs,r
÷Twkkñgú rukçCati)anBINa
bwg GayEs,r
ehIyRtLb;eTANa? dI
-ENnaMsisS[ykcitTþ ku dak;eTA
elIrhM YtrvagTwk nigBBk. BBk
dI rukçCati stV

CMhanTI % ¬ naTI¦
-[sisSsresrBIdMeNIrkarénvdþTwk kic©kareFVeI nApÞH -sisSkt;RtasMNYr

92
សនលក
ឹ កច
ិ ចការ
ម្មឹះ .........................
៣ ដាំមណើរការថ្នទក

២ ោរេៈសាំខានថ្់ នទឹកសរមាបភា
់ វេៈរស់ មានជីវតិ មលើផផនដី

១ សាំណួរ

93
វិធីរលងតលែងវដតទឹក

កបង១ហ
ុ ូោន ៥០ សនលឹក ដដលោន រូបភាពរបភព ចាំនួន១០ ខុសៗោប។
ទឹកកបងពពក
ុ ៥, ទឹកលរកាមដី ៥, ទឹកកបងដី
ុ ៥, ទឹកកបងទលនល
ុ ៥, ទឹកកបងប
ុ ឹង ៥, ទឹកកបងមហាសមុ
ុ រទ ៥, ដផន
ទឹកកក ៥, ទឹកកបងរុ
ុ កខជាតិ ៥, ទឹកកបងខល
ុ នសតវ
ួ ៥, និង ទឹកកបងខល
ុ នមនុ
ួ សស ៥ ។

១. ដាំបូងរចបល់កាតចូលោប

២. ដចកកាត លោយផាេប់រូបភាពចុេះលរកាម។ កបងអប


ុ កលលងោបក់ទទួលបានកាតចាំនួន ៥
(កាតទទួលបាន អាស្ស័យលលើចាំនួនអបកលលង)

៣. ោក់កាតសល់ ផាេប់ចុេះលរកាម ចាំកណា


ថ លវង់ លលង បនាធប់មកលបើកសនលឹកខាងលលើបងែស់)

៤. អបកលលងទី១ លរជើសលរសកាតរបស
ើ ់ោត់មួយសនលឹក លដើមផីភាជប់ លៅនឹងកាត កបងច
ុ ាំនុចទី៣។
ឧទរហណ្៍ កាតទ១
ី « ពពក » កបងច
ុ ាំនុចទី៣។ អបកលលងទី១ អាចលរសយក
ើ « ទឹកកបងដី
ុ » លហើយពនយល់
របភពទឹកលចញពី ពពក លៅ ដី (ពីពពក លៅជាទឹកលភលៀង លហើយធាលក់ចុេះប៉ោេះដផធដី បនាធប់មករជាបចូលកបងដី
ុ )

94
៥. អបកលលងទី២ លរសកាតកប
ើ ងដដរបស
ុ ់ោត់មួយ លដើមផីភាជប់ជាមួយកាតកបង
ុ ចាំនុចទី៤
ឧទហរណ្៍ កាតអបកលលងទី១ ជារូប «ទឹកកបងដី
ុ » ដូលចបេះអបកលលងទី ២ លរសយក
ើ «ទឹកកបងរុ
ុ កខជាតិ» លហើយបក
ស្សាយ (រុកខជាតិស្សូបយកទឹកពី ដីតាមឫសរបស់វា)។របសិនលបើសោជិកដដទលទៀតមិនយល់រពម ជាមួយ
ការបកស្សាយរបស់ោត់លទ អបកលលងរតូវលរសកាតលផសងលទៀត
ើ អបកលលងអាចពាោមពនយល់អបកលលងដដទ
លទៀត លៅលពលពួកលគមិនយល់រពម។

របសិនលបើអបកលលងទ២
ី ពុាំោនកាតលដើមផីភាជប់ជាមួយ « ទឹកកបងដី
ុ » ោត់រតូវបដនទមកាតមួយលទៀត ។

៦. អបកលលងទង
ាំ អស់បនថលលង ភាជប់កាតមថងមួយៗ។ ចុងលរកាយ អបកលលងមួយនាក់ លរបើអស់កាត ពីដដ លនាេះ
ោត់គឺជាអបកឈបេះ។

៧. លរកាយលពលលលងចប់ អបកលលងទង
ាំ អស់សួរបញ្ញចក់ោបពីចលនារបស់ទឹក ពីចាំណ្ុចចាប់លផថើមលៅចាំណ្ុច
បញ្ច ប់។

ូ ីធីសាស្រសថកងការលលងដលផងវដថ
ចាំណ្៖ាំ លោករគូ អបករគូអាចបថរវ បុ ទឹកតាមដដលអាចលធវើបាន។

95
២. តផនការអនុវរតការសិការាវរជាវរមររៀន
រៅតាម
មជឈមណឌលគរ ុរកាសលយភូមិភ្នគ រាជធានី រេរត
ប្ក្ុមសិក្ាស្សាវប្ាវនមនរៀន្ (ប្ក្ដាសA4)

មជឈមណឌលគរ ុរកាសលយភូមិភ្នគ រាជធានី ភាំរពញ

រ.ល ល្មេះ តួនាទី/មុខវ ិជាជ

១ ស្សី វុធ នាយករង

២ ហូ លៅម៉ោូលីន អនុរបធានការោល
ិ ័យសិកា

៣ វណ្តលី វឌណនារ ី រគូឧលទធសជីវវ ិទា

៤ លសៀន ស្សីលដត រគូឧលទធសជីវវ ិទា

៥ គីម ទូរផល
ី រគូឧលទធសដផនដីវ ិទា

៦ លុក សូលីនោ រគូឧលទធសដផនដីវ ិទា

៧ គួន បួយ រគូឧលទធសដផនដីវ ិទា

៨ សាាំង ឆុម រគូឧលទធសរូបវ ិទា

៩ ឯក លម
ី រគូឧលទធសរូបវ ិទា

១០ ស្សុន លសៀងបួរ រគូឧលទធសរូបវ ិទា


ហតទលលខា

១១ សុខ រតាយ រគូឧលទធសគីមី

១២ លោក ស្សី វុធ

* អាចលរៀបចាំឱ្យបានលរចើនរកុមកាន់របលសើរ។

99
កាលវិភ្នគអនុវរតការសិការាវរជាវរមររៀន
មជឈមណឌលគរុមកាសលយភូមិភាគរាជធានភ
ី មនាំ ពែ
២០១១ ២០១២
សបាថហ៍
១០ ១១ ១២ ១ ២ ៣

លរៀបចាំកិចដច តងការបលរងៀន លរៀបចាំកិចដច តងការបលរងៀន បលរងៀនជាមួយគរុសិសស


Tool 1 Tool 1

បលរងៀនជាមួយគរុសស
ិ ស

របជុាំដកលមែកច
ិ ដច តងការ បលរងៀនសាកលផង របជុាំដកលមែកច
ិ ដច តងការ បលរងៀនសាកលផង
បលរងៀនTool 1 Tool 2 បលរងៀន Tool 1 Tool 2

មុខវ ិជាជ៖ ជីវវ ិទាថាបក់ទី8 រូបវ ិទា ថាបក់ទី8


របធានបទ៖ ដាំណ្ឺកនាាំកបុងរុកខជាតិ ចាប់ទី1ញូតុន
អបកទទួលរបធានបទ៖ លសៀន ស្សីលដត ស្សុន លសៀនបួរ

100
២០១២
សបាថហ៍
៤ ៥ ៦ ៧ ៨
លរៀបចាំកិចដច តងការបលរងៀន
Tool 1

របជុាំដកលមែកិចដច តងការបលរងៀន
Tool 1

បលរងៀនជាមួយគរុសិសស

បលរងៀនសាកលយង Tool 2

មុខវ ិជាជ៖ ដផនដីវ ិទា ថាបក់ទី8


របធានបទ៖ វគគរពេះចនធ
អបកទទួលរបធានបទ៖ គឹម បូរ ីផល

101
ប្ក្ុមសិក្ាស្សាវប្ាវនមនរៀន្ (ប្ក្ដាសA4)

មជឈមណឌល គរ ុរកាសលយ ភូមិភ្នគ បារ់ដំបង

រ.ល ល្មេះ តួនាទី/មុខវ ិជាជ

១ លោកស្សី ផាវាន
៉ោ ់ សាលកឿត នាយិកា

២ លោក សុខ ហុីង នាយករង

៣ លោក សាំ លអឿត រូបវ ិទា

៤ លោកស្សី អន សុោវាចា ជីវវ ិទា

៥ លោក ោស សាទុាំ គីមី

៦ លោកស្សី លសា គឹមលហៀក ជីវវ ិទា

៧ លោក ទុយ លរត


៉ោ រូបវ ិទា

៨ លោក ជាប ្ងវុ ិត ដផនដីវ ិទា

៩ លោក ឃុន មុនី ជីវវ ិទា

១០ លោកស្សី លម៉ោេះ ចាន់លែត រូបវ ិទា

១១ លោកស្សី លសង លហឿង គីមី

១២ លោកស្សី សារ ី ម៉ោូនីកា ជីវវ ិទា


ហតទលលខា

១៣ លោកស្សី លទស សុផារាន


៉ោ ់ ដផនដីវ ិទា

លោក សុខ ហុង



១៤ លោកស្សី ដខម សារត
ិ គម
ី ី

* អាចលរៀបចាំឱ្យបានលរចើនរកុមកាន់របលសើរ។

102
កាលវិភ្នគអនុវរតការសិការាវរជាវរមររៀន
មជឈមណឌលគរុមកាសលយភូមិភាគ មខ្តេ បាតដ
់ ាំបង្
២០១១ ២០១២
សបាថហ៍
១០ ១១ ១២ ១ ២ ៣

ចាត់តាង
ាំ រគូលធវក
ើ ិចដច តងការ បលរងៀនសាកលផងរកុម ឆលងកិចដច តងការ បលរងៀនជាមួយគរុសិសស លធវរើ បាយការណ្៍ឆោនទី១ ចាត់តាង
ាំ រគូបនថ
គីមី ៖ សារ ិត សិកាស្សាវរជាវលមលរៀន រូប៖ រូប ៖ ជីវៈ ៖ ម៉ោូនកា


ដផនដី៖ ្ង វុ ិត គីមី និងដផនដីវ ិទា ជីវ៖ ជីវ ៖ គីមី ៖ លសង លហឿង

ឆលងកិចដច តងការ បលរងៀនជាមួយគរុសស


ិ ស បលរងៀនសាកលផង របជុាំ ជាមួយ ឆលងកិចដច តងការ
គីមី៖ និងចាត់តាង
ាំ រគូលធវក
ើ ិចដច តង រូប ៖ សាោអនុវតថន៍ និងសាោ ជីវ ៖

ដផនដី៖ រូប ៖ ទុយ លរ ៉ោត ជីវ ៖ សហការ គីមី ៖
ជីវ ៖ លសា លសងលហៀក
មុខវ ិជាជ៖
របធានបទ៖
អបកទទួលរបធានបទ៖

103
២០១២
សបាថហ៍
៤ ៥ ៦ ៧ ៨

បលរងៀនសាកលផង បលរងៀនជាមួយគរុសិសស
ជីវ ៖ ជីវ ៖
២ គីមី ៖ គីមី ៖

លធវរើ បាយការណ្៍ឆោសទី២

របជុាំរួម ជាមួយសាោអនុវតថន៍
និងសាោសហការ

មុខវ ិជាជ៖
របធានបទ៖
អបកទទួលរបធានបទ៖

104
ប្ក្ុមសិក្ាស្សាវប្ាវនមនរៀន្ (ប្ក្ដាសA4)

មជឈមណឌលគរ ុរកាលយភូមិភ្នគ រេរត កំពង់ចម

រ.ល ល្មេះ តួនាទី/មុខវ ិជាជ

១ ដវ ៉ោន សុីផាន នាយករង

២ ហួ រូនបី នាយិការង

៣ សុន ឈុនលឡង រគូឧលទធស ជីវវ ិទា

៤ អាង វ ិបុល រគូឧលទធស ជីវវ ិទា

៥ ហុឹង សុខចាន់ រគូឧលទធស រូបវ ិទា

៦ លី ពិសិដឌ រគូឧលទធស រូបវ ិទា

៧ លហង ណាលីន រគូឧលទធស ដផនដីវ ិទា

៨ លី សុខដណ្ រគូឧលទធស គីមី

៩ លហង ឌីណា រគូឧលទធស គីមី

១០ អូអុី កាសាឺហ
ុ ុីរូ ៉ោ JOCV
ហតទលលខា

១១

ល្មេះ ដវ ៉ោន សុីថាន


១២

* អាចលរៀបចាំឱ្យបានលរចើនរកុមកាន់របលសើរ។

105
កាលវិភ្នគអនុវរតការសិការាវរជាវរមររៀន
មជឈមណឌលគរុមកាសលយភូមិភាគ មខ្តេ កាំពង្់ចម
២០១១ ២០១២
សបាថហ៍
១០ ១១ ១២ ១ ២ ៣
ដកលមែការបលរងៀន ការបលរងៀនជាមួយ
សាកលផង គរុសិសស

ដែៃទី ៣

-ការរបជុាំសថីពីការសិកាមុខ -ការរបជុាំសថីពីការសិកា -ការរបជុាំសថីពីការសិកា


វ ិជាជលមលរៀន មុខវ ិជាជលមលរៀន មុខវ ិជាជលមលរៀន

ដែៃទី ១០ ដែៃទី ១៤ ដែៃទី ១៣

ដកលមែកច
ិ ដច តងការបលរងៀន -ការរបជុាំសថីពីការសិកា -ការរបជុាំសថីពីការសិកា
ដែៃទី ២០ មុខវ ិជាជលមលរៀន មុខវ ិជាជលមលរៀន

ដែៃទី ១៥ ដែៃទី ១៣

-ដកលមែកិចដច តងការ ដកលមែការបលរងៀន ការបលរងៀនជាមួយ

បលរងៀន សាកលផង គរុសិសស



ដែៃទី ២៦

មុខវ ិជាជ៖ រូបវ ិទា - ជីវវ ិទាថាបក់ទី៩ - គីមីទី៩ - ជីវវ ិទាថាបក់ទី៨ - គីមីទី៨ - ជីវវ ិទាថាបក់ទី៧
របធានបទ៖ អាង វ ិបុល (លី សុខដណ្) អាង វ ិបុល+សុនឈុនលឡង (លី សុខដណ្) អាង វ ិបុល
អបកទទួលរបធានបទ៖ ហុង
ឹ សុខចាន់ សុនឈុនលឡង - ដផនដីវ ិទាថាបក់ទី៩ - រូបវ ិទាថាបក់ទី៨ - ដផនដីវ ិទាថាបក់ទី៨ សុនឈុនលឡង
- រូបវ ិទាថាបក់ទី៩ (លហង ណាលីន) ហុង
ឹ សុខចាន់ (លហង ណាលីន) - រូបវ ិទាថាបក់ទី៧
(ហុង
ឹ សុខចាន់ +លី ពិសិដខ) លី ពិសិដខ ហុង
ឹ សុខចាន់
សុន ឈុនលឡង លី ពិសិដខ

106
២០១២
សបាថហ៍
៤ ៥ ៦ ៧ ៨
ដកលមែការបលរងៀនសាកលផង
ដែៃទី ៧

របជុាំសថីពីការសិសាស្សាវរជាវលម ការបលរងៀនជាមួយគរុសិសស
លរៀន
៣ ដែៃទី ២២

ដកលមែកិចដច តងការបលរងៀន
ដែៃទ២
ី ១

មុខវ ិជាជ៖ គីមទ


ី ី ៧ (លី សុខដណ្)
របធានបទ៖ ដផនដីទី៧ (លហង ណាលីន)
អបកទទួលរបធានបទ៖

107
ប្ក្ុមសិក្ាស្សាវប្ាវនមនរៀន្ (ប្ក្ដាសA4)

មជឈមណឌល គរ ុរកាសលយ ភូមិភ្នគ រេរត ថ្រពតវង

រ.ល ល្មេះ តួនាទី/មុខវ ិជាជ

១ ឈិត វាន
៉ោ ់នី គីមី

២ លមឿង វណាត គីមី

៣ អុន ណ្ុល រូបវ ិទា

៤ ទវ វុឌណី រូបវ ិទា

៥ ជួប តុងលហង រូបវ ិទា

៦ វណ្ីឍ ផោល រូបវ ិទា

៧ សាន សុទន ជីវវ ិទា

៨ នាក់ វ ិចិរត ជីវវ ិទា

៩ ដកវ លសរ ី ដផនដីវ ិទា

១០ ដហម វន់ ដផនដីវ ិទា


ហតទលលខា

១១ លមឿន លសឿន ដផនដីវ ិទា

១២ ោត ផានិត ដផនដីវ ិទា លោក បួយ វុទនី

* អាចលរៀបចាំឱ្យបានលរចើនរកុមកាន់របលសើរ។

108
កាលវិភ្នគអនុវរតការសិការាវរជាវរមររៀន
មជឈមណឌលគរុមកាសលយភូមិភាគ មខ្តេ ថ្រពផវង្
២០១១ ២០១២
សបាថហ៍
១០ ១១ ១២ ១ ២ ៣

រពឹក LS សាោ រពឹក LS សាោ រពឹក LS សាោ រពឹក LS សាោ រពឹក LS សាោ
ោៃច IBL ជាមួយសាោ ោៃច IBL ជាមួយសាោ ោៃច IBL ជាមួយសាោ ោៃច IBL ជាមួយសាោ ោៃច IBL ជាមួយសាោ

សហការ សហការ សហការ សហការ សហការ
ដែៃទី ៣០ ដែៃទី ២៨ ដែៃទី ២៥ ដែៃទី ២៩ ដែៃទី ២៨
មុខវ ិជាជ៖ ជីវ ៖ លរផលច
ិ គីមី ៖ ភាបក់ រកោចត់រជេះ ជីវ ៖ ដាំណ្ឹកនាាំកបុងសលឹក គីមី ៖ លាយ ជីវ ៖ ការសលងេតផាេ
របធានបទ៖ កញ្ញដ សាន សុទន លោក លមឿង វណាត លោក នាក់ វ ិចរិ ត ឈិត វា៉ោនន
់ ី កញ្ញដ សាន សុទន
អបកទទួលរបធានបទ៖ រូបវ ិទា ៖ ចរនថវ ិលវល់ ដផនដី ៖ ដលផងសថីពីវដថទឹក រូបវ ិទា ៖ និចលភាព ដផនដី ៖ វគគរពេះចនធ រូបវ ិទា ៖ សោពធអងគធាតុរាវ
លោក អន
ុ ណ្ុល លោក សាត ផានិត លោក ទវ វុឌណី កញ្ញដ ដកវ លសរ ី លោក ជួប តុងលហង

109
២០១២
សបាថហ៍
៤ ៥ ៦ ៧ ៨

រពឹក LS សាោ
ោៃច IBL ជាមួយសាោសហ
៤ ការ
ដែៃទី ៣០

មុខវ ិជាជ៖ គីមី ៖ ការកាំណ្ត់ដង់សលុី តអ. រាវ


របធានបទ៖ លោក លមឿង វណាត
អបកទទួលរបធានបទ៖ ដផនដី ៖ កាំណ្ត់រតាចលនរព.អទិ
លោក លមឿន លសឿន

110
ប្ក្ុមសិក្ាស្សាវប្ាវនមនរៀន្ (ប្ក្ដាសA4)

មជឈមណឌល គរ ុរកាសលយ ភូមិភ្នគ រេរត តាតកវ (រកុម រ ូប គីមី)

រ.ល ល្មេះ តួនាទី/មុខវ ិជាជ

១ លោក គូ សុផា របធាន

២ លោក ឱ្ម ចនធសុខា អនុរបធាន

៣ លោក លសៀន លផង សោជិករកុម

៤ លោក លអាម ភាព សោជិករកុម

៥ លោក លទព យុិត សោជិករកុម

៦ លោម ែម សុថារទន
ិ សោជិករកុម

១០ ហតទលលខា

១១

លោក ម៉ោក់ ចនបី


១២

* អាចលរៀបចាំឱ្យបានលរចើនរកុមកាន់របលសើរ។

111
ប្ក្ុមសិក្ាស្សាវប្ាវនមនរៀន្ (ប្ក្ដាសA4)

មជឈមណឌល គរ ុរកាសលយ ភូមិភ្នគ រេរត តាតកវ រកុម (ជីវ តផនដី)

រ.ល ល្មេះ តួនាទី/មុខវ ិជាជ

១ លោក ដប៉ោន រទន


ឹ ី របធាន

២ លោក ដង៉ោត សូោ អនុរបធាន

៣ លោក លសា សាលរត សោជិក

៤ កញ្ញដ ចាន់ វណ្តរ ី សោជិក

៥ កញ្ញដ ប៉ោុក ស្សីទូច សោជិក

១០ ហតទលលខា

១១

លោក ម៉ោក់ ចនបី


១២

* អាចលរៀបចាំឱ្យបានលរចើនរកុមកាន់របលសើរ។

112
កាលវិភ្នគអនុវរតការសិការាវរជាវរមររៀន
មជឈមណឌលគរុមកាសលយភូមិភាគ មខ្តេ ាផកវ
២០១១ ២០១២
សបាថហ៍
១០ ១១ ១២ ១ ២ ៣
ដកលមែកិចដច តងការ អនុវតថ និងសលងេតលមលរៀន បលរងៀនសាកលផង និង
ដែៃទី ៣ កបងថា
ុ ប ក់ និងពិភាកា ពរ ងកិចដច តងការ

ដែៃទី ៥ ដែៃទី ១

បលរងៀនសាកលផង និង ដកលមែកិចដច តងការបលរងៀន


ពរ ងកិចដច តងការ និងលរតៀមសោភរ

ដែៃទី ៨ ដែៃទី ៩

មុខវ ិជាជ៖ រូប ៖ លសៀន លផង រូប ៖ លសៀន លផង រូប ៖ លសៀន លផង ជីវ ៖ ដប៉ោន រ ឹទនី
របធានបទ៖ ដផនដី ៖ ប៉ោុក ស្សីទូច ដផនដី ៖ ប៉ោុក ស្សីទូច ដផនដី ៖ ប៉ោុក ស្សីទូច គីមី ៖ លទព យុិត
អបកទទួលរបធានបទ៖

113
២០១២
សបាថហ៍
៤ ៥ ៦ ៧ ៨

អនុវតថ និងសលងេតលមលរៀនកបង

ថាបក់ និងពិភាកា
២ ដែៃទី ១០

មុខវ ិជាជ៖ ជីវ ៖ ដប៉ោន រ ឹទនី


របធានបទ៖ គីមី ៖ លទព យុិត
អបកទទួលរបធានបទ៖

114
ប្ក្ុមសិក្ាស្សាវប្ាវនមនរៀន្ (ប្ក្ដាសA4)

មជឈមណឌលគរ ុរកាសលយភូមិភ្នគ រេរត កណ្ត


ត ល

រ.ល ល្មេះ តួនាទី/មុខវ ិជាជ

១ លោក លអឿ សុខលម៉ោង របធាន

២ លោក ថាក់ សុភា របធានរកុម គីមី

៣ លោក លប របា គម
ី ី

៤ លោក ជួន ភា គម
ី ី

៥ លោក លម៉ោង ភិរុណ្ របធានរកុមរូបវ ិទា

៦ លោក សាង សីហា រូបវ ិទា

៧ លោក លសៀន បុនលធឿន រូបវ ិទា

៨ កញ្ញដ ហូ ច័ននសារា ៉ោ របធានរកុមជីវវ ិទា

៩ លោកស្សី ឡង
ុ ពននិនី ជីវវ ិទា

១០ លោកស្សី លសាម ោនី ជីវ ិទា ហតទលលខា

១១ លោកស្សី ស្សី ច័នធលសាភា ដផនដីវ ិទា

លោក សាន់នី សារម


៉ោូ
១២ កញ្ញដ លវ ៉ោង ស្សីមន ដផនដីវ ិទា

* អាចលរៀបចាំឱ្យបានលរចើនរកុមកាន់របលសើរ។

115
កាលវិភ្នគអនុវរតការសិការាវរជាវរមររៀន
មជឈមណឌលគរុមកាសលយភូមិភាគ មខ្តេ កណ្
េ ល
២០១១ ២០១២
សបាថហ៍
១០ ១១ ១២ ១ ២ ៣
ពរ ងកិចដច តងការ លរៀបចាំសោភរៈ ពរ ងកិចដច តងការ

ដកលមែកិចដច តងការ បលរងៀនសាកលផ និងដក ពរ ងកិចដច តងការ ដកលមែកិចដច តងការ


លមែ

ដកលមែកច
ិ ដច តងការ បលរងៀនសាកលផ និងដក
លមែ

បលរងៀនគរុសស
ិ ស និងដក លរៀបចាំសោភរៈ បលរងៀនគរុសស
ិ ស និងដក បលរងៀនសាកលផ និងដក
លមែ លមែ លមែ

មុខវ ិជាជ៖ រូបវ ិទា គីមី ជីវវ ិទា


របធានបទ៖ លោក លម៉ោង ភិរុណ្ លោក ជួន ភា លោកស្សី លសាម ោនី
អបកទទួលរបធានបទ៖ ទីរបជុាំទមៃន់ វ ិធដី ញកលាយ លោយការ
លរចាេះ និងកាំណ្រកាម

116
២០១២
សបាថហ៍
៤ ៥ ៦ ៧ ៨
បលរងៀនគរុសិសស និងដកលមែ

ពរ ងកិចដច តងការ បលរងៀនសាកលផង និងដកលមែ

ដកលមែកិចដច តងការ

លរៀបចាំសោភរៈ បលរងៀនគរុសិសស និងដកលមែ

មុខវ ិជាជ៖ ដផនដីវ ិទា


របធានបទ៖ កញ្ញដ លវ ៉ោន ស្សី មន
អបកទទួលរបធានបទ៖ ចនធរោស និងសូរយរោស

117

You might also like